Sie sind auf Seite 1von 197

1988 Which of the statements above are correct?

SSCE PHYSICS A. I and II only


PAPER 2 OBJECTIVES B. II and III only
C. II, III and IV only
Answer all the questions. D. I, II, III and IV
E. I, III and IV only
1. Which of the units of the following physical
quantities are derived? 6. A ball of mass 0.5kg moving at 10ms-1
I. Area collides with another ball of equal mass at
II. Thrust rest. If the two balls move off together after
III. Pressure the impact, calculate their common velocity.
IV. Mass A. 0.2ms-1
A. I, II, III and IV B. 0.5ms-1
B. I, II and III only C. 5.0ms-1
C. I, II and IV only D. 10.0ms-1
D. I and III only E. 20.0ms-1
E. I and IV only
7. A body is projected into space with an initial
2. The slope of a straight line displacement velocity Vo at an angle θ to the horizontal.
time graph indicates the Which of the following statements is/are
A. distance traveled correct about its motion?
B. uniform velocity I. The total time of flight is VoCos θ
C. uniform acceleration g
D. acceleration at an instant
E. uniform speed
II. The maximum heights is VoSin2 θ
3. A man will exert the greatest pressure on a 2g
bench when he
A. lies flat on his back III. The range is Vo2Cos2 θ
B. lies flat on his belly g
C. stands on the both feet A. I and II only
D. stands on one foot only B. I and III only
E. stands on the toes of one foot C. II and III only
cm D. II only
6 7
E. III only
0 5
8. The diagram below shows a body resting on
an inclined plane. If the body slides down
4. The diagram above represents a portion of a the plane, what will be its acceleration? (g =
venire caliper. What is its reading? 10ms-2)
A. 7.10cm
A. 27.5ms-2 25N
B. 7.09cm
C. 6.90cm B. 14.8ms-2
300
D. 6.34cm C. 10.5ms-2
E. 6.29cm D. 6.2ms-2 15kg
E. 3.3ms-2
5. When three coplanar non-paralled forces
are in equilibrium 9. A catapult is used to project a stone. Which
I. they can be represented in magnitude and of the following energy conversions takes
direction by the three sides of a triangle place as the stone is released?
taken in order A. The Kinetic energy of the stone is converted
II. their lines or action meet at a point into gravitational potential energy
III. the magnitude of any one force equals the B. The gravitational potential energy of
magnitude of the resultant of the other two catapult is converted into the Kinetic energy
IV. any one force is the equilibrant of the other of the stone
two.
C. The gravitational potential energy is E. Change in colour with temperature
converted into elastic potential energy 15. The clinical thermometer is characteristics
D. The elastic potential energy of catapult is by having a
converted into gravitational potential A. wide range of temperature
energy of the stone B. wide bore
C. long stem
10. A ball is thrown vertically upwards from the D. contrition
ground with an initial velocity of 50cms-1. E. narrow bore
What is the total time spent by the ball in
the air? (g = 10ms-2) 16. A given mass of gas has a pressure of
A. 2.5s 80Nm-2 at a temperature of 47˚C. If the
B. 5.0s temperature is reduced to 27˚C with the
C. 10.0s volume remaining constant, the new
D. 15.0s pressure is
E. 20.0s A. 46.0Nm-2
B. 75.Nm-2
11. Which of the following correctly gives the C. 80.0Nm-2
relationship between linear speed v and D. 85.3Nm-2
angular speed w of a body moving uniformly E. 139.4Nm-2
in a circle of radius?
A. v = wr 17. Which of the following is a reason why a
B. v = w 2r concrete floor feels colder to the bare feet
C. v = wr2 than a mat on the same floor during the
D. v2 = wr rainy season?
E. v = w/r A. Mat is a better conductor of heat than the
feet
12. The motion of a body is simple harmonic if B. Mat loses heat to the bare feet at a faster
the rate than concrete floor
A. acceleration is always directed towards a C. Mat loses heat to the bare feet while the
fixed point concrete floor extracts heat from them
B. path of motion is a straight line D. Concrete floor is a better conductor of heat
C. acceleration is directed towards a fixed than the mat
point and proportional to its distance from E. Mat is a better conductor of heat than the
the point floor
D. acceleration is proportional to the square of
the distance from a fixed point 18. In which of the following is the expansion of
E. acceleration is constant and directed solids a disadvantage?
towards a fixed point A. The fitting of wheels in rims
B. Fire alarms
13. Which of the following statements about the C. The thermostat
gaseous state of matter is not correct? D. The bimetallic thermometer
The molecules of a substance in gaseous E. The balance wheel of a watch
state
A. are in a constant state of motion 19. Which of the following best explains why a
B. have different speeds person suffers a more severe burn when his
C. have a temperature which is a measure of skin exposed to steam than when boiling
the average kinetic energy water is poured onto his skin?
D. have no intermolecular attractive force A. Steam is at a higher temperature than
E. have a pressure which is determined by the boiling water
number of molecules present B. Steam posse’s greater heat energy per unit
14. Which of the following cannot be used to mass than boiling water
measure the temperature of a substance? C. Steam spreads more easily over a wider
A. Variation of pressure with temperature area of the skin than boiling water
B. Expansivity of a liquid D. Steam penetrates more deeply into the skin
C. Change in resistance of a conductor than boiling water
D. Thermo-electric effect
E. The specific latent heat of vaporization is 25. Which of the following statements is/are not
released in changing from boiling water to correct about the image formed by a plane
steam mirror?
I. The magnification produced is I
20. Which of the following statements about II. The image distance is the same as the
radiant heat is/are not correct? object distance
I. Radiant heat cannot travel through a III. The image is real
vacuum IV. The image is laterally inverted
II. Rough surfaces emit radiant heat better A. I only
than polished surfaces B. II only
III. Dark surfaces absorb radiant heat better C. III only
than bright surfaces D. I and III only
A. I only E. II and IV only
B. II only
C. III only
D. I and II only
E. II and III only

21. 0.5kg of water at 10˚C is completely 26. What will be the characteristics of the image
converted to ice at 0˚C by extracting of the object OB shown above after
1880001 of heat from it. If the specific heat reflection from the mirror?
capacity of water is 4200 J Kg-10C-1, A. Diminished, real and erect
calculate the specific latent heat of fusion of B. Magnified, real and inverted
ice. C. Diminished, virtual and erect
A. 9.0kJkg-1 D. Magnified, virtual and erect
B. 84.0kJ kg-1 E. Magnified virtual and inverted
C. 168.0kJ kg-1
D. 334.0kJ kg-1 27. A transparent rectangular block 5.0cm thick
E. 336.0kJ kg-1 is placed on a black dot. The dot when
viewed from above, is seen 3.0cm from the
22. The temperature t which the water vapour top of the block. Calculate the refractive
present in the air is just sufficient to index of the material of the block
saturate it is called the A. 2/5
A. ice point B. 3/5
B. boiling point C. 3/2
C. steam point D. 5/3
D. dew point E. 5/2
E. saturation point
28. The velocities of light in air and glass are
23. Which of the following instruments may be 3.0 x 108ms-1 and 1.8 x 108ms-1 respectively.
used to measure relative humidity? Calculate the sine of the angle of incidence
A. Hydrometer that will produce an angle of refraction of
B. Barometer 30˚ for a ray of light incident on glass.
C. Manometer A. 1.2
D. Hypsometer B. 1.0
E. Hygrometer C. 0.8
D. 0.6
24. Which of the following will affect the E. 0.3
saturation vapour pressure of a liquid?
A. Temperature of the liquid 29. In an experiment to measure the focal
B. Humidity of air length f of a converging lens, object
C. Volume of vapour distance u and corresponding image
D. Volume of the liquid distances v were measured and 1/u plotted
E. Mass of the liquid against 1/v to obtain the type of graph
below:
III. It is formed by the superposition of two
progressive waves of different amplitudes
traveling in opposite directions.
A. I only
How would f be found from this graph? B. II only
A. f = the slope of the graph C. I and II only
B. f = the intercept on the 1/u – axis D. II and III only
C. f = the intercept on the 1/v – axis E. I, II and III
D. f = reciprocal of the slope
E. f = reciprocal of the intercept in either axis 35. A boy standing some distance from the foot
of a tall cliff claps his hands and hears an
30. An object is placed 36cm from a converging echo 0.5s later. If the speed of sound is
lens of focal length 24cm. If a real image 340ms-1, how far is he from the cliff?
which is 4cm high is formed, calculate the A. 17m
height of the object. B. 34m
A. 2.0cm C. 85m
B. 4.0cm D. 170m
C. 6.0cm E. 680m
D. 8.0cm
E. 10.0cm 36. A sounding tuning fork is brought near the
end of a pipe containing an air column and
31. The ability of the eye to focus objects at the loudness of the sound increases. This
different distances is called effect is due to
A. power A. diffraction
B. accommodation B. an echo
C. normal vision C. resonance
D. persistence on vision D. interference
E. long sight E. reverberation

32. The change of the direction of a wave front 37. If the leaves of a positively charged
because of a change in the velocity of the electroscope collapse completely as an
wave in another medium is called object are brought near the cap of the
A. refraction electroscope, the object possesses
B. reflection A. an equal amount of positive charge
C. diffraction B. less amount of negative charge
D. interference C. no charge
E. dispersion D. a positive charge
E. an equal amount of negative charge
33. A source of sound produces waves in air of
wavelength 1.65m. If the speed of sound in 38. A cell can supply currents of 0.80 A and
air is 330ms-1, the period of vibration in 0.40 A through a 2Ω and a 5Ω resistor
second is respectively. Calculate the internal
A. 2.00 resistance of the cell.
B. 2.0 A. 0.2Ω
C. 0.5 B. 0.4Ω
D. 0.02 C. 1.0Ω
E. 0.005 D. 3.0Ω
E. 9.0Ω
34. Which of the following statements is/are not
correct about a standing wave? 39. Which of the following diagrams represents
I. It is formed by the superposition of two the magnetic field of two isolated unlike
progressive waves traveling in the same poles?
direction
II. All points on the wave are vibrating with
maximum displacement
A.
B.
A. 11Ω
B. 36Ω
C. C. 60Ω
D. 120Ω
E. 225Ω
D.
45. Two parallel conductors carry equal currents
in the same direction. Which of the following
statements is not a correct observation?
E. A. Echo f the conductors will experience a
force
B. Each of the conductors can move
C. The forces on the conductors will be equal
40. A battery of e.m.f. 24 V and internal D. There are concentric lines of force around
resistance 4Ω is connected to a resistor of each conductor
32Ω. What is the internal p.d of the battery? E. The two conductors will repel each other
A. 24.0V
B. 21.3V 46. It takes 4 minutes to boil a quantity of water
C. 16.0V electrically. How long will it take to boil
D. 2.7V same quantity of water using the same
E. 2.0V heating coil but with the current doubled?
(Neglect any external heat losses)
41. Calculate the total heat energy developed in A. 64 minutes
5 minutes by the system above. B. 32 minutes
A. 120J C. 8 minutes
B. 144J D. 2 minutes
C. 240J E. 1 minute
D. 740J
E. 8640J 47. Which of the following statements is not
correct?
42. A transformer with 5500 turns in its primary A. magnetic field is a region in which a
is used between a 240V a.c supply and a magnetic force may be detected
120V kettle. Calculate the number of turns B. A line of force is a line along which a
in the secondary. magnetic N-pole would tend to move if it
A. 11000 were free
B. 2750 C. Magnetic fields are scalar quantities
C. 460 D. Neutral points are obtained where the
D. 232 earth’s magnetic field is exactly equal and
E. 10 opposite to that due to the magnet
E. The dip angle is the angle between the
43. Local action is normally prevented from horizontal and the earth’s total magnetic
taking place in a simple electric cell by field
A. introducing an oxidizing agent into the acid
B. not drawing continuously for long periods 48. A lamp is marked 220V 60W. Calculate the
C. amalgamating the surface of the zinc plate energy it would consume when connected
D. using a porous pot to isolate the zinc plate to a 220V source for 1 hour.
from the acid A. 216000J
E. using dilute sulphuric acid B. 132200J
44. Calculate the value of R when G shows no C. 3600J
deflection in the circuit illustrated below. D. 864J
E. 220J
49. What is the effect of using split-rings in a E. Water molecules are separated by large
simple d.c motor? spaces between them
A. The direction of rotation of the coil is
reversed 54. Which of the following explains the concave
B. The current in the coil flows in the same meniscus of water in a glass tube?
direction A. The adhesion between water and glass is
C. The current in the coil becomes alternating greater than the cohesion between water
D. The direction of the force on the coil is molecules
reversed B. The cohesion between water molecules is
E. The contact with the voltage source is cut greater than the adhesion between glass
off and water
C. The molecules of water near the glass are
50. Which of the following statements is not moving faster than the molecules in the
correct about a loaded spiral spring? other parts
A. The extension is proportional to the load D. The molecules of water at the water-air
applied, provided the elastic limit is not boundary are often attracted to the centre
exceeded of the tube
B. It the elastic limit is not exceeded, the E. The weight of the water pulls the central
contraction is proportional to the load part ofo the surface down
removed
C. Up to the yield limit, extension is no longer 55. The unit of inductance is the
proportional to the applied load A. farad
D. Beyond the elastic limit, extension is no B. henry
longer proportional to the applied load C. hertz
E. If the spring regains its shape or form after D. ampere
deformation, it is said to be elastic E. coulomb

51. Calculate the resistivity of a wire of length 56. Which of the following is/are not affected by
2m and cross-sectional area of 0.004cm2 if a magnetic field?
its resistance is 3.0ohm I. Neutrons
A. 0.0000267 ohm cm II. Cathode rays
B. 0.00006 ohm cm III. Alpha particles
C. 0.26700 ohm cm A. I only
D. 0.37500 ohm cm B. II only
E. 0.60000 ohm cm C. III only
D. II and III only
52. Which of the following particles or rays have E. I, II and III
the greatest penetrating power
A. Beta particles 57. Which of the following is not correct about
B. Alpha particles isotopes of an element? They are
C. Gamma particles A. the same neutron number
D. Electrons B. the same proton number
E. Neutrons C. the same number of electrons
D. the same chemical properties
53. When sugar at the bottom of a glass of E. different physical property
water gradually dissolves, the water level
does not change appreciably. Which of the 58. An electron of charge 1.6 x 10-19C is
following statements best explains this? accelerated in vacuum from rest at zero volt
A. Water is suitable solvent for sugar towards a plate of 40KV. Calculate the
B. The sugar and the water have become a kinetic energy of the electron.
single solution A. 4 x 10-25J
C. The volume of the sugar molecules does not B. 4 x 10-21J
change when the sugar dissolves C. 6.4 x 10-20 J
D. Sugar molecules are smaller than water D. 6.4 x 10-15 J
molecules E. 2.5 x 10-20 J
59. Which of the following is not an b State Boyle’s law. A thread of mercury of
electromagnetic radiation? length 15cm is used to trap some air in a
A. X-ray capillary tube with uniform cross-sectional
B. Radio waves area and closed at one end. With the tube
C. Sunlight vertical and the open end uppermost, the
D. Sound waves length of the trapped air column is 20cm.
E. Infra-red radiations Calculate the length of the air column when
the tube is held.
60. Two radioactive elements A and B have half- (i) Horizontally,
lives of 100 and 50 years respectively. (ii) Vertically with the open end underneath.
Samples of A and B initially contain equal (Atmosphere pressure = 76cm of mercury)
number of atoms. What is the ratio of the
number of the remaining atoms of A to that c. Explain why it is not advisible to sterilize a
of B after 200 years? clinical thermometer in boiling water at
A. 4:1 normal atmosphere pressure.
B. s:1
C. 1:1 3a. Explain with the aid of diagram how a
D. 1:2 converging lens could be used to
E. 1:4
(i) ignite a piece of carbon,
1988 (ii) produce an enlarged picture on a screen,
THEORY SECTION B PART 1 (iii) correct an eye defect.

1a. Explain with the aid of a diagram what is b. What is a mechanical wave? Describe with
meant by the moment of a force about a the aid of a diagram, an experiment to show
point. that sound needs a material medium for
transmission. State three characteristics of
b. State the condition of equilibrium for a sound and mention the factor on which each
number of coplanar parallel forces. A metre depends.
rule is found to balance at the 48cm mark.
When a body of mass 60g is suspended at 4a. Explain what is meant by:
the 6cm mark the balance point is found to (i) Electric field intensity
be at the 30cm mark. (ii) electric lines force
Calculate:
(i) The mass of the metre rule, b. Two similar but opposite point charges – q
(ii) The distance of the balance point from the and +q each of magnitude 5.0 x 10-8C are
zero end, if the body were moved to the separated by a distance of 8.0cm in vacuum
13cm mark. as shown in the diagram below.

c. Show tht the efficiency E, the force ratio M.A


and the velocity ratio V.R of a machine are
related by the equation. Calculate the magnitude and direction of
the resultant electric field intensity E at the
M.A x 100% point P. Draw the lines of force due to this
E = V.R system of charges.
The efficiency of a machine is 80%. (Take 1_ = 9 x 109 Nm2C-2)
Determine the work done by a person using 4π Σ
this machine to raise a load of 200kg
through a vertical distance of 3.0m. c. Calculate the following in the series circuit
(Take g = 10ms-2) shown beside:
(i) reactance of the capacitor
2a. Draw a labeled diagram of a vacuum flask. (ii) impedance of the circuit
Explain how its construction minimizes heat (iii) current through the circuit
exchange with the surroundings. (iv) voltage across the capacitor
(v) average power used in the circuit
board. Measure BO = y and AO. Evaluate
y/AD.

Repeat the experiment with M – 90, 110 and


130 and 150g. In each case, determine the
corresponding values of y, OA and y/AO.
Tabulate your readings.

1988 Plot your graph with M on the vertical axis


PAPER I PRACTICAL and horizontal axis and find the slope of the
graph. State two precautions you took to
You are required to record your ensure accurate results. Attach your answer
observations as soon as they are made. The script.
observations and any mathematical working
and answers to questions should be written b. (i) Given that M/2m = y/AO in the
in your answer book, scrap paper must not experiment above, deduce value of
be used. Attach your graphs to your answer the known mass m from your graph.
book. You are not expected to copy out your
work again neatly. The record may be kept (ii) State the condition necessary for the
in pencil provided it can be read clearly. If equilibrium of three non parallel
any place of the apparatus provided has a coplanar forces
label with a letter on it, this letter must be
recorded in your answer book in order that (iii)
the Examiner may identify which set of
apparatus you used.
Calculate the magnitude of force F in
Neither a detailed description of the the
apparatus nor a full account of the method diagram above, if all the three forces
of carrying out the experiments is required. acting at O, are in equilibrium.
You should however, note any special
precautions you have taken and it must be
clear (by diagrams or otherwise) exactly
what the readings mean and how they were
obtained.
Place the prism provided on a tracing
1a. paper and draw its outline ABC.
Remove the prism. Measure and
record the value of the angle A. Draw
a normal to the line AB and N. Also
draw another line TN to the normal
such that i = 45˚ Erect two pins at P1
and P2. Replace the prism and fix two
other points at P3 and P4 such that
In the diagram above, a thread AC, fixed at the pins appear to be in a straight
pulley A passes over a pulley at C on a force line with the images of the pins of P1
board and carries an unknown weight M. and P2 when viewed from the side AC
Retain this weight m throughout the and of the prism. Remove the prism.
experiment. Draw a line along the direction Join P3 and P4 producing it to meet
of AC on the paper held behind the thread. line TN at Z. Draw the normal XY.
Locate the midpoint B of AC and mark its Measure and record the angle of
position on this line. Draw BD at right angles emergency e and the angle of
to AC. By means of a loop of thread, deviation d. Evaluate (d – e).
suspend a mass M = 70g from AC and
adjust the position of the loop so that the Repeat the experiment with i = 50˚,
line of actions of the weight of M lies along 55˚ 60˚ and 65˚. In each case,
BD. Ensure that M and m hand off the force measure and record the
corresponding values of e, d and (d –
e). Tabulate your readings. Plot a b. (i) Given that E = V + IR in the
graph with (d – e) on the vertical axis experiment
and i on the horizontal axis starting above, use your graph to determine
both axis from the origin. Find the the value of R (ii) (a). What did the
slope of the graph and the intercepts voltmeter measure when the jockey
L1 and L2 on the vertical and made contact with potentiometer
horizontal axis respectively. Evaluate wire at J?
Lo = (11 + 12)/2
(ii) What would be wrong with the
State two precautions you took to connections if the ammeter deflected
ensure accurate results. Attach your in the opposite direction during the
traces to your answer script. experiment?

2. (i) What is the physical significance of (iii) State two advantages of using a
the potentiometer over a voltmeter for
intercept of the graph on the measuring potential difference.
horizontal axis, if d = I + e – A where
the symbols are as shown on the
diagram above. 1989
SSCE PHYSICS
(ii) If a ray of light is incident normally PAPER 2 OBJECTIVE
on face AB of the prism, draw a ray
diagram to show the path of the ray 1. Which of the following is not a vector
as it passes through the prism. quantity?
(Take refractive index of glass – 1.5) A. Momentum
B. Force
C. Velocity
D. Temperature
(iii) Distinguish between deviation and E. Displacement
refraction.
2. Which of the following is not a consequence
3. (a) of a force field?
A. weight
B. surface tension
C. Gravitational pull
D. Magnetic force
E. Electric force
Connect up the circuit as shown above. PQ
is the potentiometer wire. Record the 3. A boy pulls a nail from the wall with a string
ammeter reading Io and the voltmeter tied to the nail. The string is inclined at an
reading VO when the jockey is not in angle of 60˚ to the wall. If the tension in the
contact with the potentiometer wire PQ. string is 4N what is the effective force used
Then place the jockey to make contact with in pulling the nail?
PQ at J such that PJ – 40cm. Read and record A. 2N
I and V. Repeat the experiment with PJ – 50, B. 2√3N
60, yo and 80cm. In each case, read and C. 4N
record the values of I and V. Tabulate your D. 4√3N
readings. E. 8N

Plot a graph with 1 on the vertical axis and


V on the horizontal axis starting both axis
from the origin. Determining the slope of
the graph and the value of V when 1 = 0.
State two precautions you took to ensure 4. The diagram above represents the
accurate results. trajectory of a projectile with an initial
velocity U. If g is the acceleration of free D. weight only
fall, the time taken to reach the maximum E. upthrust only
height is
A. 2Ug 9. Which of the following statements is correct
sin θ about the equilibrium of the object
represented in the diagrams below?
B. U sin θ
2g

C. u con θ
g
A. X and Y are in stable equilibrium
D. U__ B. Y and Z are in neutral equilibrium
g sin θ C. X is in unstable equilibrium
D. Z is in stable equilibrium
E. U sin θ E. Y is in neutral equilibrium
g
10. A stone tied to a string is made to receive in
a horizontal circle of radius 4m with an
5. Using the diagram below, calculate the angular speed of 2 radians per second. With
moment of the force of 10 N about point P what tangential velocity will the stone move
off the circle if the string cuts?
A. 60 Nm A. 16.0ms-1
B. 60 3Nm B. 8.0ms-1
C. 6.0ms-1
C. 120Nm
D. 2.0ms-1
D. 120 3Nm E. 0.5ms-1
E. 240Nm
11. The motion of the prongs of a sounding
6. Which of the following is used to determine turning fork is
the relative density of the acid in a car A. random
battery? B. translational
A. Hypsometer C. rotational
B. Hygrometer D. vibratory
C. Manometer E. vibratory and rotational
D. Hydrometer
E. Spectrometer 12. Which of the following is a derived unit?
A. Metre
7. A block of material of volume 20cm3 and B. Coulomb
density 2.5gcm3 is suspended from a spring C. Kilogram
balance with half the volume of the block D. Second
immersed in water. What is the reading of E. Ampere
the spring balance? (Density of water =
1.0g cm-3) 13. An engine raises 100kg of water through a
A. 8g height of 60m in 20s. What is the power of
B. 25g the engine? {Take g = 10ms-2}
C. 30g A. 120.000W
D. 40g B. 3000W
E. 50g C. 333W
D. 300W
8. A piece of cork which is floating on water is E. 30W
acted upon by the forces of
A. weight and viscosity 14. A thermometer has its stem marked in
B. weight and upthrust millimeter instead of degree Celsius. The
C. upthrust and viscosity lower fixed point is 30mm and the upper
fixed point is 180mm. Calculate the C. 117200J
temperature in degree Celsius when the D. 113000J
thermometer reads 45mm. E. 4200J
A. 67.5˚C 20. A block of ice at its melting point is left on a
B. 30.0˚C table in the atmosphere and it is observed
C. 25.0˚C to melt gradually. Which of the following
D. 10. ˚C statements is true about the melting
E. 10.0˚C process?
A. The reaction force of the table on the ice
15. Which of the following surface will radiate breaks the ice water molecules
heat energy best? B. heat extracted from the ice block makes the
A. Red ice block cold
B. White C. The ice block absorbs heat from the
C. Black atmosphere and its temperature rises
D. Yellow D. The atmosphere and its temperature
E. Blue remained constant as it melts

16. Which of the following statements is not 21. All the heat generated by a current of 2 A
correct? passing through a 6Ω resistor for 25 s is
A. Evaporation takes place only at the surface used to evaporate 5g of a liquid at its
of a liquid? boiling point. What is the specific latent heat
B. Boiling takes place through out the volume of the liquid?
of a liquid A. 60 Jg-1
C. Evaporation takes place at all temperatures B. 120Jg-1
D. Boiling takes place at a particular C. 300Jg-1
temperature for a liquid at a given external D. 360Jg-1
pressure E. 1500Jg-1
E. The boiling point of a liquid is not affected
by impurities 22. A positively charged sphere is suspended
inside a hollow can placed on an insulating
17. Water in an open container boils at a lower base. If the can is momentarily earthed and
temperature when heated at the top of a the charged sphere is then withdrawn,
mountain than at sea-level because at the which of the following diagrams best
top of a mountain the represents the charge distribution on the
A. relative humidity is higher than that at sea- can after this process?
level
B. rays of the sun add more heat to the water
C. temperature is lower than that at sea-level
D. pressure is lower than that at sea-level
E. Pressure is more than that at sea-level A.

18. Which of the following substances is not a


good conductor of electricity?
A. Human body
B. Glass B.
C. Aluminium
D. Earth
E. Copper

19. Calculate the heat energy required to C.


vaporize 50g of water initially at 80˚C if the
specific heat capacity of water is 4.2Jg-1K-1,
(Specific latent heat of vaporization of water
is 2260Jg-1)
A. 533000J D.
B. 230200J
What will be the characteristics of the image
E. of the object OB shown above after
refraction through the lens?
A. Magnified, virtual and inverted
23. Which of the following instruments can be B. Real, inverted and magnified
used to compare the relative magnitudes of C. Diminished, virtual and inverted
charge on two given bodies? D. Erect, real and diminished
A. The electrophorus E. Diminished, virtual and erect
B. Ebonite rod 29. A beam of light consisting of both green G
C. Proof planes and Violet V colours is incident on a prism.
D. Gold leaf electroscope Which of the following diagrams correctly
E. Capacitor illustrates the path of the beam?

24. A short chain is usually attached to the back


of a petrol tanker trailing behind it to ensure A.
that the
A. petrol tanker is balanced on the road
B. heat generated by friction in the engine can
be conducted to the floor
C. charges generated by friction in the tanker B.
is conducted to the earth
D. tanker moves slowly as the chain touches
the road surface
E. chain produces sound for the resonance of C.
the tanker’s engine

25. A ray of light is incident on a plane mirror at


an angle of 35˚. What is the angle made by
the reflected ray with the surface of the D.
mirror?
A. 125˚
B. 70˚
C. 65˚
D. 55˚
E. 35˚
E.
26. The image in a pin-hole camera is always
A. diminished
B. enlarged 30. The distance between a node and an
C. upright antinodes for a transverse wave is equal to
D. inverted A. the wavelength
E. blurred B. thrice the wavelength
C. One-half of the wavelengths
27. The refractive index for a given transparent D. one-quarter of the wave length
medium is 1.4. Which of the following is the
minimum angle for total internal reflection 31. Which of the following is a stringed
to take place in the medium? instrument?
A. 30 A. Flute
B. 36˚ B. Trumpet
C. 44˚ C. Piano
D. 46˚ D. Drum
E. 54˚ E. Saxophone

28.
32. A sound pulse sent vertically downwards 37. What is the gravitational potential due to a
into the earth is reflected from two different molecule of mass m at a distance from it?
layers of the earth such that echoes are (G – gravitational constant)
heard after 1.2s and 1.4s. Assuming the A. Gm2_
speed of the pulse is 2000ms-1, calculate r2
the distance between the layers.
A. 200m B. Gm
B. 400m r
C. 2400m
D. 2600m C. Gm2
E. 2800m r

33. In a wave, the maximum displacement of D. G2m2


particles from their equilibrium positions is r
called
A. frequency E. m2_
B. amplitude Gr2
C. period
D. wavelength 38. Calculate the escape velocity for a rocket
E. wave velocity fired from the earth’s surface at a point
where the acceleration due to gravity is
34. A string is stretched tightly between two 10ms-2 and the radius of the earth is 6 x
point 50cm apart. It is plucked at its centre 106m.
and the velocity of the wave produced is A. 7.8 x 103ms-1
300ms-1. Calculate the number of vibrations B. 1.1 x 104ms-1
made by the string in one second. C. 3.5 x 107ms-1
A. 3 D. 6.0 x 104ms-1
B. 6 E. 1.2 x 108ms-1
C. 150
D. 300 39. The electric force between two point charge
E. 600 each of magnitude q at a distance of r apart
in air of permittivity Eo is
35. Which of the following is not a property of A. q2__
longitudinal waves? 4 π Eor
A. Compression
B. Reflection B. rEo_
C. Refraction q2
D. Polarization
E. Diffraction C. 4π q
E0
36.
D. qr2_
E0

The diagrams above show lines of force in E. q2___


electric fields. In which of the diagram 4 π E0r2
would a positive test charge experience less
force as it moves from X to Y? 40. What is the total capacitance in the circuit
A. III only represented by the diagram above?
B. II only A. C1 = C + C1
C. I only B. 1 + 1 + 1
D. I and II only C1 C2 C3
E. I and III only
C. C1C2C3____
C1 + C2 + C3
D. C2__ 45. In a metre bridge experiment, there is a
C3 + C1 zero deflection of the galvanometer when I1
– 40cm as shown in the diagram below.
E. C 1C 2C 3 Calculate the value of the resistance R.

41. Calculate the energy stored in a 10 µ F A. 62/3Ω


capacitor if the potential difference between B. 15Ω
the plates is 20V. C. 20Ω
1.0 x 10-9J D. 25Ω
A. 2.0 x 10-4J E. 24Ω
B. 2.0 x 10-4J
C. 2.0 x 10-3J 46. Which of the following is not true of a
D. 1.0 x 103J discharge tube?
E. 2.0 x 103J A. The pressure of gas must be very low
B. The glass tube must have two electrodes at
42. In the diagram below, the galvanometer the ends
indicates a null-deflection. What is the C. The gas in the tube must be carbon dioxide
potential difference between X and Y? D. Air is gradually pumped out of the tube
E. The coating on the walls of the tube at a
A. O volt certain stage glows
B. 1.5 volts
C. 2.0 volts 47. Which of the following diagrams gives the
D. 3.0 volts correct magnetic field pattern around a
E. 4.0 volts current-carrying straight conductor?

43. The circuit represented beside is used for A.


charging an accumulator from a.d.c. supply
of e.mf. E1 and internal resistance r1. If the
e.m.f. of the accumulator is E2 with an B.
internal resistance of r2, then the equation
for the charging current is
A. I = E1 + E2 C.
r1 + r2

B. I = E1 – E2 D.
r1 + r2
E.
C. E1 – E1
r1 + r2
48. Which of the following is/are correct about a
D. I = E1 – E2 plane rectangular current carrying coil
r1 – r2 placed in a uniform magnetic field?
I. The forces in the two parallel sides produce
E. I = E1 + E2 a resultant translational motion
r1 – r2 II. The torgue is maximum when the plane of
the coil is parallel to the direction of the
44. A portable generator is connected to six 100 field
W lamps and a 600 W amplifying system. III. The forces on the two parallel sides are
How much energy is consumed if the equal and opposite
generator runs for 6 hours? A. I only
A. 0.2k Wh B. II only
B. 1.2k Wh C. III only
C. 3.0k Wh D. I and III only
D. 4.2k Wh E. II and III only
E. 7.2k Wh
49. Which of the following materials should be II. When the electrons jump from one orbit to
used fro the core in order to produce the another, they emit discrete energy packets
strongest magnetic effect at J in the circuit or quanta
illustrated in the diagram below? III. The energy emitted by the electrons in their
orbits contain all possible wavelengths
A. Copper A. I only
B. Iron B. I and II only
C. Brass C. II and III only
D. Plastic D. I and III only
E. Steel E. I, II and III

50. When a metal is heated to a high 55. An elastic string of length / is stretched
temperature, electrons are emitted from its through a length e by a force F. The area of
surface. This process is known as cross-section of the string is A and its
A. photo electric emission Young’s modulus is E. Which of the following
B. artificial radioactivity expressions is correct?
C. field emission A. F = EAe2
D. secondary emission 1
E. thermionic emission
B. F = EAL
51. In an a.c. circuit the peak value of the c
potential difference is 180V. What is the
instantaneous p.d. when it has reached 1/8th C. F = EAe
of a cycle? L
A. 45V
B. 90V D. F = EAe
C. 90 2V 12

E. F = EA
D. 180V
el
E. 180 2V

52. Which of the following will not lower the


surface tension of water?
A. Detergent
B. Methylated spirit
C. Soap solution
D. Grease 56. In the diagram above, V is viscosity and U is
E. Camphor up-thrust. Which of the following equations
holds when the falling body P has attained a
53. Which of the following gives rise to the line terminal velocity?
spectra observed in atoms? A. V + ng = U
A. kinetic energy of a moving atom B. V + U = mg = 0
B. Potential energy of an electron inside an C. V = mg_
atom U
C. Change of an electron from a higher to a D. V – U = mg
lower energy level in the atom E. V + mg + U = 0
D. Disturbed proton in the nucleus
E. Excitation of an electron in the atom 57. An electron jumps from one energy level to
another in an atom radiating 4.5 x 10-19
54. Which of the following is/are essential joules. If Planck’s constant is 6.6 x 10-34Js
properties of the Bohr’s model of the atom? What is the wavelength of the radiation?
I. The electrons in the orbits have discrete (Take velocity of light = 3 x 108ms-1)
values for their angular momentum A. 4.4 x 10-4m
B. 4.4 x 10-7m
C. 4.0 x 10-7m
D. 1.5 x 10-7m linear expansivity of steel is
E. 1.5 x 10-19m 1.0 x 10-5K-1 calculate the
safety gap that must be left
58. If a nucleus 3H decays, a nucleus 3 He is between successive bars if
found the highest temperature
1 2 expected is 41˚C
Accompanied with the emission of a
A. neutron (c) State three advantages and two
B. proton disadvantages of thermal expansion
C. beta particle of solids
D. gamma particle
E. alpha particle 2. (a) What is a wave motion?

59. Which of the following statements is a The equation, y = A sin λ 2π


correct consequence of the uncertainty (Vt – X) represents a wave train in
principle? which y is the vertical displacement
A. The uncertainty in our knowledge of energy of a particle at distance X from the
and the duration taken to measure it are origin in the medium through which
each less than Planck’s constant the wave is traveling. Explain, with
B. The complete knowledge of the position of a the aid of a diagram, what A and λ
particle implies the complete ignorance of represent.
its energy
C. It is possible to measure exactly both the (b) (i) Describe an experiment to
position and momentum of a particle at the determine the frequency of a
same time note emitted by a source of
D. Both momentum and energy of a particle sound.
can be known with absolute certainty (ii) A pipe closed at one end is
E. A particle’s kinetic energy cannot be 1m long. The air in the pipe is
measured accurately at anytime set into vibration and
fundamental note is
60. The phenomenon of radioactivity was first produced. If the velocity of
discovered by sound in air is 340ms-1,
A. Marie Curie calculate the frequency of the
B. Sir J. J. Thompson note.
C. Henri Beequerel
D. Niels Bohr (c) State two differences between a
E. Enrico Fermi sound wave and a radio wave.

3. (a) State the laws of electromagnetic


induction.
1989
SECTION B THEORY (b) (i) Describe a simple experiment
to
1. (a) What is meant by the statement: show how an induced e.m.f.
“The can be produced.
linear expansivity of a solid is 1.0 x
10-5 (ii) State two factors on which
K-1? the
magnitude of the induced e.m.f. depends
(b) (i) Describe an experiment to
determine the linear (c) Explain what is meant by the r.m.s.
expansivity of a steel rod value of an alternating current.
(ii) Steel bars, each of length 3m
at (d) (i) If the alternating current is
29˚C are to be used for represented by 1, 10, w and
constructing a rail line. If the wt represent.
point on the rule where mass m is attached
(ii) Calculate the instantaneous to the rule. With length L = 90cm,
value determine the time for the loaded metre
of such a current, if in a rule to perform 20 vertical oscillations.
circuit it has r.m.s. value Repeat once and find average time for 20
15.0A when its phase angle is oscillations.
30˚
Repeat the experiment with length L =
4. (ai) Explain the terms: photoelectric 85cm, 75cm and 70cm. In each case,
emission determine the average time t taken by the
and threshold frequency metre rule to perform 20 vertical
oscillations. Calculate the period T of
(ii) Einstein’s photoelectric equation can oscillation in each case and tabulate your
be readings.
written as E = hf = hfo.
What does such each of the symbols Plot a graph with lot 100T along the vertical
used in the equation above axis and log L along the horizontal axis.
represent? Determine the slope of the graph.

(b) Calculate the frequency of the proton b. (i) Explain what is meant by a couple
whose energy is required to eject a and
surface electron with a kinetic define the moment of a couple.
energy of 1.97 x 10-16eV if the work
functions of the metal is 1.33 x 10- (ii) If R is the reaction at the bench. M
16
eV. (leV = 1.6 x 10-19J; h = 6.60 x the
10-34 JS). mass of the metre rule and m the
mass of the attached load, draw a
(c) In a photoelectric cell, no electrons diagram to indicate the forces acting
are emitted until the threshold on the rule when in equilibrium.
frequency of light is reached. Explain Write down a relationship between R,
what happens to the energy of the M and m.
light before emission of electrons
begins. State one factor that may (iii) Explain the term centre of gravity of
affect the numbers of emitted a body and state how it is related to
electrons. the stability of an object.

(d) Explain what is meant by the duality 2. (a)


of matter, illustrating your answer
with observation phenomena.

Contact the voltmeter across the given 2Ω


standard resistor and join the combination
1989 in series with the accumulator, the
PAPER 1 PRACTICAL resistance box S and a key K as shown
above. Adjust the resistance box S to 15Ω
1(a) and record the reading of the voltmeter.
Repeat with values of S = 12Ω, 10Ω, 8V, 6
and 4Ω. In each case, record the
corresponding reading V of the voltmeter.
Find the values of I/v in each case. Tabulate
your readings.
Clamp the loaded metre rule firmly to the Plot a graph of 1/v along the vertical axis
edge of the bench as a horizontal cantilever and S along the horizontal axis. Determine
as shown above capable of performing the slope m of the graph. Also determine
vertical oscillations. L is the distance the intercept on the vertical axis. Evaluate
between the edge of the bench and the the ratio e/m. State two necessary
precautions taken when performing this thickness 1.6cm and of refractive
experiment. index 1.6 is viewed along the normal
to the opposite surface. By how
b. (i) State two conditions under which a much does the point object appear
piece to be displaced?
of wire in a closed electrical circuit (iii) Draw a diagram to illustrate what is
may not obey Ohm’s law. meant by a lateral displacement of a
ray of light which passes through a
(ii) Explain why the e.m.f of a cell is rectangular glass block.
usually greater than the p.d. at its
terminals when the cell is used to
supply current to an external circuit. 1990
SSCE PHYSICS
(iii) Four identical cells each e.m.f. E and PAPER 2 OBJECTIVE
internal resistance r are connected in
series with a resistance R. Write 1. Which of the following instruments is
down the formula for the current in suitable for making the accurate
the circuit. measurement of the internal diameter of a
test tube?
A. A metre rule
B. A pair of calipers
C. A micrometer screw gauge
D. A tape rule
E. A set square
3. (a) Trace the outline EFGH of the given
rectangular glass block. Remove the 2. A particle of mass 2.5 x 10-6kg revolving
block and draw a normal KL cutting around the earth has a radial acceleration of
the faces at S and Q such that ES is 4 x 107ms-3. What is the centripetal force
about one-quarter of EF. Draw a line of the particle?
making an angle i = 30˚ and KS as A. 6.25 x 10-14N
shown. Draw four other lines for i = B. 1/6- x 10-13N
35˚, 45˚, 55˚, and 65˚. C. 5.00 x 10N
D. 1.00 x 102N
Replace the glass block carefully on its E. 2.00 x 103N
outline. Fix two pins at P1 and P2 on the
incident line making the angle i = 30˚ with 3. An oscillating pendulum has a velocity of
KS. By looking through the side HG, fix two 2ms-1 at the equilibrium position O and
other pins at P3 and P4 such that the pins at velocity at same point R. Using the diagram
P3 and P4 appear to be in line with the below, calculate the height h of P above O.
images of pins at P1 and P2. Mark the (Take g = 10ms2)
positions of P3 and P4. Remove the glass
block. Draw a line to pass through P3 and P4 A. 5.0m
and produce it to meet HG at T1 and KL at X1 B. 2.0m
respectively. Measure and record ST, and X1 C. 0.4m
T1. Repeat the experiment for I = 35˚, 45˚, D. 0.2m
55˚ and 65˚. In each case, measure and E. 0.1m
record ST and XT.
4. Which of the following are contact forces?
Plot a graph with XT on the vertical axis and I. Force of tension
ST on the horizontal axis. Calculate the II. Force of friction
slope of the graph. State two precautions III. Magnetic force
you took when performing the experiment. IV. Force of reaction
A. I, II and III only
b. (i) State Snell’s law of refraction. B. I, II and IV only
(ii) A point object placed in contact with C. I, III and IV only
one surface of a glass block of D. II, III and IV only
E. I and IV only
10. An object R leaves a platform XY with a
5. A plane inclined at an angle of 30˚ to the horizontal velocity of 7ms-1 and lands at O. If
horizontal has an efficiency of 50%. The it takes the same object 0.3s to fall freely
force parallel to the plan required to push a from Y to P, calculate the distance PQ. (Take
load of 120N uniformly up the plan is g = 10ms-2)
A. 40√3N A. 7.30m
B. 60N B. 2.55m
C. 120N C. 2.10m
D. 200N D. 1.65m
E. 240N E. 0.45m

6. The diagram above represents a portion of a 11. A simple pendulum makes 50oscillations in
micrometer screw guage. What is its one minute. What is the period of
reading? oscillation?
A. 3.7mm A. 0.02s
B. 3.67mm B. 0.20s
C. 3.50mm C. 0.83s
D. 3.33mm D. 1.20s
E. 3.17mm E. 50.00s

7. A girl whose mass is 55kg stands on a 12. The body P shown in the diagram above is
spring weighing machine inside a life. When in equilibrium. If the mass of the body is
the lift starts to ascent, its acceleration is 10kg, calculate the tension T in the string
2mm2. What will be the reading on the (Take g = 10ms-2)
machine? (Take g = 10ms-2) A. 20/√3N
A. 66kg 4N B. 20N
B. 55kg C. 50N
C. 44kg 6Am D. 200√3N
D. 22kg E. 200N
E. 11kg
13. An object weighs 10.0N in air and 7.0 in
water. What is its weight when immersed in
a liquid of relative density 1.5?
A. 4.50N
4N B. 4.67N
8. Two forces each of 4 N act on the opposite C. 5.50N
sides of a rectangular plate as shown in the D. 6.67N
diagram above. Calculate the magnitude of E. 8.50N
the couple acting on the plate.
A. 6.4Nm 14. The absolute zero temperature is defined as
B. 3.2Nm the temperature at which
C. 1.6Nm A. thermal motion ceases
D. 0.8Nm B. the temperature of a gas is 273˚C
E. 0.0Nm C. ice melts
D. the volume of a real gas is maximum
9. A force F is applied to a body P as shown in E. pressure of a real gas is maximum
the diagram below. If the body P moves
through a distance r, which of the following 15. A cube made of a metal of linear
represents the work done? expansivity α is warned through a
A. Fx temperature of t. If the initial volume of the
B. F cos θ cube is V, what is the increase in volume of
the cube?
x 1 α
A. /3 Vt
C. Fx tan θ B. ½ α Vt
D. Fx sin θ C. α Vt
E. Fx cos θ D. 2 α Vt
E. 3 α Vt
16. Dry hydrogen is trapped by a pellet of 21. Which of the following is not a suitable
mercury in a uniform capillary tube closed method for reducing loss of heat from a
at one end. If the length of the column of piece of hot iron?
hydrogen at 270˚C is 1.0m, at what A. Wrapping it in cotton wool
temperature will the length be 1.20m? B. painting it black
A. 22.5˚C C. placing it in a vacuum
B. 32.4˚C D. placing it on the rubber supports
C. 77.0˚C E. keeping it in a closed wooden box
D. 87.0˚C 22. When the direction of vibration of the
E. 360˚C particles of a medium is perpendicular to
the direction of travel of a wave, the wave
17. Two solids P and Q of equal mass are transmitted is known as
heated to the same temperature and A. sound wave
dropped simultaneously into two identical B. transverse wave
containers X and Y containing equal C. longitudinal wave
volumes of water at room temperature. The D. stationary wave
temperature of X will rise higher than that of E. mechanical wave
Y only if the
A. volume of P is greater than that of Q 23. A bat emits a sound wave at a speed of
B. specific heat capacity of P is higher than 1650 ms-1 and received the echoes 0.15s
that of Q later. Calculate the distance of the bat from
C. density of P is greater than that of Q the reflector.
D. density of P is lower than that of Q A. 8.75m
E. specific heat capacitor of P is lower than B. 16.50m
that of Q C. 87.75m
D. 123.75m
18. If the temperature of a small quantity of E. 330.00m
water in a closed container gradually
increased from 0˚ to 4˚C, then the density 24. A tuning fork sounds louder when its stem is
of the water within this range, pressed against a table top than when held
A. increases for a while and then decreases in air because
B. decreases for a while and then increases A. a larger mass of air is set vibrating by the
C. increases gradually table top
D. decreases gradually B. the whole table vibrates in resonance
E. remains the same C. the whole table has acquired a larger
frequency
19. How much heat is given out when a piece of D. the fork and table has acquired a larger
iron of mass 50g and specific heat capacitor frequency
460 Jkg-1 K-1 cools from 85˚C to 2˚C? E. the amplitude of the vibrating fork is
A. 1.38 x 106J increased
B. 2.53 x 104J
C. 7.66 x 103J 25. Which of the following is/are characteristics
D. 1.38 x 103J of sound?
E. 1.27 x 103J I. pitch
II. Loudness
20. Heat transfer by convection in a liquid is III. Quality
due to the IV. Noise
A. translatory motion of the molecules of the A. I only
liquid B. II only
B. increased vibration of the molecules of the C. I and II only
liquid about their mean positions D. I, II and III only
C. expansion of the liquid as it is heated E. I, II, III and IV
D. latent heat of vaporization of the liquid
E. ability of the liquid to evaporate at all 26. The diagram below represents part of a
temperatures wave motion in air. If the wave travels with
a speed of 300ms-1, calculate the frequency 31. A parallel beam of light is to be obtained
of the wave from the headlamp of a car. At hitch of the
following positions should the source of light
A. 360Hz be placed from the pole of its spherical
B. 300Hz mirror?
C. 250Hz A. At the focal point
D. 150Hz B. At the centre of curvature
E. 100Hz C. Beyond the centre of curvature
D. Between the focal point and the pole
27. Which of the following is/are necessary for E. Between the focal point and the centre of
the production of interference with two curvature
wave trains?
I. The sources must be close to each other 32. The diagram beside shows an incident ray
II. The waves must have the same frequency AO inclined at an angle of 50˚ to the
III. The waves must have the same amplitude interface CB. The refracted ray CB is found
IV. The waves must travel with the speed of to like along the surface. What is the
light refractive index of the medium X with
A. I, II, III and IV respect to air?
B. I, II and III only A. Sin 50˚
C. I and II only Sin 40˚
D. II, III and IV only
E. I, II and IV only B. Sin 40˚
Sin 50˚
28. A stone is dropped into the middle of a pool
of water. Which of the following statements C. Sin 90˚
is/are correct? Sin 50˚
I. Spherical waves are set up in the water
II. The water moves outwards to the sides of D. Sin 40˚
the pool Sin 90˚
III. Energy is transmitted outwards from the
centre of disturbance E. Sin 90˚
A. I only Sin 40˚
B. II only
C. III only 33. The eye defect illustrated by the diagram
D. I and III only below is
E. II and III only A. astigmatism
B. long sightedness
29. A concave mirror of radius of curvature C. loss of accommodation
20cm has a pin placed at 15cm from its D. short sightedness
pole. What will be the magnification of the E. colour blindness
image formed?
A. 4.00 34. A simple microscope forms an image twice
B. 2.00 the size of the object. If the focal length of
C. 1.33 the lens of the microscope is 20cm, how far
D. 1.50 is the object from the lens?
E. 0.25 A. 10m
B. 20m
30. Where will the image of the object OB C. 30m
shown in the diagram below be located after D. 40m
reflection from the mirror? E. 60m
A. At F
B. beyond C 35. Which of the following pairs of light rays
C. between O and P shows the widest separation in the
D. between P and F spectrum of white light?
E. between F and C A. Yellow and red
B. Orange and green
C. Violet and blue D. 20Ω
D. Blue and green E. 10Ω
E. Red and indigo
41. Which of the following field patterns
36. Given that the gravitational constant is 7 x between two unlike charges is correct?
10-41 Nm2 kg2, what is the force of
attraction between 10˚kg mass of lead A.
hanging one metre away from a 103kg
mass of iron?
A. 7 x 10-20N B.
B. 7 x 10-8N
C. 7 x 10-2N
D. 7 x 103N C.
E. 7 x 108N
D.
37. Calculate the current in the 3Ω resistor
shown in the diagram below.
A. 3.0A
B. 4.0A E.
C. 4.3A
D. 12.0A 42. A charge of 1.0 x 10-5 coulombs experiences
E. 39.0A a force of 40N at a certain point in space.
What is the electric field intensity?
38. What is the electric potential energy A. 8.00 x 106NC-1
between two protons each of charge q and B. 4.00 x10-46NC-1
at a distance of r apart? (Permittivity of C. 4.00 x 10-4NC-1
free space = Σ0) D. 2.00 x 10-4NC-1
A. 4 π Σ0q2r E. 2.50 x 10-7NC-1

B. qr_ 43. If the frequency of the a.c circuit illustrated


4 π Σ0 is 500/ π Hz, what would be the reactance in
the circuit?
C. q_ A. 0.0009Ω
4 π Σ01 π
B. 400Ω
D. q 2_ C. 1030Ω
4 π Σ0r D. 1400Ω
E. 2500Ω
E. q 2_
4 π Σ0r2 44. Calculate the resistance of the filament of a
lamp rated 240V, 40W
39. A work of 30 joules is done in transferring 5 A. 240Ω
millicuoulombs of charge from a point B to a B. 360Ω
point A in an electric field. The potential C. 720Ω
difference between B and A is D. 1440Ω
A. 1.7 x 10-4V E. 2880Ω
B. 3.4 x 10-4V
C. 1.5 x 10-1V 45. Birds can perch safely on overhead electric
D. 6.0 x 103V power cables without being electrocuted
E. 1.2 x 104V because
A. birds can withstand high p.d. of the electric
40. What is the value of R when G shows no power cables
deflection in the circuit illustrated beside? B. any accumulated charge on the birds is
A. 80Ω quickly dissipated
B. 75Ω C. the feet of the birds are well insulated
C. 45Ω
D. the current is a.c. and has no particular B. Molecules
direction C. Electrons
E. there is no p.d. between the birds and the D. Neutrons
cables E. Ions

46. A cell of e.m.f. 1.5V and internal resistance 51. When the pressure of the gas in a discharge
of tube in gradually lowered
2.5Ω is connected in series with an ammeter I. There is a violet glow in the tube
of resistance 0.5Ω and a resistor of II. The glow in the tube splits into bands which
resistance 7.0Ω. Calculate the current in the get further apart
circuit. III. The glow and the bands eventually
A. 0.15A disappear and there is a green fluorescence
B. 0.20A in the tube
C. 0.60A IV. The tube breaks into pieces
D. 3.00A Which of the following are correct?
E. 6.67A A. I and II only
47. A rectangular coil of wire can rotate in a B. I and III only
magnetic field. The ends of the coil are C. I and IV only
soldered to the two halves of a split ring. D. I, II and III only
Two carbon brushes are made to press E. I, II, III and IV only
lightly against the split ring and when these
are connected in circuit with a battery and 52. Which of the following statements about the
rheostat, the coil rotates. This is a atom is/are correct?
description of I. Energy is needed to remove electrons from
A. a suspended-coil galvanometer an atom
B. a moving-cell ammeter II. Particles in the nucleus of an atom are
C. a.d.c generator bound together by strong forces
D. an electric motor III. A large amount of energy is required to
E. an induction coil separate the particles in the nucleus of an
atom
48. How long will it take to heat 3kg of water IV. When the nucleus of a uranium atom is
from 28˚C to 88˚C in an electric taking a split, the energy released accounts for the
current of 6A from e.m.f. source of 220V? difference in the masses of the products
[Take the specific heat capacity of water = and the parent nucleus.
4180J-1K-1] A. I only
A. 96s B. I and II only
B. 1.20s C. I and III only
C. 570s D. II and IV only
D. 600s E. I, II, III and IV
E. 1200s
53. The nucleon number and the proton number
49. Which of the following reduce(s) the effect of a neutral atom of an element are 238 and
of the back e.m.f generated in the primary 92 respectively. What is the number of
coil of an induction coil? neutrons in the atom?
I. The capacitor in the circuit A. 330
II. The make and break contact in the circuit B. 165
III. The ratio of turns in the secondary C. 146
A. I only D. 119
B. II only E. 73
C. I and II only 54. Water does not drip through an open
D. II and III only umbrella of silk material unless the inside of
E. I, II and III the umbrella is touched. Which of the
following phenomenon is responsible for
50. Which of the following particles conduct this?
electricity through salty water? A. Surface tension
A. Atom B. Hydrostatic up thrust
C. Viscosity
D. Diffusion
E. Osmosis 1990
SECTION B THEORY
55. A radioactive substance has a half-life of 20
hours. What fraction of the original 1. (a) Using a suitable diagram, explain
radioactive nuclei will remain after 80 how the
hours? Following can be obtained from a
A. 1/32 velocity-time graph.
B. 1/16
C. `/8 (i) Acceleration; (ii) retardation
D. ¼ (iii) Total distance covered
E. ½
(b) Show that the displacements of a
56. A metal is illuminated with a radiation of body moving with uniform
energy 6.88cV. If the kinetic energy of the acceleration is given by s = ut + ½
emitted electrons is 1.50cV, calculate the ar2 where u is the velocity of the
work function of the metal. body at time t = 0.
A. 0.22cV
B. 4.59cV (c) A particle moving in a straight line
C. 3.38cV with uniform deceleration has a
D. 8.38cV velocity of 40ms-1 at a point P,
E. 10.32cV 20ms-1 at a point Q and comes to
rest at a point R where QR = 50m.
57. Which of the following is not a crystal? Calculate the:
A. Sodium chloride (i) distance PQ;
B. Lead (ii) time taken to cover PQ
C. Copper (II) tetraoxosulphate (VI) (iii) time taken to cover PR
D. Glass
E. Germanium 2. (a) Define the boiling point of a liquid
(b) Describe an experiment to determine
58. Viscosity in a liquid does not depend on the the boiling point of small quantity of
A. Nature of the liquid a liquid
B. relative velocity between the liquid layers (c) A piece of copper of mass 300g at a
C. area of the surface in contact temperature of 950˚C is quickly
D. temperature of the liquid transferred to a vessel of neglible
E. normal reaction between the liquid layers thermal capacity containing 250g of
water at 25˚C. If the final steady
59. What is the difference between a crystalline temperature of the mixture is 100˚C,
and an amorphous solid? A crystalline solid calculate the mass of the water that
A. has cubic structure while amorphous solid will boil away.
has hexagonal structure
B. has regularly-repeating pattern while an Specific heat capacity of copper =
amorphous solid has not 4.0x102Jkg-1K-1
C. is white while amorphous solid is yellow Specific heat capacity of water =
D. is always hard while amorphous solid is 4.2x103Jkg-1K-1
always soft Specific latent of vaporization of stream =
E. melts on heating while an amorphous solid 2.26x106Jk-1
sublimates on heating
60. Which of the following is not an evidence of (d) State four other effects of heat on a
the particle nature of matter? substance other than expansion.
A. Diffusion
B. Bronwnian motion 3. (a) Define the capacitance of a capacitor
C. Diffraction (b) State three factors on which the
D. Crystal structure capacitance of a parallel-plate
E. Photoelectricity depends.
(c) Derive a formula for the energy W experiment for h = 40, 60, 80 and 100cm.
stored in a charged capacitor C In each case record the corresponding
carrying a charge Q on either plate. values of t, T and tabulate your results.
(d) Two capacitors of capacitance 4μF
and Plot a graph of h against T2,. Determine the
6μF are connected in series to a slope S of the graph and the intercept I on
100V d.c. supply. Draw the circuit the vertical axis. State two precautions you
diagram and calculate the: took to ensure accurate results.
(i)charge in either plate of each capacitor;
(ii) p.d. across each capacitor. b. (i) In the experiment above, h, T and L
are
(iii) energy of the combined capacitors related by the equation h = L – g T2
4. (a) Explain the terms nuclear fission and 4
nuclear fusion π2
(b) State two advantages of fusion over Using this equation and the value of
fission and explain briefly why, in your slope s, determine the value of
spite of these advantages, fusion is g.
not normally used for the generation
of power. (ii) Why is a spherical bob preferred to
(c) 238
U is a long half-life alpha emitter bob of other shapes for use in this
and experiment?
92
decays to thorium Th which, in
turn, decays by beta emission with a (iii) State two examples of simple
small decay constant to an isotope of harmonic motion other than the
protactinium Pa. The protactinium simple pendulum.
isotope usually decays by beta
emission to an element Y. Explain 2. (a) You are provided with two
the italicized terms and write down illuminated
wires at a fixed distance apart for
the decay scheme of 238U as stated use as object. Measure and record
above. 92
their separation. Do not alter the
(d) State three uses of radioisotopes. distance between the wires
throughout the experiment. Place
the illuminated object at a distance x
1990 – 20cm from the lens to produce a
PAPER I PRACTICALS focused image on the screen.
Measure and record the distance y of
1. the image from the lens. Also,
measure the size of the image which
is the distance between the images
of the wires. Evaluate m a/a0.

Repeat the experiment for x = 25,


30, 35, and 40cm. In each case,
You are provided with a pendulum bob, a determine the corresponding values
string retort stand and clamp. Suspend the of a, y and m. Calculate your
bob from the retort stand such that the bob readings.
just touches the floor. The point of
suspension to the floor L = 1.5m should be Plot a graph of m against y.
kept constant throughout the experiment. Determine the slope S of the graph
Reduce the length of the pendulum with h = and the value of y for which m = 0.
20cm as shown in the diagram above. In State two precautions you took to
this position displace the bob and record the ensure accurate results.
time t for 20 oscillations. Hence, determine
the period T of oscillation. Record your b. (i) If the relation between m and y in
values of h, t, T and T2. Repeat the the
experiment above is given by m = c Greater than the p.d across its
+ y/k, where c and k are constant, terminals when the cell is used to
use your graph to determine the supply current to an external circuit
value of k.

(ii) What will be the nature and position


of
the image formed by the lens above 1991
if the object is situated at a distance SSCE PHYSICS
less than the focal length of the lens? PAPER 2 OBJECTIVES

(iii) An object, place 50cm away from the 1. Which of the following is a fundamental
focus of an emerging lens of focal quantity?
length 15cm, produces a focused A. Speed
image on a screen. Calculate the B. Density
distance between the object and the C. Length
screen. D. Impulse
E. Energy

3. (a) 2. A body moves with a constant speed but


has acceleration. This is possible if it
A. moves in a straight line
Connect the circuit as shown above. B. moves in a circle
Close the key K and record the C. is oscillating
ammeter reading 1o. Remove the D. is in equilibrium
plug of the key. Connect the E. has a varying acceleration
voltmeter across the resistor R = 1Ω
and the rheostat in series with the 3. A ball is projected horizontally from the top
resistor. Connect the voltmeter of a hill with a velocity of 20ms-1, if it
across the resistor reaches the ground 4 seconds later, what is
the height of the hill?
With the key closed adjust the A. 20m
rheostat to make the ammeter B. 40m
reading I = 0.3A. Record the C. 80m
corresponding voltmeter reading V. D. 160m
E. 200m
Repeat the experiment for I = 0.6,
0.9, 1.2, 1.5 and 1.8A. In each case 4. Two forces, whose resultant is 100N are
determine the corresponding value perpendicular to each other. If one of them
of V. Tabulate your readings. makes an angle of 60˚ with the resultant,
Plot a graph of 1 against V. calculate its magnitude (Sin 60˚ = 0.8660,
Determine the slope s of the graph. Cos 60˚ = 0.5000)
Evaluate 1/s. State two precautions A. 200.0N
you took to ensure accurate results B. 172.2N
C. 115.5N
b. (i) Draw the final circuit diagram of the D. 86.6N
experiment above. E. 50.0N

(ii) Use your graph to determine the 5. A body moving with uniform acceleration
value of has two points (5, 15 and 20, 60) on the
V when I – 0.8A. also, state what 1/s velocity-time graph of its motion Calculate
represents. a.
A. 0.25ms-2
(iii) Explain why the e.m.f of a cell is B. 3.00ms-2
usually C. 4.00ms-2
D. 9.00ms-2
E. 16.00ms-2 D. North-east
E. South-west
6. The diagram below shows the position of a
simple pendulum set in motion. At which of 12.
the motion. At which of the position does
the pendulum have maximum kinetic
energy.

The diagram above represents Six’s


maximum thermometer. If the temperature
7. An orange fruit drops to the ground from the of the surrounding falls, which of the
top of a tree 4.5m tall. How long does it take following correctly states how the steel
to reach the ground? (g = 10ms-2) indices in the thermometer would respond
A. 3.0s to the change in temperature?
B. 4.5s A. S1 would move upwards but S2, would not
C. 6.0s move
D. 7.5s B. S1 would not move upwards but S2 would
E. 9.0s move
C. Both S1 and S2 would move upward
8. Power is defined as the D. S1 would move upwards and S2 would move
A. capacity to exert a force downward
B. product of force and time E. S1 would move downwards but S2 would
C. product of force and distance move upwards
D. ability to do work
E. energy expended per unit time 13. A wire, 20m long, is heated from a
temperature of 5˚C to 55˚C. If the change in
9. The product PV where P is pressure and V is length is 0.02m, calculate the linear
volume has the same unit as expansibility of the wire
A. power A. 1.0 x 10-3K-1
B. work B. 2.0 x 10-4K-1
C. acceleration C. 2.0 x 10-3K-1
D. impulse D. 1.0 x 10-3K-1
E. force E. 1.0 x 10-6K-1

10. The diagram beside represents a block-and- 14. A waterfall is 420m high. Calculate the
tackle pulley system on which an effort of difference in temperature of the water
50N is just able to lift a load of weight W. If between the top and bottom of the
the efficiency of the machine is 40% find the waterfall. Neglect heat losses. (g = 10.0ms-
value of W. 2
, specific heat capacity of water = 4.20 x
103 JKg-1K-1)
A. 300N A. 0.1˚C
B. 200N B. 1.0˚C
C. 140N C. 4.2˚C
D. 120N D. 42.0˚C
E. 20N E. 100.0˚C

11. A boatman facing north wants to cross a


flowing river to a point directly opposite its
position at the other bank. If the river is
flowing eastwards, in what direction he row
his boat?
A. West 15. The set-up illustrated above shows a
B. East capillary tube of uniform cross-section area
C. North-west in two different arrangements. Using the
data in the diagrams calculate the pressure D. ice absorbs latent/heat during melting
of the atmosphere. E. ice makes better thermal contact than
A. 76cm of Hg water
B. 75cm of Hg
C. 74cm of Hg 21. The saturation vapour pressure of a liquid
D. 60cm of Hg depends on its
E. 45cm of Hg A. volume
B. temperature
16. A fixed mass of gas of volume 600cm3 at a C. mass
temperature of 27˚C is cooled at constant D. density
pressure to a temperature of 0˚C. What is E. pressure
the change in volume?
A. 54cm3
B. 273cm3
C. 300cm3
D. 546cm3
E. 600cm3 22. The graph below shows the expansion of
water as the temperature increases from
17. The boiling point of a liquid depends on the 0˚C. Which of the following deductions from
following except the the graph are true?
A. nature of the liquid
B. external pressure
C. volume of the liquid
D. impurities present in the liquid
E. degree of its molecular cohesive force
Temperature 0C
18. A steam trap is a component of the
apparatus used in determining the specific
latent heat of vaporation of steam. In the I. Water has its maximum density at Q
steady state, the steam trap II. The volume of water is greater at 0˚C than
A. stores the steam for future use 4˚C
B. prevents the steam from escaping III. The volume of water decreases uniformly
C. ensures that only dry steam gets into the when cooled from 100˚C to 0˚C
calorimeter IV. When water solidifies, its volume increases
D. allows condensed steam to go into the A. I and II only
calorimeter B. I and III only
E. determines the quantity of steam used C. II and III only
D. I, II and III only
19. A tap supplies water at 26˚C while another E. I, II and IV only
supplies water at 82˚C. If a man wishes to
bathe with water at 40˚C, the ratio of the 23. Two mirrors are inclined as shown in the
mass of hot water to that of cold water diagram below, a ray of light RO strikes the
required is arrangement at O and emerges along PQ.
A. 1:3 The emergent ray has been deviated
B. 3:1 through
C. 3:7
D. 7:3
E. 15 : 8
A. 230˚
20. Melting ice cools an orange drink far better B. 180˚
than the same mass of ice-cold water C. 120˚
because D. 60˚
A. melting ice is at a lower temperature than E. 30˚
ice-cold water
B. water has a higher specific heat than ice 24. An image which can be formed on a screen
C. floats and cools the air above the drink is said to be
A. virtual C. Lens
B. blurred D. Diaphragm
C. inverted E. Focusing ring
D. erect
E. real 30. In the arrangement illustrated below, Y and
B are yellow and blue transparent light
25. Calculate the refractive index of a the filters respectively. The colour of a white
material of the glass block shown in the opaque object when viewed through the
diagram below if YZ = 4cm. filters is

A. 0.40 A. blue
B. 0.60 B. yellow
C. 1.50 C. black
D. 1.67 D. red
E. 2.50 E. green

26. A ray of light is incident at an angle of 30˚ 31. Which of the following is not a mechanical
on a glass prism of refractive index 1.5. wave
Calculate the angle through which the ray is A. Wave propagated in stretched string
minimally deviated in the prism. (the B. Waves in closed pipes
medium surrounding the prism is air) C. Radio waves
A. 10.5˚ D. Water waves
B. 19.5˚ E. Sound waves
C. 21.1˚
D. 38.9˚ 32. Which of the following remain(s) unchanged
E. 40.5˚ as light travels from one medium to the
other?
27. Which of the following are true of plane- I. Speed
polarised light? II. Wavelength
I. Plane polarization of light is the formation of III. Frequency
hydrogen bubbles on the particles of light A. I only
II. Plane polarization of light is the splitting of B. II only
light into its spectra components C. III only
III. A plane-polarized light vibrates in one plane D. I and II only
IV. Polarization of light is characteristics of E. II and III only
transverse vibration
A. I and II only 33. Which of the following statements above
B. I and IV only wave is/are correct?
C. II and III only I. A wavefront is a line which contains all
D. III and IV only particles whose vibrations are in phase
E. I and III only II. The direction of propagation of a wave is
the line drawn parallel to the wavefront
28. At which of the following distances from the III. A wavefront is a circle which is common to
lens should a slide be placed in a slide all particles that are to be in the same state
projector if f is the focal length of the of disturbance
projection lens? A. I only
A. Less than f B. II only
B. Greater than 2f C. III only
C. Greater than f but less than 2f D. I and II only
D. Equal to f E. II and III only
E. Equal to 2f
34. What type of motion does the skin of a
29. What part of the camera corresponds to the talking drum perform when it is being struck
iris of the eye? with the drum stick
A. Shutter A. Random
B. Film B. Rotational
C. Vibratory
D. Translational
E. Circular B.

35. The sound from a bell in an enclosed jar


gradually faints away while the jar is being C.
evacuated. Which of the following explains
this observation?
A. The pressure is reduced D.
B. The sound waves are pumped out
C. The waves are absorbed in the inner walls
D. There is no more material medium E.
E. The air is disturbed
40. A particle of charge q and mass m moving
36. Which of the following statements is true of with a velocity v enters a uniform magnetic
the diagram shown below? field B in the direction of the field. The force
on the particle is
A. qvB
B. mqvB
C. qvB/m
A. The two points A and B are in phase D. mvB/q
B. Distance AB is half the wavelength of the E. O
wave
C. AC and BD are the amplitudes of the wave 41.
D. The two points A and B represents the wave If the frequency of the e.m.f source in the
crests a.c circuit illustrated above is 500/x Hz,
E. The wave has four crests what is the reactance of the inductor?
A. /450
37. A man standing 300m away from a wall B. 0.9
sounds a whistle. The echo from the wall C. 450/
reaches him 1.8s later. Calculate the D. 450
velocity of sound in air. E. 900
A. 540.0ms-1
B. 333.3ms-1 42. Two objects of masses 80kg are separated
C. 270.0ms-1 by a distance of 0.2m. If the gravitational
D. 166.7ms-1 constant is 6.6 x 10-11Nm-2kg-2, calculate the
E. 83.3ms-1 gravitational attraction between them.
A. 4.9 x 10-9N
38. In which of the following is a stationary B. 1.3 x 10-8N
wave produced? C. 6.6 x 10-8N
I. A vibrating tuning fork held near the end of D. 6.6 x 10-6N
a resonate tube close at the end E. 2.6 x 10-5N
II. A string tightly stretched between two
points and plucked at its middle 43. The capacitors C1 and C2 are connected as
III. The prongs of a tuning fork vibrating in air shown in the diagram below. The
A. I only capacitance of C2 is twice C1. When the key
B. II only is opened, the energy stored up in C1 is W. If
C. I and II only the key is later closed and the system is
D. II and III only allowed to attain electrical equilibrium, the
E. I, II and III only total energy stored in the system will be
A. ½W
39. Which of the following modes of vibration of B. 2
/3 W
a stretched string is the first overtone? C. W
D. 2W
A. E. 3W
44. In which of the points labeled A, B, C, D and transformer is used. If the number of turns
E on the conductor shown below would in the primary coil of the transformer is 900,
electric charge tend to concentrate most? calculate the number of turns in the
secondary coil of the transformer.
A. A. 30
B. B. 240
C. C. 248
D. D. 450
E. E. 1248

45. Which of the following is stored by a dry 50. The diagram below shows a current-carrying
Leclanche cell? wire between the poles of a magnet. In
A. Chemical energy which direction would the wire tend to
B. Nuclear energy move?
C. Solar energy
D. Heat energy
E. Electrical energy

46. Using the data in the circuit illustrated A. Into the paper
below, calculate the value of R B. Out of the paper
C. Towards the north pole of the paper
D. Towards the south pole of the magnet
E. Towards the top of the page
A. 0.02Ω
B. 0.05Ω 51. A induction coil is generally used to
C. 5.00Ω A. rectify an alternating current
D. 20.0Ω B. produce a larges input voltage
E. 50.0Ω C. smoothen a pushing direct
D. modulate an incoming radio signal
47. A 90W immersion heater is used to supply E. produce a large output voltage
energy for 5 minutes. The energy supplied
is used to completely melt 160g of a solid at 52. The unit of stress is
its melting point. Calculate the specific A. Nm
latent heat of fusion of the solid. B. N
A. 2.81 Jg-1 C. Nm-2
B. 6.25Jg-1 D. Nm2
C. 8.89Jg-1 E. Nm-1
D. 168.751Jg-1
E. 533.33Jg-1 53. Which of the following statements about
viscocity are correct?
48. In the circuit shown below, R is the resistor A. viscocity opposes the gravitational force on
whose resistance increases with increase in that ball
temperature. L1 and L2 are identical lamps. B. viscocity opposes the upthrust on the ball
If the temperature of R increases C. viscocity is in the same direction as the
upthrust on the ball
D. viscocity is in the same direction as the
upthrust on the ball
A. L1 becomes brighter and L2 becomes E. the ball falls faster the more viscuous the
dimmer liquid is
B. L1 becomes brighter and L2 does not change
C. L2 becomes dimmer and L1 does not change
D. L1 becomes dimmer and L2 does not change 54. Which of the following statements is not
E. L1 and L2 becomes brighter correct. Isotapes of an element have
A. the same number of electric charges on the
49. A house is supplied with a 240V a.c mains. nucleus
To operate door bell rated at 8V, a B. the same chemical properties
C. different nucleon numbers B. 2.21eV
D. different proton numbers C. 4.42eV
E. different atomic masses D. 5.76eV
E. 11.51eV
55. Which of the following representation is
correct for an atom X with 28 electrons and 60. Which of the following has the highest
30 neutrons? surface tension?
A. 30X A. Cold water
28 B. Soapy water
C. Warm water
B. 28X D. Oily water
30 E. Salt water

C. 58X
30
1991
D. 58X PART B THEORY
28
1. (a) Explain the terms uniform
E. 30X acceleration
2 and average speed.

56. When a metal surface is irradiated, (b) A body at rest is given an initial
photoelectrons may be ejected from the uniform acceleration of 8.0ms-2 after
metal. The kinetic energy of the ejected which the acceleration is reduced to
electrons depends on the e 5.0ms-2 for the next 20s. The body
A. source of the radiation maintains the speed attained for 60s
B. intensity of the radiation after which it is brought to rest in
C. detection device for the electrons 20s. Draw the velocity-time graph of
D. amplitude of the radiation the motion using the information
E. frequency of the radiation given above.

57. An electron of charge 1.6 x 10-19c is (c) Using the graph, calculate the:
accelerated in vacuum from rest at zero volt (i) maximum speed attained
towards a plate of 40K.v. Calculate the during the motion
kinetic energy of the electron. (ii) average retardation as the
A. 4.0 x 10-25J body is being brought to rest
B. 4.0 x 10-21J (iii) total distance traveled during
C. 6.4 x 10-20J the first 50s
D. 6.4 x 10-15J (iv) average speed during the
E. 2.5 x 1020J same interval as in II

58. The half-life of a radioactive substance is 2 2. (a) Distinguish between temperature


second. Calculate the decay constants. and
A. 0.035s-1 heat. State the units in which they
B. 0.151s-1 are measured.
C. 0.347s-1 (b) (i) Describe with the aid of
D. 0.576s-1 labeled
E. 1.386s-1 diagram, how the upper fixed
point is determined for a
59. The work function of a metal is 4.65eV and mercury-in-glass
the metal illuminated with a radiation of thermometer, State one
6.86eV. What is the kinetic energy of the precaution to ensure accurate
electrons ejected from the surface of the results.
metal? (ii) State one advantage which a
A. 1.48eV constant-volume gas
thermometer has over other heated of 100˚C is transferred into
thermometers and one the calorimeter and the final
reason why it is seldom used temperature of the mixture T1 is
as an everyday laboratory recorded. The colorimeter is then
instrument emptied. The experiment is repeated
with water at the initial temperature.
(c) Using the kinetic theory of matter, T0 and of masses, m = 60, 70, 80, 90
explain why evaporation causes and 100g to obtain the final
cooling. temperatures. T1 = T2, T3, T4, T5 and
T6 of the mixtures respectively. The
3. (a) Explain the terms reactance and room temperature and the six final
impedance in and a.c circuit. temperatures have been drawn to
scale in fig.1. Measure the
(b) A source of e.m.f 240V and temperature T0 and T1 on the
frequency 50Hz is connected to a diagram. Calculate (100 – T) and (T1,
resistor, an inductor and a capacitor - T0) and Z = 100 – T1 Tabulate your
in series. When the current in the T1 – T0
capacitor is 10A, the potential readings.
difference across the resistor is 140A
and that across the inductor is 50V. Plot a graph of Z against m, starting
Draw the vector diagram of the both axes from the origin (O, O).
potential differences across the Determine from your graph the
inductor, capacitor and the resistor. (i) slope of the graph
Calculate the: (ii) value of the final temperature
(i) potential difference across eof the mixture if water of
the capacitor mass 30g was used in the
(ii) capacitance of the capacitor experiment. State two
(iii) inductance of the indicator precautions you would take if
you were to perform this
4. (a) (i) By means of a labeled- experiment in the laboratory
diagram,
describe the mode of b. (i) 200g of water at 100˚C is
operation of a modern X-ray poured into 50g of water at
tube. 30˚C. Assuming that the heat
(ii) State the energy information absorbed by the container is
which take place during the negligible, calculate the final
operation. temperature of the mixture
(Neglect heat losses to the
(b) Explain the terms hardness and surrounding).
intensity as applied to X-rays tube.
(ii) Explain how heat loss by
(c) (i) State three uses of X-rays conduction is minimized in a
(ii) State one hazard of over- vacuum flask
exposure to X-rays in a
radiological laboratory, (iii) State two differences
indicating two safety between boiling and
precautions. evaporation of liquid.

2. (a) In the diagram shown below, an


1991 illuminated object is placed on the
PAPER I PRACTICAL axis of a thin converging lens and its
image is produced on a screen. The
1. (a) A copper calorimeter contains water procedure is repeated by adjusting
of the object positions to obtain
mass, m = 50g at room corresponding image positions. Fig. 2
temperature, T0. A block of copper shows the object position 01, 02, 03,
04, 05, and the corresponding image repeated four times to obtain values
position, 11,12,13,14, and 15 of R, I1 and I2. Fig. 3 shows the
resistance R(i) – (v) in the resistance
box, and the corresponding values of
the currents I1 (I – v) through
ammeters A1 and 12, (I – v) through
ammeters A2 are respectively shown
by fig. 4 and fig.5. Read and record
Measure and record the object the values of R, I1 and I2. Calculate Z
distances u and the image distances = I2+(I1 – I2) and R-1 for each case.
v. Evaluate (u + v) and uv for each Tabulate your readings. Plot a graph
case. Tabulate your readings. R-1 against Z. Determine the slope s
of the graph, evaluate I/s. State two
Plot a graph of (u = v) against uv. precautions you would take if you
Determine slope s of the graph. were to perform this experiment in
Evaluate I/s. State two precautions the laboratory.
you would take if you were to
perform this experiment in the b. (i) What is meant by the
laboratory. electromotive force of a cell
(ii) State four factors on which
b. (i) Draw a ray diagram the resistance of a given wire
illustrating how a plane mirror depends.
can be used to determine the (iii) With the aid of a diagram
focal length of a converging below, explain how you would
lens. convert a millimeter into a
voltmeter.
(ii) explain how a converging lens
could be used to ignite a
piece of carbon paper

(iii) A illuminated object is placed


on the axis of a converging
lens and a magnified image is
obtained on the screen. If the
distance of the image from
the lens is 45cm and the
magnification is 2, calculate
the focal length of the lens.

3. (a)

In the circuit diagram shown above,


R is a resistance box. X is an
unknown resistance. E is a battery. K
is a key and A1 and A2 are ammeters.
The top of the resistance box is also
shown above, where shaded circles
indicated that plugs are inserted and
unshaded circles indicated that plugs
have been removed. With the circuit
connected up and key closed, the
resistance box is set at a new value
to obtain new corresponding values
of I1 and I2. The experiment is
1992 the distance covered by the body during the
SSCE PHYSICS acceleration.
PAPER 2 OBJECTIVE A. 12m
B. 24m
1. Which of the following is scalar quantity? C. 48m
A. Momentum D. 72m
B. Acceleration E. 96m
C. Displacement
D. Distance 6. Which of the following has the same unit as
E. Force the moment of a force?
A. Force
2. The diagram below represents the trajectory B. Power
of a projectile with an initial velocity U. C. Work
Calculate the time taken to reach the D. Momentum
maximum height. (Take gas acceleration E. Impulse
due to gravity)
7. Solid weighs 0.040 N in air and 0.024 N
A. U2sin2θ when fully immersed in a liquid of density
g 800kg m-3. What is the volume of the solid?
(g = 10 ms-2)
B. 2Usinθ A. 2.0 x 106m3
g B. 2.5 x 10-6m3
C. 3.0 x 10-6m3
C. U cos θ D. 2.0 x 10-5m3
g E. 3.0 x 10-5m3

D. U2sinθ 8. Which of the following will reduce the


g frequency of oscillation of a simple
pendulum?
E. U sinθ A. increasing the mass of the bob
g B. decreasing the mass of the bob
C. increasing the length of the string
3. What change in velocity would be produced D. decreasing the length of the string
on a body of mass 4kg if a constant force of E. increasing the amplitude of oscillation
16N acts on it for 2s?
A. 0.5ms-1 9. A force of 20N applied parallel to the
B. 2.0ms-1 surface of a horizontal table, is just
C. 8.0ms-1 sufficient to make a block of mass 4kg move
D. 32.0ms-1 on the table. Calculate the coefficient of
E. 128.0ms-1 friction between the block and the Table (g
= 10ms2)
4. Which of the following statements about A. 0.05
solid friction is/are correct. B. 0.20
I. Friction depends on the nature of the C. 0.50
surface in contact D. 0.80
II. Friction depends on the area of the contact E. 2.00
III. Friction always acts in the direction of
motion 10. A barometer can be used in determining the
A. I only I. height of a mountain
B. II only II. depth of a mine
C. III only III. dew point
D. I and II only Which of the following is correct?
E. I, II and III A. I, II and III
B. II and III only
5. A body accelerates uniformity from rest at C. I and III only
the rate of 3ms-2 for 8 seconds. Calculate D. I and II only
E. III only E. Blue

11. A body of mass 5kg falls from a height of 16. A gas which obeys Charles’ law exactly has
10m above the ground. What is the kinetic a volume of 283cm3 at 10˚C. what is its
energy of the body just before it strikes the volume at 30˚C?
ground? (Neglect energy losses and take g A. 142cm3
as 10ms2) B. 293cm3
A. 5J C. 303cm3
B. 25 J D. 566cm3
C. 250 J E. 849cm3
D. 500 J
E. 625 J 17. Which of the following best explains why a
person suffers a more severe burn when his
12. Steel bars, each of length 3.0m at 28˚C are skin is exposed to steam than when boiling
to be used for constructing a rail line. If the water pours on his skin?
linear expansivity of steel is 1.0 x 105˚C-1, A. Steam is at a higher temperature than
what is the safety gap that must be left boiling water
between successive bars if the highest B. Steam possesses greater heat energy per
temperature expected is 40˚C? unit mass than boiling water
A. 1.2 x 10-1cm C. Steam spreads more easily over a wider
B. 8.4 x 10-2 area of the skin that boiling water
C. 7.2 x 10-2cm D. Steam penetrates more deeply into the skin
D. 3.6 x 10-2cm than boiling water
E. 1.8 x 10-2cm E. The specific latent heat of vaporization is
released in changing from boiling water to
13. Which of the following statements is/are steam
correct?
I. Pure water freezes at 0˚C under normal 18. A small circular membrane is 10cm below
pressure the surface of a pool of mercury when the
II. Water has its highest density at 4˚C marometric height is 76cm of mercury. If
III. The volume of a given amount of water at the density of mercury is 13,600kg m-3,
0˚C is less that its volume at 4˚C what is the pressure of the membrane in
A. I only Nm-3? (g = 10ms2)
B. II only A. 1.17 x 107 Nm-2
C. I and II only B. 6.80 x 105 N m-2
D. I and III only C. 1.17 x 105 N m-2
E. II and III only D. 1.03 x 103 N m-2
E. 1.36 x 104 N m-2
14. A given mass of gas at a temperature of
30˚C is trapped in a tube of volume V. 19. Which of the following instruments is used
Calculate the temperature of the gas when to measure relative humidity?
the volume is reduced to two-third of its A. Hydrometer
marginal value by applying a pressure twice B. Barometer
the original value? C. Manometer
A. -71˚C D. Hypsometer
B. 40˚C E. Hygrometer
C. 131˚C
D. 313˚C 20. A metal of mass 1.5kg was heated from
E. 404˚C 27˚C to 47˚C in 4 minutes by a boiler ring of
75 W rating. Calculate the specific heat
15. Which of the following colours of surfaces capacity of the metal. (Neglect heat losses
will radiate heat energy best? to the surrounding)
A. Red A. 2.5 x 10-3 J kg-1˚C-1
B. White B. 6.0 x 102 J kg-1˚C-1
C. Black C. 2.5 x 102 J kg-1˚C-1
D. Yellow D. 1.4 x 10-2 J kg-1˚C-1
E. 1.0 x 10J kg-1˚C-1

21. Which of the following diagrams shows the 25. In the experiment to measure the focal
correct images of the letters JP as seen length f of a converging lens, object
through a plane mirror? distances u and corresponding image
distances v were measured and 1__ plotted
A. 1_
U V
obtain the graph illustrated above.
B. How would F be determined from this
graph?
A. f = the slope of the graph
B. f = the intercept on the 1/u axis
C. C. f = the intercept on the 1/v axis
D. f = reciprocal of the slope
E. f = reciprocal of the intercept on either axis
D.
26. The image of a pin formed by a diverging
lens of focal length 20cm is 5 cm from the
E. lens. Calculate the distance of the pin from
the lens.
A. -3.3cm
B. 3.3cm
22. A ray of light strikes a plane mirror at an C. 10.0cm
angle of incidence i. Determine in terms of I, D. 15.0cm
the angle of deviation of the ray after E. 20.0cm
reflection from the mirror.
A. i 27. As an object is moved close to the focus of a
B. 2i converging lens from infinity, its image
C. 90˚-i I. moves closer to the lens
D. 90˚+i II. moves away from the lens
E. 180˚ - 2i III. remains the same size
IV. becomes smaller.
23. Images formed by a convex mirror are Which of the statements above is/are
always correct?
A. inverted, real and diminished A. III and IV only
B. inverted, virtual and diminished B. I and III only
C. erect, virtual and diminished C. IV only
D. erect, real and magnified D. III only
E. erect, virtual and magnified E. II only

24. Which of the following statements is not 28. A real image of an object formed by a
correct for a light ray passing through a converging lens of focal length 15cm is
rectangular glass block which is surrounded three times the size of the object. What is
by air? It the distance of the object from the lens?
A. suffers a displacement at the point of A. 30cm
emergence B. 25cm
B. emerges parallel to the incident ray C. 20cm
C. is partly reflected at the point of incidence D. 15cm
D. is deviated at the point of emergence E. 10cm
E. is refracted in the block
29. The eye defect illustrated in the diagram
below can be corrected by a
A. convex mirror A. Radio waves
B. concave mirror B. Light waves
C. diverging lens C. X-rays
D. converging lens D. Infra-red rays
E. plano-converging lens E. Ultraviolet rays

30. When an astronomical telescope is in 36. How far from a cliff should a boy stand in
normal adjustment, the focal length of the order to hear the echo of his clap 0.9s later?
objective is 50 cm and that of the eyepiece (Speed of sound in air – 330ms-s1)
is 2.5cm. What is the distance between the A. 36.67m
lenses? B. 74.25m
A. 20.0cm C. 148.50m
B. 47.5cm D. 297.00m
C. 50.0cm E. 366.67m
D. 52.5cm
E. 125.0cm 37. Which of the following waves are
longitudinal waves?
31. Which of the following is used for controlling I. Ripples on the surface of water
the amount of light entering the eye? II. Waves produced by a tuning fork vibrating
A. Cornea in air
B. Pupil III. Light waves
C. Iris IV. Waves produced by a flute
D. Optic Nerve A. I and II only
E. Ciliary muscle B. I and III only
C. II and III only
32. The angle of deviation of light of various D. II and IV only
colours passing through a glass prism E. III and IV only
decreases in the order of
A. blue, orange and red 38. Which of the following statements is not
B. red, blue and orange true?
C. blue, red and orange A. Musical notes consist of the combinations of
D. red, orange and blue sound of regular frequency
E. orange, blue and red B. The combinations of overtones produced by
an instrument determines the quality of the
33. The change of the direction of a wavefront sound
as a result of a change in the velocity of the C. The loudness of sound is determined by its
wave in another medium is called frequency
A. refraction D. The pitch of a note depends on the
B. reflection frequency of vibration of the source
C. diffraction E. Noise can be produced by a combination of
D. interference sound of irregular frequency
E. polarization
39. A pipe closed at one end is 1.0m long. The
34. The diagram below shows a wavefrom in air in the pipe is set into vibration and a
which energy is transferred from A to B in a fundamental note of frequency 85 Hz is
time of 2.5 x 10-3s. Calculate the frequency produced. Calculate the velocity of sound in
of the wave. the air. (Neglect end correction)
A. 340.0ms-1
A. 2.0 x 103Hz B. 330.0ms-1
B. 1.0 x 103Hz C. 170.0ms-1
C. 4.0 x 10-3Hz D. 85.0ms-1
D. 1.0 x 10-3Hz E. 42.5ms-1
E. 5.0 x 10-4Hz
40. The diagram below shows the magnetic
35. Which of the following radiations have the field around a bar magnet which has its N-
highest frequency?
pole pointing north. The neutral points in 46. Calculate the time in which 4.8kg J of
the field are likely to occur at energy would be expended when an electric
heater of resistance 1.8 x 103Ω is used on a
A. 1 and 2 240V mains supply. (Neglect heat losses to
B. 3 and 4 the surroundings).
C. 5 and 6 A. 150.0s
D. 7 and 8 B. 90.0s
E. 9 and 10 C. 36.0s
D. 20.0s
41. A negatively charged rod is brought near E. 2.7s
the cap of a gold-leaf electroscope. The cap
is earthed momentarily while the rod is near 47. Lenz’s law of electromagnetic induction
to it. The rod is then removed. Which of the states that
following is correct? A. electromotive force is induced whenever
A. The cap will be positively charged and there is a change in the magnetic flux linked
negatively charged leaves will diverge with the circuit
B. The cap will be negatively charged and B. the induced current in a conductor is in such
positively charges leaves will diverge a direction as to oppose the change
C. The cap and the leaves will be positively producing it
charge. C. the induced e.m.f in a circuit is proportional
D. The cap and the leaves will be negatively to the rate of change of the number of lines
charged of force linking the circuit
E. The leaves will not diverge because the cap D. a force is exerted on a current carrying
has been earthed conductor in a magnetic field
E. the induced e.m.f is proportional to the
42. Calculate the rate at which energy is used current producing the magnetic flux
up in the circuit illustrated by the diagram
below. 48. Which of the following statements is not
correct?
A. 0W A. a magnetic field is a region where a
B. 2W magnetic force can be detected
C. 4W B. magnetic fields are scalar quantities
D. 8W C. the magnitude of the magnetic force
E. 16 W experienced by a moving charge depends
on the speed on the charge
43. Which of the following is not a consequence D. the angle between the direction of the
of hydrogen bubbles covering the copper earth’s magnetic field and horizontal is
plate of a primary cell? called then angle of dip
A. Polarization E. iron fillings can be used to trace out the
B. Local action magnetic around a bar magnet
C. Formation of hydrogen electrode
D. Increase in the resistance of the cell 49. Calculate the energy stored in a 20μF
E. Generation of less current by the cell capacitor if the potential difference between
the plates is 40V.
44. Calculate the length of a constant wire of A. 3.2 x 10-3 J
cross-section area 4 π x 10- π m2 and B. 1.6 x 10-3 J
resistivity 1.1 x 10-6Ωm required to C. 8.0 x 10-4 J
construct a standard deviation resistor of D. 4.0 x 10-4 J
resistance 21Ω. (Take π as 22/7). E. 2.0 x 10-4 J
A. 0.42
B. 2.40m 50. The diagram below show lines in electric
C. 6.25m fields. In which of the diagrams would a
D. 183.75m positive test charge experience the least
E. 590.00m force as it moves from X to Y?

A. I only
B. II only
C. III only
D. I and II only
E. II and III only

51. What is the electric potential at point A. Accelerated and they continue in a straight
distance r from proton of charge q placed in line
a medium of permittivity ∈ 0? B. directionally reversed at the end of the
` plates
A. q2__ C. deflected towards plate Y
4 π ∈ ,r2 D. attracted by both plates
E. deflected towards plate X
B. q__
4 π ∈ ,r 56. An element whose half-life is 3 years has N
atoms. How many atoms would have
C. q2__ decayed after 9 years?
4 π ∈ 0r A. 1 N atoms
8
D. qr__
4π ∈, B. 1 N atoms
3
E. 4π ∈ ,q2r
C. 2 N atoms
52. If the frequency of the a.c circuit illustrated 3
above is 500 Hz. What would be the
reactance in the D. 5 N atoms
π circuit? 6
A. 0.01Ω
B. 190.91Ω E. 7 N atoms
C. 200.00Ω 8
D. 795.46Ω
E. 1000.00Ω 57. When the nucleus of a uranium atom is split
into two fragments of nearly equal mass,
53. Which of the following statements is correct the sum of the masses of the fragments is
about cathode rays?. They are fast moving less than the mass of the original nucleus.
A. atoms This difference is a measure of the
B. ions A. experimental error in calculating the
C. neutrons separate masses
D. protons B. change of a momentum of each fragment
E. electrons C. potential energy lost
D. nuclear energy released
54. Which of the following gives rise to the line E. kinetic energy lost
spectra obtained from atoms?
A. Kinetic energy of a moving atom 58. A sheet of paper is placed in the path of a
B. Potential energy of an electron inside an beam of radiations from a radioactive
atom source. Which of the following radiations will
C. Change of an electron from a higher to a pass through the paper?
lower energy level in the atom I. Alpha rays
D. Disturbed proton in the nuclues II. Beta Rays
E. Excitation of an electron in the atom III. Gamma rays
A. I only
55. A stream of alpha particles is projected into B. II only
an electric field between two plates X and Y C. III only
as shown in the diagram below. Which of D. II and III only
the following is correct? The particles are E. I, II and III
Alpha
particles
59. The process by which a metal, heated to a
high temperature, gives off electrons from (b) Explain with aid of diagrams, how a
its surface is known as concave mirror could be used to:
A. photoelectric emission (i) ignite a piece of carbon
B. thermionic emission paper;
C. radiative emission (ii) produce an exact copy of a
D. field emission picture on a screen
E. secondary emission
3. (a) Draw a simple labeled diagram
60. An elastic string of length I is elastically illustrating
stretched through a length e by a force F. the principle of step-down
The area of cross section of the string is A transformer and explain how it works
and its Young’s modulus is E. Which of the
following expressions is correct? (b) State three ways by which energy is
A. F = EAe2 lost in a transformer and how they
I can be minimized.

B. F = EAI (c) If a transformer is used to light a


e lamp rated at 60W, 220V from a
4400V a.c supply, calculate the
C. F = EAe (i) ratio of the number of turns of
I the primary coil to the
secondary coil in the
D. F = EAe transformer.
F (ii) current taken from the mains
circuit if the efficiency of the
E. F = EA transformer is 95%.
eI
4. (a) Explain what is meant by
1992 photoelectric
PART B THEORY emission.

1. (a) Explain what is meant by the (b) Draw a labeled diagram showing the
following structure of a simple type of a
statements: The specific latent heat photocell and explain its mode of
of fusion of ice is 3.4 x 102Jkg-1 operation.

(b) Describe an experiment to determine (c) State four applications of


the specific latent heat of fusion of photoelectric emission.
ice. State two precautions necessary
to obtain an accurate result. (d) In a photocell, no electrons are
emitted until the threshold frequency
(c) Using the kinetic theory of matter. of light is reached. Explain what
Explain why ice can change to water happens to the energy of the light
of 0oC without any change in before emission of electrons begins.
temperature. State one factor that may affect the
number of emitted electrons.
2. (a) (i) What is meant by resonance?
(ii) Outline the necessary steps
taken in a simple experiment 1992
to illustrate resonance. PAPER I PRACTICAL
(iii) Explain why a vibrating
turning fork sounds louder 1.
when its stem is pressed
against a table top than when
held in air.
(a) Suspend a metre rule horizontally on the diagram above. Tabulate your
the knife-edge supplied. Adjust the readings.
metre rule carefully until it settles Plot a graph of d against i. Draw a
horizontally. Read and record the smooth curve through your points.
point of balance G of the metre rule. Determine the value of d when I is
Keep the knife-edge at the point G 90˚. State two precautions you took
throughout the experiment. Suspend to obtain accurate results [Attach
the object labeled Q at the 20cm your traces to your answer booklet].
mark of the metre rule. On the other
side of G, suspend the mass m= 20g (b) (i) State Snell’s law of refraction
and adjust its position until the and explain why refraction
metre rule settles horizontally. Read occurs at the boundary
off the position R of the mass m on between two media.
the metre rule. Record the distance y (ii) Sketch a diagram to show the
between G and R. Also, record the path of the ray through the
value of m. Repeat the procedure glass block when the angle of
keeping edge at G and the object Q incidence I = 90˚ in the
at the 20cm mark but using the experiment above.
masses m = 30, 40, 50 and 60g on (iii) A coin lies at the bottom of a
other side of G. In each case, record tank containing water to a
the distance y and the corresponding depth of 130cm. If the
value of m. Determine the values of refractive index of water is
I
/y, and tabulate your readings. Plot a 1.3 calculate the apparent
graph of m against I/y. Determine displacement of the coin
the slope s of the graph. Evaluate when viewed vertically from
s/x. State two precautions you took above.
to obtain accurate results. 3. (a)

(b) (i) Explain what is meant by the


centre of gravity of a body.
State how this is related to
the stability of the body.
Connect the metre bridge as shown
(ii) A body of mass 58g is in the diagram above. Ab is bare
suspended at the 20cm mark constantan wire. For a length L =
of uniform metre rule. The 20cm of the wire, connect the circuit
metre rule is adjusted on a and locate a balance point S on the
pivot until it settles wire PQ. Record the value of y and
horizontally at the 40cm evaluate y-1. Repeat the experiment
mark. Determine the mass of with L = 40, 60, 80, and 100cm. In
the metre rule. each case, determine and record the
corresponding value of y and y-1.
(iii) State the condition of Tabulate your readings. Starting both
equilibrium for a body acted axes from the origin, plot a graph of
upon by a number of co- L against y-1. Determine the slope s
planar parallel forces. of the graph and the intercept I on
2. (a) the vertical axis. State two
precautions you took to obtain
accurate results.
(b) (i) Given that L = 200y-1 – 2 in
Trace the paths of five rays through the
the given glass block for angles θ = r r
65˚, 55˚, 35˚, and 25˚. For each ray, experiment above, use your
measure and record the angle of graph to determine the value
incidence I and the corresponding of r.
lateral displacement d as shown in
(ii) State two advantages of M1 + M2
using a potentiometer over a
voltmeter for measuring B. M2U___
potential difference. M1 + M2

(iii) State four factors on which C. M1 + M2


the resistance of a wire M1U
depends.
00
1993 D. M1 + M2
SSCE PHYSICS M1U
PAPER 2 OBJECTIVE
E. M1U__
1. Which of the following is not an example of M1 – M2
a force?
A. Tension 6. A body of mass, m = 0.5kg is suspended by
B. Weight a string and pulled by a horizontal force of
C. Friction 12n as shown in the diagram below.
D. Mass Calculate the tension T in the string if the
E. Thrust body is in equilibrium. (Take g as 10ms-1)
A. 7.0N
2. A body moves along circular path with B. 10.9
uniform angular speed of 0.6 and s-1 and at C. 12.5
a constant speed of 3.0 m s-1. Calculate the D. 13.0
acceleration of the body towards the centre E. 17.0N
of the circle.
A. 25.0ms-2 7. A particles states from rest and moves with
B. 5.4ms-2 a constant acceleration of 0.5ms-2.
C. 5.0ms-2 Calculate the time taken by particle to vocer
D. 1.8ms-2 a distance of 25m.
E. 0.2ms-2 A. 2.5s
B. 7.1s
3. Which of the following is a derived unit? C. 10.0s
A. Ampere D. 50.0s
B. Kilogramme E. 100.0s
C. Second
D. Ohm 8. A block of material of volume 2 x 103kg m-3
E. Kelvin and density 2.5 x 103 kg m-3 is suspended
from a spring balance with half the volume
4. A boy timed 20 oscillations of a certain of the block immersed in water. What is the
pendulum three times and obtained 44.3s, reading of the spring balance? (Density of
45.5s and 45.7s respectively. Calculate the water 1.0 x 103 kg
mean period of oscillation of the pendulum. m-1, g = 10ms-2)
A. 0.13s A. 0.10N
B. 2.22s B. 0.25N
C. 2.26s C. 0.30N
D. 44.30s D. 0.40N
E. 45.17s E. 0.50N

5. A boy mass M1 moving with a velocity U 9. An object is projected with velocity of 100
collides with a stationary body of mass M2 ms-1 from the ground level at an angle 0 to
and both move with a common velocity V. If the vertical. If the total time of flight of the
linear momentum is conserved, which of the projectile is 10s, calculate 0. [g = 10ms-2]
following expressions correctly represents A. 00
V? B. 300
A. M1U__ C. 450
D. 600 IV. Polarization is characteristics of transverse
E. 900 waves
A. I and II only
10. How far will a body move in 4 seconds if B. I and IV only
uniformly accelerated from rest at the rate C. II and III only
of 2m s-3? D. III and IV only
A. 32m E. None of the above
B. 24m
C. 16m 16. An image which cannot be formed on a
D. 12m screen is said to be
E. 8m A. Inverted
B. real
11. If the temperature of water is gradually C. virtual
increased from 0o to 4oC, the density of the D. erect
water within this range. E. blurred
A. increases for a while and then decreases
B. decreases for a while and then increases 17. Longitudinal waves cannot be
C. increases gradually A. diffracted
D. decreases gradually B. refracted
E. remains the same C. polarized
D. reflected
12. The expansion of solid can be considered a E. superposed
disadvantage in the
A. fire alarm system 18. The images formed by a diverging lens
B. thermostat always
C. riveting of steel plates A. diminished, virtual and inverted
D. balance wheel of a watch B. diminished, virtual and real
E. fitting of wheels on rims C. diminished, virtual and erect
D. magnified, virtual and erect
13. A solid metal cube of side 10cm is heated E. magnified, real and inverted
from 10˚C to 60˚C. If the linear expansivity
of the metal is 1.2 x 10-5K-1, calculate the 19. In the normal use of simple microscope, a
increase in its volume. person sees an
A. 0.6cm3 A. inverted, virtual and magnified image
B. 1.2cm3 B. erect, virtual and magnified image
C. 1.8cm3 C. erect, real and magnified
D. 3.6cm3 D. magnified, virtual and erect
E. 6.0cm3 E. inverted and real image the same size as
the object
14. A gas has a volume of 546cm3 at 0˚C. What
is the volume of the gas at –100˚C if its 20. A lens of focal lengths 15.0 cm forms an
pressure remains constant? upright image four times the size of an
A. 346cm3 object. Calculate the distance of the image
B. 446cm3 from the lens.
C. 546cm3 A. 11.3cm
D. 646cm3 B. 18.8cm
E. 746cm3 C. 37.5cm
D. 45.0cm
15. Which of the following are correct of plane- E. 75.0cm
polarized light?
I. Plane polarization of light is the formation of 21. An object is placed between two mirrors
hydrogen bubbles on the particles of light which are inclined at an angle of 12o˚ and
II. Plane-polarization of light is the splitting of facing each other. Determine the number of
light into its spectral components images observed in the two mirrors.
III. Plane-polarized light vibrates in one plane A. 1
only B. 2
C. 3 C. Belt
D. 4 D. Piano
E. 5 E. Sonometer
22. In a ripple tank experiment, a vibrating
plate is used to generate ripples in the 28. A short chain is usually attached to the back
water. If the distance between two of a petrol tanker and trailing behind it so
successive throughs is 3.5cm and the wave that the
travels a distance of 31.5cm in 1.5s, A. petrol tanker is balanced on the road
calculate the frequency of the vibrator. B. heat generated by friction in the engine can
A. 3.0HZ be conducted to the earth
B. 6.0Hz C. charges generated by friction in the tanker
C. 12.0Hz are conducted to the earth
D. 97.0Hz D. tanker can move slowly as the chain
E. 73.5Hz touches the road surface
E. chain can produce sound in resonance with
23. Which of the following have the longest the noise from the tanker’s engine
wave lengths
A. Infra-red rays 29. Which of the following indicates the correct
B. Gamma rays sequence of steps taken in charging a gold-
C. X-rays leaf electroscope by indication?
D. Ultra-violet rays I. Removing the ebonite rod
E. Radio waves II. Bringing a negatively-charged ebonite rod
close to the cap
24. A man stands in front of a tall wall and III. Earthing the cap
produces a sound. If he receives the echo of IV. Disconnecting the earthing
the sound two seconds later, calculate the A. III, II and IV
distance from the wall. (the speed of sound B. II, I, III and IV
in air is 300ms-1). C. III, IV,II and I
A. 600m D. II, III, IV and I
B. 400m E. III, IV, I and II
C. 330m
D. 328m 30. Which of the following instruments can be
E. 165m used to compare the relative magnitude of
similar charges on two given bodies?
25. Which of the following is correct about light A. Electrophorus
and sound waves? B. Ebonite rod
A. Both of them are associated with energy C. Proof plane
transfer D. Gold-leaf electroscope
B. Both of them need material media for E. Capacitor
propagation
C. They are both longitudinal waves 31. The unit of inductance is
D. Their velocities in air are equal A. Farad
E. Both of their can be polarized B. Henry
C. Hertz
26. The quality of a note produced by a D. Weber
vibrating string is determined by the E. Tesia
A. pitched of the note
B. tension of the string 32. A household refrigerator consumes
C. material of the strings electrical energy at the rate of 200W. If
D. presence of overtones electricity cost 5k per k Wh. calculate the
E. thickness of the string cost of operating the appliance for 30 days.
A. N7.20
27. Which of the following is a percussion B. N12.00
instrument? C. N30.00
A. Flute D. N33.33
B. organ E. N72.00
B. 21oVo sinwt
33. Calculate the escape velocity of a satellite
launched from the earth’s surface. (Take g C. IoVo
as 10ms-3 and the radius of the earth as 6.4 2
x 106m). D. IoVo sin2wt
A. 8.00 x 104ms-1 E. Io2Vo2
B. 1.13 x 104ms-1
C. 3.58 x 102ms-1 38. A point charge of 1.0 x 10-7C experiences a
D. 5.59 x 10-2ms-1 force of 0.01 N in a uniform electric field.
E. 3.95 x 10-2ms-1 Calculate the magnitude of the strength of
the field.
34. What is the gravitational potential due to A. 106 Vm-1
point mass m at a distance r from it? (G = B. 105 Vm-1
gravitational constant) gravitational) C. 10-4 Vm-1
A. -Gm2 D. 10-6 Vm-1
r2 E. 10-9 Vm-1

B. -Gm2 39. The potential difference between two points


r A and B situated at a distance d apart is V.
Which of the following expresses the
C. -m2 magnitude of the electric field intensity
Gr2 between the two points assuming the field
is uniform?
D. -Gm A. Vd
r B. dV-1
C. Vd-1
E. G2m2 D. V 2d
r E. d 2V

35. In a series R – C circuit, the resistance of the 40. Which of the following have the greatest
resistor is 4Ω and the capacitive reactance penetrating power?
is 3Ω. Calculate the impedance of the A. Beta rays
circuit. B. Alpha rays
A. 1Ω C. Gamma ray
B. 4Ω D. Electrons
C. 5Ω E. Neutrons
D. 7Ω
E. 12Ω 41. A particle of charge 5 C moves
perpendicularly to a magnetic field of
36. The vector diagrams shown below magnitude -.01T. If the velocity of the
represents the resistance, R, the capacitive charge is 1.5ms-1, calculate the magnitude
reactance, Xc, the inductive reactance, XL of the force exerted on the particle.
and the impedance, Z, in an R – L – C circuit. A. 0.050N
The current in the circuit will be maximum B. 0.075N
when C. 0.300N
A. XL < Xc D. 3.300N
B. XL = Xc E. 0.000N
C. Xc <XL
D. R = XL 42. The function of the manganese (IV) oxide in
E. R = Xc a teclanche cell is to
A. decrease the e.m.f of the cell
37. In a purely resistive a.c circuit, the current, I B. prevent local action in the cell
= Io sinwt and the voltage, V = Vo sinwt. C. prevent polarization in the cell
Calculate the instantaneous power D. increase the density of the electrolyte
dissipated in the circuit in time t. E. increase the resistance of the cell
A. IoVo
D. IR2t
E. V2Rt

43. 49. Electric motor primarily converts


A. electrical energy into chemical energy
B. electrical energy into heat energy
C. kinetic energy into potential energy
A. D. Electrical energy into mechanical energy
B. E. Mechanical energy into light energy
C.
D. 50. What amount of current would pass through
E. a 10Ω coil if it takes 21s for the coil to just
melt a lump of ice of mass 10g at 0O if
44. Electrical resistance is the property of an there are no heat losses?
electrical conductor that causes electrical A. 16.00A
energy to be converted into B. 4.00A
A. mechanical energy C. 0.50A
B. solar energy D. 0.25A
C. heat energy E. 0.06A
D. magnetic energy
E. chemical energy 52. 24
Na → 24
Mg+ + X + Energy
11 12
45. Calculate the current, 1 in the diagram
shown below. (Neglect the internal The equation above represents a nuclear
resistance of the cell) decay of sodium isotope. What is X
A. 4.0A A. Alpha particle
B. 2.0A B. Beta particle
C. 1.0A C. Gamma ray
D. 0.5A D. X-ray
E. 0.1A E. Neutron

46. Which of the following is not a conductor of 53. If Einstein’s photoelectric equation is
electricity? represented by hf = W = O where h is
A. Human Plank’s constant f, the frequency of incident
B. Glass radiation and W the work function of the
C. Silver material, what does the symbol O stand for?
D. Earth A. Energy of the photons
E. Copper B. Maximum kinetic energy of the ejected
electrons
47. In the diagram below, the galvanometer C. Threshold frequency of the photons
indicates a null-deflection. What is the D. Stopping potential of the electrons
potential difference between P and Q? E. Velocity of the electrons
A. o.0V
B. 1.5V 54. The phenomenon of radioactivity was first
C. 2.0V discovered by
D. 3.0V A. Marie Curie
E. 4.0V B. Sir J. J. Thompson
C. Henri Becquerel
48. When a potential difference, V is applied D. Niels Bohr
across the ends of a resistor of resistance, E. Enrico Fermi
R, a current, I passes through the resistor.
The heat generated in the resistor in time, T 55. A radioactive substance has a half-life of 20
is given by the expression hours. What fraction of the original
A. V2It radioactive nuclide will remain after 80
B. I2t hours
C. I2Rt A. 1_
32
B. Alpha particles
B. 1_ C. Beta particles
16 D. Protons
E. Neutrons
C. 1_
8

D. 1_ 1993
4 PART B THEORY

E. 1_ 1. (a) (i) Explain what is meant by a


2 machine.
(ii) Define the terms: mechanical
56. Which of the following is used in a nuclear advantage, velocity ratio and
reactor to slow down fast moving neutrons? efficiency as applied to a
A. Carbondioxide gas machine. Derive equation
B. Liquid sodium metal connecting the three terms.
C. Concrete shield
D. Graphite rods (b) Explain why the efficiency of a
E. Boron rods machine is usually less than 100%

57. According to the kinetic theory of gases, the (c) A screw jack whose pitch is 4.4 mm
collision of the molecules of a gas with the is used to raise a body of mass
walls of the container is mainly responsible 8000kg through a height of 20cm.
for the The length of the tommy bar of the
I. temperature of the gas jack is 70cm. If the efficiency of the
II. viscosity of the gas jack is 80%, calculate the:
III. pressure of the gas (i) velocity ratio of the jack;
Which of the statement above is/are (ii) mechanical advantage of the
correct? jack;
A. I only (iii) effort required in raising the
B. II only body;
C. III only (iv) work done by the effort in
D. II and III only raising the body.
E. I, II and III only (G. = 10ms-1, 2 π = 22/7)

58. Eight α - particles and size β - particles are 2. (a) With the aid of labeled diagram,
emitted from an atom of 238 describe
A. 206 an experiment to illustrate the
B. 234 relationship between the volume and
C. 238 the temperature of a given mass of
D. 240 air at constant pressure.
E. 252
(b) A uniform capillary tube of negligible
59. When an atom is in the ground state, it is expansivity, sealed at one end,
said to be contains air trapped by a pellet of
A. grounded mercury. The trapped air column is
B. excited 13.7cm long at 0oC and 18.7cm long
C. stable at 100oC, calculate the cubical
D. ionized expansivity of the air at constant
E. rediating pressure.
60. Which of the following are emitted from a
radioactive substance without a latering (c) Using the kinetic theory of gases,
either the nucleon number or the proton explain why the volume of a fixed
number of the substance? mass of gas at constant pressure
A. Gamma rays
increases with increase in vertical oscillation and determine 1
temperature. for 20 oscillations.

3. (a) State the laws of electromagnetic Calclulate the period T of oscillation.


induction Evaluate T2. Repeat the experiment
(b) Draw a labeled diagram of a simple for values of M = 100, 150, 200 and
d.c generator and explain how it 250g. Determine the corresponding
works. values of t, T1 and T2 in each case.
(c) State three methods by which high Tabulate your readings.
e.m.f could be obtained from the
generator. Plot a graph of T2 against M starting
both axes from the origin (0, 0).
Determine the slope s and the
4. (a) Explain the following, illustrating intercept I of the graph. State two
your precautions you took to obtain
answer with one example in each accurate results.
case:
(i) nuclear fusion (ii) nuclear (b) (i) Define the moment of a force
fission about a point
(iii) radiation hazards. (ii) If R is the reaction from the
bench, m is the mass of the
(b) State two advantages of fusion over meter rule and M the mass of
and explain briefly why, in spite of the attached load, draw a
these advantages, fusion is not force diagram of the
normally used for the generation of arrangement when the rule is
power. in equilibrium and write down
and equation relating R, m
(c) The current, I in an a.c circuit is and M together (Take g as
given by the equation: I = 30 sin 100 the acceleration due to
π where t is the time in seconds. gravity).
Deduce the following from this (iii) Explain what is meant by
equation: stating that a body is in table
(i) frequency of the current; equilibrium.
(ii) peak value of the current;
(iii) r.m.s value of the current. 2. (a) Use the measuring cylinder provided
to
measure 100ml of water and pour it
1993 into the tin labeled A supplied. Heat
PAPER I PRACTICAL the water in the tin on a cork or
wooden stand. Insert the
thermometer into the tin and record
1. (a) the temperature of the water every
0 minute until the temperature falls to
60oC. Repeat the experiment with tin
labeled B using exactly the same
volume of water and recording the
temperature every minute until the
Clamp the uniform meter rule temperature falls to 60oC. Tabulate
supplied to the edge of the bench your readings.
such that 90cm of the rule projects
from the edge and the rule is On the same piece of graph paper
capable of performing vertical and using axis and scales, plot two
oscillation as shown above. With M = graphs of temperature against time
50g fixed to the free end of the rule, from the readings obtained using tins
deflect the rule so that it performs A and B. Label your graphs
appropriately as A and B to
correspond with the tin used. From (b) (i) Give that R, L, S and K are
each graph, read off the time taken related by the equation where
to cool from 80oC to 65oC. State two K and S are constants
precautions you took to obtain I = I + I
accurate results. R KL S deduce from
graph
(b) (i) Explain how heat losses by the values of k and S,.
radiation and convection are
minimized in a vacuum flask. (ii) In the circuit diagram shown
in
(ii) State the factors on which the 3(a) above, b represents the
quantity of heat in a body point of contact of the jockey
depends. Write down an with the bridge wire. What is
equation relating the quantity the potential difference
of the heat to the factors. between point a and point b
at the balance point? Explain
(iii) State four factors on which your answer briefly.
the
rate of evaporation of a liquid (iv) Deduce from your graph the
in an open container effective resistance R if a
depends. length of Im of the constant
3. (a) wire is connected across the
resistance S.

1994
Connect the 2Ω resistor and the SSCE PHYSICS
object labeled s across the gaps AB PAPER 2 OBJECTIVE
and CD respectively in the meter
bridge circuit as shown in the 1. Which of the following is scalar quantity?
diagram above. Also connect one A. Tension
end of the constant wire labeled W to B. Weight
point C in the circuit. By means of a C. Impulse
crocodile clip, connect a length L = D. Upthrust
20cm of the wire across s. Complete E. Distance
the circuit and close the key K.
Determine the balance length X 2. The diagram below represents the velocity
measured along the meter bridge time graph of a body-in motion. the total
wire. distance traveled by the body is 195m.
Calculate the acceleration of the body in
Evaluate R where R = (200 – 2) section OP of the graph
x A. 1.0ms-2
B. 2.5ms-2
Determine the values of R-1 and L-1. C. 7.8ms-2
Repeat the experiment for L = 30, 40 D. 9.3ms-2
50 and 60cm. In each case, E. 19.5ms-2
determine the corresponding values
of X, R, R-1 and R-1. Tabulate your 3. The density of water is 1gcm-3 while that of
readings. Plot a graph of R-1 against ice is 0.9-3. Calculate the change in volume
L-1 starting both axes from the origin when 90g of ice is completely melted.
(0, 0). Determine the slope of the A. 0cm3
graph and its intercept along the B. 9cm3
vertical axes. State two precautions C. 10cm3
you took to obtain accurate results. D. 90cm3
E. 100cm3
B. 35.0˚C
4. A body accelerates uniformly from rest at C. 37.3˚C
2ms-2. Calculate its velocity after traveling D. 52.7˚C
9m. E. 62.7˚C
A. 36.00ms-1
B. 18.00ms-1 10. A metal rod of length 40.00cm at 20˚C is
C. 6.00ms-1 heated to a temperature of 45˚C. If the new
D. 4.5ms-1 length of the rod is 40.05cm, calculate its
E. 4.24ms-1 linear expensivity.
A. 1.2 x 10-3 K-1
5. In which of the following simple machines is B. 2.5 x 10-5 K-1
the effort applied between the load and the C. 3.5 x 10-5 K-1
fulcrum? D. 4.5 x 10-5 K-1
A. Scissors E. 5.0 x 10-4 K-1
B. Pliers
C. Wheel barrow 11. A gas occupies a certain volume at 27˚C. At
D. Sugar tongs what temperature will its volume be three
E. Nutcrackers times the original volume assuming that its
pressure remains constant?
6. A ball is dropped from a height of 45m A. 81˚C
above the ground. Calculate the velocity of B. 162˚C
the ball just before it strikes the ground C. 354˚C
(Neglect air resistance and take g as 10ms- D. 627˚C
2
) E. 900˚C
A. 21.2ms-1
B. 30.0ms-1 12. 500cm3 of a gas is collected at 0˚C and at a
C. 300.0ms-1 pressure of 72.0cm of mercury. What is the
D. 450.0ms-1 volume of the gas at the same temperature
E. 900.0ms-1 and a pressure of 76.0cm of mercury?

7. What is the moment of the force F about A. 76 x 500 cm3


point P in the diagram shown beside? 72
A. Fy cos θ
B. Fy B. 72 x 500 cm3
C. Fy tan θ 76
D. Fy sin θ
E. Fy coses θ C. 72 x 76 cm3
76 x 500
8. The density of a body is 5 x 103kg m-3 and it
weighs 1.0 N in air. Calculate the apparent D. 76c__ cm3
weight of the body when totally immersed in 72 x 500
water. (Density of water = 103gm-3, g =
I0ms-2) E. 72__ cm3
A. 0.02N 76 x 500
B. 0.08N
C. 0.20N 13. The pressure of air in a tyre is 22.5Nm-2 at
D. 0.80N 27˚C. If the air in the tyre heats up to 47˚C,
E. 1.00N calculate the new pressure of the air,
assuming that no air leaks out and that the
9. The ice and steam points on a mercury in change in volume of the air can be
glass, thermometer are found to be 90.0mm neglected.
apart. What temperature is recorded in A. 21.I0N m-2
degree. Celsius when the length of the B. 24.00N m-2
mercury thread is 33.6mm above the ice C. 39.07N m-2
point mark? D. 45.00N m-2
A. 33.6˚C E. 97.25N m-2
minutes. Calculate the heat supplied per
14. When two objects P and Q are supplied with minute. (Specific heat capacity of water =
the same quantity of heat, the temperature 4200J kg-1 K-1)
change in P is observed to be twice that in A. 9000J
Q. If the masses of P and Q are the same. B. 12600J
Calculate the ratio of the specific heat C. 21000J
capacities of P to Q. D. 25200J
A. 1:1 E. 88200J
B. 1:3
C. 1:4 20. A hygrometer is an instrument used to
D. 1:2 measure
E. 4:1 A. dew point
B. temperature
15. The instrument used for measuring C. relative density
atmospheric pressure is D. relative humidity
A. hydrometer E. vapour pressure
B. ratemeter
C. barometer 21. An electric kettle rated at 1500W boils away
D. photometer 0.3kg of a liquid at its boiling point in 300s.
E. manometer Calculate the specific latent heat of
vaporization of the liquid.
16. Calculate the heat required to convert 20g A. 1.5 x 106J kg-1
of ice at 0˚C to water at 16˚C (Specific B. 1.35 x 106 J kg-1
latent heat of fusion of ice = 336J g-1) C. 1.50 x 105 J kg-1
(Specific heat capacity of water = 4,2Jg-1k) D. 4.50 x 102 J kg-1
A. 1.34 x 103J E. 3.00 x 102 J kg-1
B. 5.38 x 103J
C. 6.70 x 103J 22. 80% of the heat supplied to a 30g block of
D. 7.06 x 193J ice at 0˚C completely melts it to water at
E. 8.06 x 103J 0˚C. Calculate the total heat energy
supplied. (Specific latent heat of ice = 336 J
17. A tap supplies water at 30˚C while another g-1)
supplies water at 86˚C. If a man wishes to A. 26880J
bath with water at 44˚C, calculate the ratio B. 12600J
of the mass of hot water to that of cold C. 10080J
water required. D. 8064J
A. 1:3 E. 4200J
B. 3:1
C. 3:7 23. Calculate the angle of reflection of the ray
D. 7:3 of light at mirror m, in diagram shown
E. 15 : 8 below.
A. 120˚
18. A steam trap is a component of the B. 90˚
apparatus used in determining the specific C. 60˚
latent heat of vaporization of steam. In the D. 45˚
steady state, the steam trap E. 30˚
A. stores the steam for future use
B. prevents the steam from escaping 24. The image of an object placed at the centre
C. ensures that only dry steam gets into the of curvature of a concave mirror is
calorimeter A. inverted and magnified
D. allows condensed steam to go into the B. at the principal focus
calorimeter C. real and diminished
E. determines the quantity of steam used D. erect and virtual
E. at the centre of curvature
19. 500g of water is heated so that its
temperature rises from 30˚C to 72˚C in 7
25. The diagram below shows a ray of light IK
incident on a plane mirror at K. Calculate
the angle of deviation of the ray after
reflection. D. E.
A. 35˚
B. 55˚
C. 70˚ 30. The inability of the eye to focus near objects
D. 105˚ is known as
E. 145˚ A. long sight
B. astimatism
26. In the diagram below F and C respectively C. presbyopia
represent the focal point and the optical D. glaucoma
centre of the lens, what will be the E. short sight
characteristics of the image formed when
rays from the object OA are re-fracted 31. A light wave travels from air into a medium
through the lens? of refractive index 1.54. If the wavelength of
the light in air is 5.9 x 10-7m, calculate its
wavelength in the medium.
A. 2.5 x 10-7m
B. 3.8 x 10-7m
C. 4.6 x 10-7m
D. 5.2 x 10-7m
E. 6.1 x 10-7m
A. magnified, virtual and inverted
B. real, inverted and magnified
C. diminished, virtual and inverted 32. Which of the following are transverse
D. erect, real and diminished waves?
E. diminished, virtual and erect I. Ripples on water
II. Sound waves in air
27. If the critical angle of a glass-air boundary is III. Light waves from the sun
c and the refractive index of the glass is n, A. II only
which of the following relationships is B. I and II only
correct? C. II and III only
A. n = 90/sin c D. I and III only
B. n = sin c/90 E. I, II and III
C. sin 90 sin c = n
D. sin c = I/n 33. A note of frequency 2000Hz has a velocity
E. n = sin c_ of 400ms-1. What is the wavelength of the
sin 45 note?
28. A simple magnifying glass is used to view A. 800.0m
an object. At what distance from the lens B. 200.0m
must the object be placed so that an image C. 5.0m
5 times the size of the object is produced D. 2.0m
20cm from the lens? E. 0.2m
A. 2cm
B. 4cm 34. Which of the following has the shortest
C. 15cm wavelength?
D. 25cm A. infrared ray
E. 100cm B. Gamma ray
C. Ultraviolet ray
29. Which of the following diagrams correctly D. Radio wave
illustrates refraction and partial reflection of E. Visible light
light traveling from glass to air?
35. A cell of e.m.f 1.5V and internal resistance
A. B. C. 2.5Ω is connected in series with an ammeter
of resistance 0.5Ω and a resistor of
resistance 7.0Ω. Calculate the current in the A. Niels Bohr
circuit. B. Ernest Rutherfold
A. 0.15A C. Werner Heinsenberg
B. 0.20A D. Louis de Broglie
C. 2.10A E. Erwin Schroedinger
D. 3.00A
E. 6.60A 41. A battery of e.m.f. 10V and internal
resistance 2Ω is connected to a resistance
36. The electrical energy supplied by a of 6Ω. Calculate the p.d across the
Leclanche cell is obtained from terminals.
A. nuclear energy A. 8.75 V
B. magnetic energy B. 7.50 V
C. solar energy C. 5.00 V
D. chemical energy D. 2.50 V
E. mechanical energy E. 1.25 V

37. A battery of e.m.f E and negligible internal 42. A rectangular coil of wire can rotate in a
resistance supplies a current 1 to the magnetic field. The two ends of the coil are
combination of two resistances R1 and R2 soldered respectively to the two halves of a
as shown in the diagram below. Calculate commutator. Two carbon brushes are made
the current through R2 to press lightly against the commutator and
when these are connected in circuit with a
A. R11_ battery and rheostat, the coil rotates. This is
R1 a description of
A. a moving-coil ammeter
B. R21_ B. a.d.c. generator
R1 C. ad.c electric motor
D. an induction coil
C. R21__ E. an a.c. generator
R1+R2
43. A car fuse is marked 15A and operates
D. R11__ normally on a 12V battery. Calculate the
R1 + R2 resistance of the fuse wire.
A. 180.0Ω
E. R1R21_ B. 27.0Ω
R1 – R2 C. 3.0Ω
D. 1.3Ω
38. Which of the following is not part of an a.c E. 0.8Ω
generator?
A. Carbon brushes 44. A particle of charge q and mass m moving
B. slip rings with a velocity v enters a uniform magnetic
C. Commutator field of strength B in the direction of the
D. Field magnet field. The force on the particle is
E. Armature A. qvB
B. mqvB
39. Calculate the effective resistance between C. qvb/m
points X and Y in the diagram beside. D. mvB/q
A. 0.5Ω E. 0
B. 2.0Ω
C. 3.0Ω 45. A lamp marked 100W, 250V is lit for 10
D. 8.0Ω hours. If it operates normally and I kWh of
E. 13.8Ω electrical energy cost 2k. What is the cost of
lighting the lamp?
40. Which of the following scientists postulated A. 1k
that moving particles exhibit wave B. 2k
properties? C. 5k
D. 10k
E. 20k 51. A transformer with 5500 turns in its primary
winding is used between a 240V a.c supply
46. What is the total capacitance in the circuit and 120V kettle. Calculate the number of
represented by the diagram beside? turns in the secondary winding.
A. C1 + C2 + C3 A. 11999
B. 2750
B. 1+1+1 C. 460
C1 C2 C3 D. 550
E. 232
C. C1 C2 C3__
C1 + C2 + C3 52. The electric force between two point
charges each of magnitude q and at a
D. C2_____ distance r apart in air of permitivityo is
C1 + C3
A. q2__
E. C1 C2 C3 4 π ∈ or2

47. Calculate the time taken to heat 2kg of B. q__


water from 50˚C to 100˚C in an electric 4 π ∈ or
kettle taking 5A from a 210 V supply.
(Specific heat capacity of water = 4200kg- C. 4 π q__
1 -1
k ∈0
A. 2.5 minutes
B. 5.7 minutes D. qr2__
C. 6.7 minutes ∈o
D. 7.5 minutes
E. 9.5 minutes E. q2__
4 π ∈ 0r
48. Which of the following diagrams represents
the magnetic fields pattern between two 53. Beta-particles are
isolated unlike poles? A. electrons
B. protons
A. B. C. C. neutrons
D. helium nuclei
E. tritium nuceli
D. E. 54. Calculate the peak voltage of a mains
supply of r.m.s. value of 220V
A. 112V
B. 150V
49. Calculate the current I in the diagram below. C. 222V
(Neglect the internal resistance of the cell) D. 240V
A. I.0A E. 311V
B. 2.0A
C. 2.5A 55. Which of the following substances is not an
D. 4.0A insulator?
E. 5.0A A. Aluminium
B. Glass
50. Which of the following is a stringed C. Plastic
instrument? D. Clay
A. Flute E. Cork
B. Trumpet
C. Piano 56. In an a.c. circuit the peak value of the
D. Drum potential difference is 180V. What is the
E. Organ
instantaneous potential difference when the (b) (i) Describe an experiment to
phase angle is 40o? determine the specific heat
A. 45 V capacity of copper using a
B. 90 V copper ball.
C. 90 2 V
(ii) State two precautions
D. 180 V
necessary to obtain accurate
E. 180 2 V results.

57. When atom 0 losses or gains a charge, it (iii) A piece of copper ball of mass
becomes 20g at 200oC is placed in a
A. an Ions copper calorimeter of mass
B. an electron 60g containing 50g of water
C. a neutron at 30oC. Ignoring heat losses,
D. a proton calculate the final steady
E. a deuteron temperature of the mixture.

58. In 24 days a radioactive isotope decreases (Specific heat capacity of water –


in mass from 64g to 2g. What is the half-life 4.2Jg-1K-1)
of the radioactive material? (Specific heat capacity of copper –
A. 0.75 days 0.4Jg-1K-1)
B. 2.58 days
C. 4.00 days 2. (a) Describe an experiment to show how
D. 4.80 days the
E. 6.00 days frequency of the note emitted by a
vibrating string depends on the
59. The nucleon number and the proton number tension in the string.
of a neutral atom of an element are 238 and
92 respectively. What is the number of (b) Draw diagrams showing a vibrating
neutrons in the atom? string
A. 330 fixed at both ends emitting.
B. 238 (i) fundamental frequency
C. 146 (ii) second overtone. Indicate the
D. 119 nodes and antinodes on the
E. 73 diagrams.

60. The work function of a metal 8.6 x 10-19J. (c) With the aid of a ray diagram, show
Calculate the wavelength of its threshold how a virtual image of an object is
frequency. (Speed of light in vacuum = 3 x formed by a:
108ms-1. Plank’s constant = 6.6 x 10-34Js) (i) concave mirror
A. 0.8 x 10-7m (ii) converging lens.
B. 1.0 x 10-7m
C. 2.3 x 10-7m 3. (a) Explain what is meant by a magnetic
D. 3.8 x 10-7m field.
E. 12.4 x 10-7m
(b) Describe an experiment to show that
magnetic field exists around a
straight wire carrying current.

1994 (iii) Draw a labeled diagram showing the


SECTION B THEORY pattern and directions of the
magnetic field produced around the
1. (a) What is meant by the statement: The wire. (Neglect the earth’s magnetic
specific heat capacity of copper is field)
400 J kg-1 K-1?
(c) Sketch the magnetic field due to two results. Determine the slope s of the
straight parallel wire carrying current graph. Evaluate m=50s. State two
in the same direction. Indicate the precautions taken to ensure accurate
neutral point in the field. results.

(d) Explain, with the aid labeled (b) (i) A uniform meter rule AB is
diagram, how a delicate magnetic balanced on a knife edge
material could be protected from the which is 55cm from B. If a
earth’s magnetic field. mass of 10g is hung at p,
which is 10cm from A.
4. (a) State the laws of electromagnetic Calculate the mass of the
induction metre rule.

(b) Explain how one of the laws (ii) Explain what is meant by the
illustrates the principle of moment of a force about a
conservation of energy. point.

(c) (i) Draw a labeled diagram of a (iii) By means of simple diagrams


simple d.c. electric motor and only, distinguish between two
explain how it works. classes of levers and give one
example of each.
(ii) State two reasons why the
efficiency of an electric motor 2. (a)
is less than 100%.

1994
PAPER I PRACTICAL
Trace the outline ABC of the
1. (a) equilateral
triangular glass prism as shown in
the diagram above. Draw a line RN
such that it makes an angle I = 5o
You are provided with a uniform with the normal. Erect two pins at P1
meter rule, a knife edge and a mass and P2 on the line RN. Replace the
of 50g. Balance the meter rule on prism. Looking through the face BC
the knife edge and record the scale of the prism, fix one pin at P2 and
mark C. Corresponding to the another pin at P4 such that they
distance of its centre of gravity from appear to be in a straight line with
the end of the rule. Suspend the the images at P1 and P2. Remove the
given mass from the meter rule such prism and the pins. Draw a line to
that y = 2cm as shown on the join P4 and P3. Produce the line P4 P3
diagram above. Balance the meter and that of RN to meet at D. Measure
rule with the hanging mass on the and record angles e and a.
knife edge. Read and record the
position of the pivot. Determine the Repeat the experiment for I = 10o,
values of d and x. Repeat the 15o, 20o and 25o respectively using a
experiment with Y = 4, 6, 8, 10 and different outline in each case.
12cm respectively. In each case Tabulate your readings. Plot a graph
record the position of the pivot and with a on the vertical axis and (I + e)
determine the corresponding values on the horizontal axis, starting both
of d and x. Tabulate your readings. axes from the origin (0,0). Determine
Plot a graph of d against x. the slope of the graph and the
Determine the slope s of the graph. intercept on the vertical axis. State
Evaluate m = 50s. State two two precautions taken to ensure
precautions taken to ensure accurate accurate results.
(Attach your traces to your answer 1995
booklet) SSCE PHYSICS
PAPER 2 OBJECTIVE
(b) (i) What is meant by the critical
angle of a medium? 1. Which of the following is a fundamental
(ii) Show by a ray diagram how a quantity?
right-angle glass prism A. Heat capacity
(iii) Distinguish between B. Electric current
dispersion and refraction of a C. Torque
beam of white light as it D. Reactance
passes through a triangular E. Density
glass prism.
2. A boy cycles continuously through a
3. (a) distance of 1.0km in 5 minutes. Calculate
his average speed.
A. 0.3ms-1
B. 3.3ms-1
C. 16.6ms-1
Connect the circuit as shown in the D. 20.0ms-1
diagram above. Close the key, Read E. 200.0ms-1
and record the ammeter reading, Io,
when the crocodile clip is not in 3. A student found out from a simple
contact with the constantan wire. pendulum experiment that 20 oscillations
Open the key. With the clip making were completed in 38 seconds. What is the
contact with the constantan wire L = period of oscillation of the pendulum?
80cm, close the key. Read and A. 8.0s
record the ammeter reading L. B. 3.8s
C. 2.0s
Repeat the procedure when L = 70, D. 1.9s
60, 50 and 40cm respectively. In E. 0.5s
each case read and record the
ammeter reading I, Evaluate L-1. 4. A body of mass 7.5kg is to be pulled up
Tabulate your readings. Plot a graph along a plane which is inclined at 30o to the
with I on the vertical axis and L-1. horizontal. If the efficiency of the plane is
Tabulate your readings. Plot a graph 75% what is the minimum force required to
with I on the vertical axis and L-1I on pull the body up the plane?
the horizontal axis. Starting both A. 5.0N
axes from the origin (0, 0). Calculate B. 20.0N
the scope s of the graph and the C. 50.0N
intercept K on the vertical axis. D. 75.0N
Evaluate p = k/s. State two E. 200.0N
precautions taken to ensure accurate
results. 5. The angular speed of an object describing a
circle of radius 4m with a linear constant
(b) (i) Explain what is meant by the speed of
e.m.f of a cell. 10ms-1 is
(ii) State two effects of electric A. 40.00rads-1
current and one application of B. 14.00rads-1
each. C. 2.50rads-1
(ii) The resistivity of a given wire D. 1.58rads-1
of cross sectional area E. 40rads-1
0.7mm2 is 4.9 x 10-4Ωmm.
Calculate the resistance of a 6. At what angle to the horizontal must the
2m length of the wire. nozzle of a machine gun be kept when firing
to obtain a maximum horizontal range for
the bullets?
A. 0.0o 12. A brass rod is 2m long at a certain
B. 22.5o temperature. Calculate the linear expansion
C. 30.0o of the rod for a temperature change of
D. 45.0o 100K. (Take the linear expansitivity of brass
E. 90.0o at 1.8 x 10-5 K-1
A. 0.3600m
7. Which of the following will reduce the B. 0.1800m
frequency of oscillation of a simple C. 0.360m
pendulum? D. 0.0036m
A. Increasing the mass of the bob E. 0.0018m
B. Decreasing the mass of the bob
C. Increasing the length of the string 13. A body of specific heat capacity 450J kg-1 K-1
D. Decreasing the length of the string falls to the ground from rest through a
E. Decreasing the amplitude of the oscillation vertical height of 20m. Assuming
conservation of energy, calculate the
8. A car traveling at a uniform speed of 120km change in temperature of the body striking
h-1 passes two stations in 4 minutes. the ground level. (g = 10ms-1)
Calculate the distance between the two A. 0oC
stations. B. 2/9oC
A. 8km C. 4/9˚C
B. 15km D. 9/4˚C
C. 22km E. 9/2˚C
D. 30km
E. 60km 14. Hot water at temperature of t is added to
ice that amount of water at a temperature
9. A machine has an efficiency of 60%. If the of 30˚C. If the resulting temperature of the
machine is required to overcome a load of mixture is 50˚C, calculate t.
30N with a force of 20N, calculate its A. 90˚C
mechanical advantage? B. 80˚C
A. 0.7 C. 50˚C
B. 0.9 D. 40˚C
C. 1.5 E. 30˚C
D. 2.5
E. 10.0 15. An air bubble of volume 2cm3 is formed 20m
under water. What will be its volume when it
10. A body weighing 10N in air is partially rises to just below the surface of the water if
immersed in water. It displaces water of the atmospheric pressure is equivalent to a
mass 0.3kg. What is the puthrust on the height of 10m of water?
body? (g = 10ms-2) A. 6cm3
A. 13.0N B. 4cm3
B. 7.0N C. 3cm3
C. 3.3N D. 2cm3
D. 3.0N E. 1cm3
E. 0.3N
16. The pressure of a given mass of gas
11. A mercury-in-glass thermometer reads n – changes from 200Nm-2 to 100Nm-2, while its
20o at the ice point and 100o at the steam temperature drops from 127˚C to -73˚C.
point. Calculate the Celsius temperature Calculate the ratio of the final volume of the
corresponding to 70o on the thermometer gas to its initial volume
A. 41.0oC A. 2.4:1
B. 50.0oC B. 2.0:1
C. 62.5oC C. 1.2:1
D. 70.0oC D. 1.0:1
E. 75.0oC E. 1.0:2
17. The temperature of glass vessel containing E. 30˚
100cm3 of mercury is raised from 10˚C to
100˚C. Calculate the apparent cubic
expansion of the mercury. (Real cubic 22. A ray of light is incident on a body x as
expansivity of mercury = 1.82 x 10-4K-1) shown in the diagram below. What is the
A. 0.52cm3 refractive index of the body?
B. 1.42cm3 A. 1.63
C. 1.87cm3 B. 1.50
D. 1.87cm3 C. 1.49
E. 5.22cm3 D. 1.33
E. 1.02
18. An iron rod of mass 2kg and at a
temperature of 280˚C is dropped into some 23. Light travels in straight lines. In which of the
quantity of water initially at a temperature following is this principle manifested?
of 30˚C. If the temperature of the mixture is I. Pinhole camera
70˚C, calculate the mass of the water. II. Formation of shadows
(Neglect heat looses to the surroundings) III. Diffraction of light
(Specific heat capacity of iron = 460 J kg-1 K- IV. Occurrence of eclipse
1
) (Specific heat capacity of water = 4200J A. I and III only
kg-1 K-1) B. II and III only
A. 0.58kg C. I, II and III only
B. 0.77kg D. I, II and IV only
C. 1.15kg E. I, II, III and IV only
D. 1.50kg
E. 2.04kg 24. the image of any real object formed by a
diverging lens is always
19. How much heat is required to convert 20g A. inverted
of ice at 0˚C to water at the same B. magnified
temperature? [Specific latent heat of ice = C. erect
336 J g-1] D. real
A. 1.35 x 103J E. the same size as the object
B. 5.38 x 103J
C. 6.72 x 103J 25. Which of the following is not self-luminous?
D. 7.06 x 103J A. Incandescent electric bulb
E. 8.06 x 103J B. Incandescent fluorescent tube
C. Lighted candle
20. When two objects P and Q are supplied with D. The moon
the same quantity of heat, the temperature E. The sun
change in P is observed to be twice that in
Q. If the masses of P and Q are the same, 26. An object is placed on the principal axis and
calculate the ratio of the specific heat at the centre of curvature of a concave
capacities of Q to P. mirror. The image of the object formed by
A. 4:1 the mirror is
B. 2:1 A. real, inverted and magnified
C. 1:1 B. at the principal focus
D. 1:2 C. real and diminished
E. 1:4 D. erect and virtual
E. at the centre of curvature
21. A ray of light is incident on mirror m 1 and
after reflection is incident on mirror m2 as 27. A wire is stretched between two points, 1m
shown in the diagram below. Calculate the apart, if the speed of the wave generated
angle of reflection of the ray at mirror m2. on plucking the wire is 200ms-1, what is the
A. 120˚ minimum frequency which will resonate
B. 90˚ with the wire.
C. 60˚ A. 50 Hz
D. 45˚ B. 75Hz
C. 100Hz A. 0.02s
D. 150Hz B. 0.05s
E. 200Hz C. 0.20s
D. 0.50s
28. Sixty complete waves pass a particular E. 50.00s
point in 4s. If the distance between three
successive trouhghs of the waves is 15m, 34. A listener can detect the instrument from
calculate the speed of the waves. which a noise is being sounded because
A. 300ms-1 different instruments produce the same
B. 225.0ms-1 note with different
C. 112.5ms-1 A. Frequencies
D. 75.0ms-1 B. pitches
E. 16.0ms-1 C. loudness
D. intensities
29. Which of the following media allow(s) the E. overtones
transmission of sound wave through them?
I. Air 35. What is the escape velocity of a body on the
II. Liquid surface of the earth of radius R if the
III. Solid gravitational constant is G and the mass of
A. I only the earth is M? (Neglect energy losses to
B. II only the surroundings)
C. III only ___
D. I and II only A. √2GR
E. I, II and III only
___
30. Which of the wave characteristics can be B. √2G
used to distinguish a transverse wave from R
a longitudinal wave? ____
A. Reflection C. √2GR2
B. Refraction
C. Diffraction ___
D. Poliarisation D. √2GM
E. Interference R
____
31. A sound note of frequency 250Hz and E. √2GM
wavelength 1.3m is produced at a point
near a hill. If the echo of the sound is 36. A charge of 1.6 x 10-10C is placed in a
received 1 second later at the point, how far uniform electric field of intensity 2.0 x 102
away is the hill from the point? NC-1. What is the magnitude of the electric
A. 131.3m force exerted on the charge?
B. 162.5m A. 3.2 x 105M
C. 248.7m B. 1.8 x 105N
D. 251.3m C. 3.2 x 10-5N
E. 325.0m D. 1.8 x 10-1N
E. 4.0 x 10-6N
32. The period of a wave is 0.02 second.
Calculate its wavelength if its speed is 37. A balloon containing 546cm3 of air is heated
330ms-1 from 0oC to 10oC. If the pressure is kept
A. 6.6m constant, what will be its volume at 10oC.
B. 5.0m A. 526cm3
C. 4.m B. 546cm3
D. 3.n C. 556cm3
E. 2.m D. 566cm3
E. 819cm3
33. An alternating voltage with a frequency of
50Hz has a period of
38. Calculate the inductance L of the coil in the 43. A capacitor of capacitance 25μF is
circuit shown beside. connected to an a.c power source of
A. 14.4H frequency 200Hz. Calculate
B. 3.8H π
C. 0.6H the reactance of the capacitor.
D. 0.4H A. 0.01Ω
E. 0.2H B. 0.02Ω
C. 50.00Ω
39. A series R-L-C circuit is shown in the D. 100.00Ω
diagram below. Which of the following E. 200.00Ω
vector diagrams correctly represents the 44. The energy stored in a capacitor of
phase relationship among I, V, VR, VC and VL capacitance 5μF is 40J. Calculate the
if VC is greater than VL and the symbols have voltage applied across its terminal?
their usual meaning? A. 4000V
B. 200V
C. 16V
D. 6V
E. 4V

45. The effective capacitance between points X


A. B. C. and Y in the diagram beside is 1.0μF. What
is the value of the capacitance C, measured
in micro-farad?
A. 5.0μ
B. 4.0μ
D. E. C. 3.0μ
D. 1.0μ
E. 0.5μ
40. Calculate the electric potential at a distance
of 20.0cm from a point charge of 0.015C 46. Calculate the quantity of charge flowing
placed in air of permittivity [Take 1_ as 9.0 through a conductor if a current of 10A
x 10oNm2C-2] passes through the conductor for 10s.
4π ∈0 A. 100C
A. 3.40 x 10oV B. 20C
B. 6.75 x 10RV C. 15C
6.75 x 10 λ V
D. 10C
C.
E. 1C
D. 3.45 x 105V
E. 8.30 x 10-12V
47. An alternating current, having the waveform
41. Calculate the inductive reactance of the
shown in the diagram beside, is represented
circuit shown beside.
by the equation X = Xo, sinwt. Which of the
A. 50.00Ω
following represent Xo?
B. 5.00Ω
A. OS
C. 0.50Ω
B. OR
D. 0.05Ω
C. PQ
E. 0.02Ω
D. RT
E. OT
42. If the current in the resistor R is question 41
is 0.05A, calculate the p.d across the
48. What is the potential difference between X
inductor.
and Y in the diagram beside if the battery is
A. 2.5V
of negligible internal resistance?
B. 25.0V
A. 12.5V
C. 49.0V
B. 5.8V
D. 50.0V
C. 5.0V
E. 250.0V
D. 3.0V
E. 2.0V C. 8 days
D. 24 days
49. Calculate the current in the 3 resistor in E. 42 days
question 48.
A. 0.3A 55. A moving-coil meter with an internal
B. 0.4A resistance of 100Ω has a full-scale
C. 1.0A deflection when a current of 10m A flows
D. 1.5A through it. What value of resistance would
E. 2.5A convert it to read 10V at full-scale
deflection?
50. A given wire of resistance 10Ω has a length A. 20Ω
of 5m and a cross-sectional area of 4.0 x 10- B. 100Ω
3
m2. Calculate the conductivity of the wire. C. 600Ω
A. 0.80 x 107 Ω-1m-1 D. 900Ω
B. 1.25 x 107 Ω-1m-1 E. 1000Ω
C. 2.02 x 107 Ω-1m-1
D. 2.50 x 107 Ω-1m-1 56. Which of the following are emitted from a
E. 5.00 x 107 Ω-1m-1 radioactive substance without altering
either the nucleon number or the proton
51. Which of the following is a wind instrument. number of the substance?
A. Guitar A. Protons
B. Organ B. Neutrons
C. Violin C. Gamma rays
D. Piano D. Beta particles
E. Harp E. Alpha particles

52. A cell of e.m.f 1.5V and internal resistance 57. Which of the following is not a product of
2.5Ω and a load of resistance 7.0Ω. nuclear fission?
Calculate the current in the circuit. A. Alpha particle
A. 0.15A B. Beta particle
B. 0.20A C. Gamma ray
C. 2.10A D. X-ray
D. 3.00A E. Neutron
E. 6.67A
58. Calculate the energy carried by an Ex-ray of
53. Which of the following statements are not wavelength 6.0 x 10-40m. [Plank constant =
true of a moving-coil millimeter? 6.6 x 10-34 Js, velocity of light = 3.0 x 108ms-
I. It can be used to measure alternating 1
]
current A. 3.3 x 10-12J
II. It has a linear scale B. 3.3 x 10-16J
III. It can be adapted to read higher values of C. 1.1 x 10-16J
currents D. 3.3 x 10-24J
IV. A resistor connected in parallel with the E. 3.3 x 10-32J
millimeter would convert it to a volt meter
A. I and IV only 59. Which of the following cannot be explained
B. II and III only by the molecular theory of matter?
C. III and IV only A. Conduction
D. I, II and III only B. Radiation
E. I, III and IV only C. Convection
D. Evaporation
54. A radioactive nuclide of mass 6.0g has a E. Expansion
half-life of 8 days. Calculate the time during
which 5.25g of the nuclide would have 60. Which of the following is used for shielding
decayed radioactive fall-outs?
A. 1 day A. Plastic
B. 2 days B. Wood
C. Textile (e) State two methods by which the
D. Aluminium surface tension of a liquid may be
E. Lead reduced.

4. (a) Define the boiling point of a liquid.


1995 (b) Describe with the aid of a labeled
SECTION B THEORY diagram, an experiment to
determine the boiling point of a small
1. (a) Explain what is meant by quantity of a liquid.
acceleration of (c) State two factors that may affect the
free fall due to gravity, g. boiling point of a liquid.
(d) using the kinetic theory of matter,
(b) State two reasons why g varies on explain why pure water changes to
the surface of the earth. steam at S.T.P. without any change
in temperature, although heat is
(c) A stone is projected upwards at an being supplied to the water.
angle of 30o to the horizontal from
the top of a tower of height 100m 1995
and it hits the ground at a point Q. If PAPER I PRACTICAL
the initial velocity of projection is
100ms-1, calculate the 1. (a)
(i) maximum height of the stone
above the ground;
(ii) time it takes to reach this Suspend the given spiral spring
height; vertically as shown in the diagram
(iii) time of flight; above. Attach a scale pan and note
(iv) horizontal distance from the the pointer on the metre rule. Add a
foot of the tower to the point mass m = 70g to the scale pan and
Q (neglect air resistance and note the new position of the pointer.
take g as 10 ms-2) Determine the extension produced.

2. (a) State Faraday’s law of Repeat the experiment for m = 90,


electromagnetic 110, 130 and 150g respectively. In
inductions. each case determine the extension e
(b) Draw a labeled diagram of an produced. Ignore the mass of the
induction coil and explain how it scale pan and tabulate your
works. readings. If a hanger is used, both
(c) How is the effect of eddy currents the mass of the hanger and the
minimized in the coil? added slotted masses should be
(d) State two reasons why a capacitor equal to 70, 90, 110, 130 and 150g.
should be included in the primary
circuit of the coil. Plot a graph with e on the vertical
(e) State three uses of an induction coil. axis and m on the horizontal axis,
starting both axes from the origin
3. (a) What is surface tension? Explain the (0,0). Determine the slope of the
phenomenon in terms of graph and the intercept on the axis.
intermolecular forces. Also determine the difference in the
(b) Describe a simple experiment to extension x when the mass
demonstrate the surface tension of a increased from 100g to 150g. State
liquid. two precautions taken to ensure
(c) State three examples to illustrate the accurate results. With the load of
effects of surface tension. 150g on the scale pan/hanger, set
(d) Why does water wet a clean glass the spring into small vertical
surface whereas mercury does not? oscillation and determine the time
for 10 complete oscillations.
Calculate the period T1 of oscillation.
taken to ensure accurate results.
Repeat the experiment for a load of [Attach your traces to your answer
100g and determine the script].
corresponding period T2 of
oscillation. Evaluate the expression K (i) Using your graph, deduce the
= 39.5x value of L when I = 0o
T12 – T22 (ii) State Snell’s law of refraction
and explain why refraction
(b) (i) State Hooke’s law. occurs at the boundary
(ii) Deduce the force constant of between two media.
the (iii) A microscope is focused on a
spiral spring from the graph. mark on a table. When the
(Take g as 10m-2) mark is covered by a plate of
(iii) A spiral spring is compressed glass 3.00cm thick, the
by 0.03m. Calculate the microscope has to be raised
energy stored in the spring if 1.18cm for the mark to be
its force constant is 300Nm-1. once more in focus. Calculate
2. (a) the refractive index of the
glass.

3. (a)

Measure and record the thickness b


of the glass block provided. Trace
the outline ABCD of the glass on the Connect the circuit as shown above.
sheet of paper as shown above. Open the key K and record the
Remove the block and draw a normal voltmeter reading E. Set the
at N. Draw an incident ray such that resistance R in the resistance box
the angle of incidence i = 25o, Fix equal to 1Ω. Close the key, read and
two pins at points P and Q on the record the potential difference V on
incident ray. Replace the glass block the voltmeter. Evaluate R-1 and V-1.
and fix two other pins at points R and Repeat the experiment for R = 2, 3,
Y such that the pins appear to be in a 4, 5 and 6Ω respectively. In each
straight line with the images of the case, read and record the voltmeter
pins at P and Q when viewed through reading V. Also evaluate R-1 and V-1.
the side DC of the glass block. Tabulate your readings. Plot graph
Remove the block and join the points with V-1 on the vertical axis and R-1
at R and Y, producing the line to on the horizontal axis, starting both
meet DC at X. Join NX and measure axes from the origin (0,0). Determine
its length L. Evaluate L-3 and sin2i. the slopes of the graph and the
intercept I on the vertical axis.
Evaluate the expression: K = s/1.
Repeat the experiment for i = 35o, State two precautions taken to
45o, 55o and 65o. In each case, ensure accurate results.
determine the corresponding values
of L, L-2 and sin2i. Tabulate your (b) (i) State ohm’s law.
readings (ii) If the p.d across the
resistance box in the circuit is
Plot a graph of L-2 on the vertical measured, it would be
axis and sin2i on the horizontal axis, observed that its value is less
starting both axes from the origin (0, than the e.m.f. of the cell.
0). Determine the slopes of the Explain the reason for this
graph and the intercept I on the difference.
vertical axis. Evaluate the expression (iii) Explain what is meant by the
K = (i/s)1/2. State two precautions potential difference between
two points in an electric B. 1.13 x 104Ms-1
circuit. C. 8.00 x 103Ms-1
D. 4.23 x 103Ms-1
E. 6.41 x 102Ms-1
1996
SSCE PHYSICS 6. The velocity ratio and efficiency of a system
PAPER 2 OBJECTIVE of pulleys are 6 and 80% respectively. How
much effort is required to lift a load of mass
1. The diagram below represents a portion of a 120Kg with this system.
micrometer screw guage in which the A. 25N
horizontal scale is in millimeter. What is the B. 90N
reading on the instrument. C. 96N
A. 3.72mm D. 250N
B. 3.50mm E. 960N
C. 3.30mm
D. 3.28mm 7. Two spanners X and Y of lengths 15cm and
E. 3.22mm 20cm respectively are used in turn to give a
screw ff pitch 2mm one complete rotation. If
Rx and Ry are the respective velocity
2. A loaded test-tube which floats upright in rations of the spanners then the ration Rx
water is carefully and slightly depressed and Ry is
then released. Which of the following best A. 1 : 50
describes the subsequent motion of the test B. 3 : 20
tube? C. 3:4
A. Circular D. 4:3
B. Rotational E. 20 : 3
C. Random
D. Oscillatory 8. The magnitude of the force required to
E. Translational make an object of mass M move with speed
V in a circular path of radius R is given by
3. The diagram above illustrates the velocity- the expression
time graph of the motion of a body. A. mr
Calculate the total distance covered by the v
body.
A. 50m B. (mr)2
B. 65m v
C. 175m
D. 225m C. mv2
E. 300m r

4. A ball of mass 5.0kg hits a smooth vertical D. mv


wall normally with a speed of 2ms-1 and r2
rebounds with the same speed. Determine
the impulse experienced by the ball. E. mv
A. 20.0kg ms-1 r
B. 10.0kg ms-1
C. 5.0kg ms-1 9. A body of mass 4.2kg moving with velocity
D. 1.3kg ms-1 10ms-1 due east, hits a stationary body of
E. 0.0kg ms-1 mass 2.8kg. If they stick together after
collision and move with velocity V due east,
5. A rocket is launched from the surface of the calculate the value of V.
earth. If the radius of the earth is 4.6 x 106m A. 3ms-1
and the acceleration of free fall due to B. 6ms-1
gravity is 10ms-1, calculate the escape C. 7ms-1
velocity of the rocket. D. 15ms-1
A. 2.53 x 104Ms-1 E. 17ms-1
15. The pressure of a fixed mass of gas is 2.0 x
10. A body of mass 2kg is suspended from the 105Nm-2 at a known temperature,
ceiling of a lift with a light intensible string. assuming that the temperature remains
If the lift moves upwards with acceleration constant, what will be the pressure of the
of 2ms-2, calculate the magnitude of the gas if its volume is halved?
tension in the string. (g = 10ms-2) A. 1.0 x 105Nm-2
A. 24N B. 2.0 x 105Nm-2
B. 20N C. 3.0 x 105Nm-2
C. 16N D. 4.0 x 105Nm-2
D. 4N E. 5.0 x 105Nm-2
E. 0N
16. Which of the following statements is not
11. A stationary object of mass 4kg is set in correct?
motion by a net force of 50N. If the object A. Heat energy can be transformed into
attains a speed of 5ms-1 in time. Calculate mechanical energy
the value of it. B. That total heat content of a body is the sum
A. 0.20s of the kinetic energies of its molecules
B. 0.40s C. If a body is heat, its molecules move faster
C. 0.63s D. The total heat content of a body is the
D. 0.80s product of its specific heat capacity and its
E. 1.30s mass
E. If a body is cooled, molecular movement
12. Two simple pendula and y make 400 and remains constant
500 oscillations respectively in equal time. If
the period of oscillations of x is 1.5 seconds, 17. A 400W immersion heater is used to heat a
what is the period of oscillation of y? liquid of mass 0.5kg. if the temperature of
A. 0.53s the liquid increases by 2.5oC in one second,
B. 0.83s calculate the specific heat capacity of the
C. 1.20s liquid. (Neglect heat loses to the
D. 1.50s surroundings)
E. 1.88s A. 80 J kg-1K-1
B. 320 J kg-1K-1
13. A platinum - resistance thermometer has a C. 500 J kg-1K-1
resistance of 5Ω at 0o and 9Ω at 100oC. D. 1200 J kg-1K-1
Assuming that resistance changes uniformly E. 2000 J kg-1K-1
with temperature, calculate the resistance
of the thermometer when the temperature 18. What is the difference in the amount of heat
is 45oC. given out by 4kg of steam and 4kg of water
A. 13.9Ω when both are cooled from 100oC to 80oC
B. 8.9Ω A. 336,000J
C. 6.8Ω B. 672,000J
D. 3.2Ω C. 8,704,000J
E. 1.8Ω D. 9,040,000J
E. 9,376,000J
14. The linear expansivity of a metal P is twice
that of another metal Q. When these 19. The heating element in an electric kettle is
material are heated through the same usually located near the bottom of the
temperature change, their increase in kettle because
length is the same. Calculate the ratio of the A. water is a good conductor of heat
original length of P to that of Q. B. heat can be more quickly radiated to all
A. 1:4 parts of the water
B. 1:2 C. no heat can be lost to the surroundings
C. 2:1 D. the convectional currents which are set up
D. 4:1 can carry heat to all parts of the water
E. 6:1 E. the molecules of the heating element can
carry heat to all parts of the water
26. A ray of light strikes a plane mirror at a
20. The inside of a vacuum flask is usually glancing angle of 50o. Calculate the angle
coated with silver to reduce heat lost by between the incident and reflected rays.
A. convection A. 20o
B. conduction B. 40o
C. radiation C. 50o
D. absorption D. 80o
E. evaporation E. 100o

27. A boy walks away from a plane mirror at a


21. The saturated vapour pressure of a liquid speed of 3msp1m in a direction normal to
increases as the the surface of the mirror.
A. volume of the liquid increases A. 9.0ms-1
B. volume of the liquid decreases B. 6.0ms-1
C. temperature of the liquid increases C. 4.5ms-1
D. temperature of the liquid decreases D. 1.5ms-1
E. mass of the liquid decreases E. 0.0ms-1

22. Cloud formation is the direct result of 28. The graph above shows the relationship
A. precipitation between sin i and sin r for a ray of light in
B. vaporisation air incident on a liquid surface. If i and r rare
C. Fusion respectively angles of incidence and
D. sublimation refraction deduce the refractive index of the
E. condensation liquid from the graph.
A. 1.63
23. Water boils at a lower temperature when B. 1.50
heated at the top of a mountain than at sea- C. 1.33
level because at the top of the mountain the D. 1.25
A. saturation vapour pressure of water is E. 0.80
higher than at sea-level
B. sun adds more heat to the water 29. When a ray of light is incident normally on
C. temperature is lower than at sea-level an air-glass interface, its angle of refraction
D. relative humidity of the atmosphere is lower is
E. pressure of the atmosphere is lower A. 0o
B. 42o
24. The main reason why rice cooks faster in a C. 45o
pressure cooker than in cooking pot is that D. 60o
A. less heat escapes from the cooker E. 90o
B. the vapour pressure in the cooker is
constant 30. A ray of light is incident on a glass block as
C. the rice is not affected by the draught over shown in the diagram beside. If the
the cooker reflected and the refractive rays are
D. the vapour pressure in the cooker decreases perpendicular to each other, what is the
E. the boiling point of water in the cooker is refractive index of the glass relative to air?
raised A. 1.65
B. 1.58
25. In which of the following media would sound C. 1.52
waves travel fastest? D. 1.50
A. Kerosine E. 1.48
B. Alcohol
C. Water 31. The refractive index of a medium relative to
D. Iron air of 1.8. Calculate the critical angle for the
E. Air medium to the nearest degree.
A. 18o
B. 34o
C. 45o
D. 68o B. 1.0ms-1
E. 90o C. 2.0ms-1
D. 3.0ms-1
32. A converging lens of focal length 5cm forms E. 4.0ms-1
a virtual image which is 10cm from the lens.
How far from the lens is the object? 38. A sound wave of velocity 350ms-1 is
A. 2.0cm directed towards the surface of water. if the
B. 3.3cm ratio of the wavelength of sound in water to
C. 5.0cm that air is 425:100, calculate the velocity of
D. 10.0cm the wave in water.
E. 15.0cm A. 82.4ms-1
B. 148.8ms-1
33. Which of the following instruments produces C. 350.0ms-1
sound by the vibration of air column? D. 1487.5ms-1
A. piano E. 3500.5ms-1
B. guitar
C. flute 39. The speed of sound in air is 330ms -1, how
D. handbell far from the centre of a storm is an observer
E. talking drum who hears a thunder clap 2s after the
lightning flash? (Neglect the time taken by
34. The walls and ceilings of many standard light to travel to the observer)
auditoria are covered with perforated pads A. 1320m
to B. 660m
A. increase the intensity of sound waves C. 600m
B. increase the loudness of sound waves D. 330m
C. reduce the effect of reverberations of sound E. 165m
waves
D. increase the interference effect of sound 40. The diagram below illustrates a variation of
waves the displacement y of a wave particle with
E. decrease the frequency of sound waves time t. If the velocity of the wave is 250ms-1,
calculate the distance between two
35. A string of length 1.0m vibrates in 10 loops. successive particles which are in phase.
if the total mass of the string is 1.0 x 10-3kg A. 2.0m
and the tension in it is 10N, calculate the B. 2.5m
frequency of the vibration C. 3.0m
A. 50Hz D. 5.0m
B. 100Hz E. 50.0m
C. 250Hz
D. 400Hz 41. The diagram above represents a stationary
E. 500Hz wave set up in a string which is fixed at
both ends. If the frequency of the wave is
36. A siren has a disc of 64 holes and makes 20 256Hz and the distance N1 N2 equals 1.5m,
revolutions per second. Calculate the calculate the speed of the wave.
frequency of the sound from the siren. A. 128ms-1
A. 3.2Hz B. 192ms-1
B. 16.0Hz C. 384ms-1
C. 20.0Hz D. 768ms-1
D. 640.0Hz E. 1536ms-1
E. 1280.0Hz
42. Calculate the power delivered by a 3-hase
37. The distance between two successive cest line of its voltage and current are 132KV
of a water is 1m. If a particle on the surface and 60A respectively.
of the water makes to complete vertical A. 2.38 x 107W
oscillations in 1 second. Calculate the speed B. 7.92 x 106W
of the wave. C. 3.96 x 106W
A. 0.5ms-1 D. 2.64 x 106W
E. 7.92 x 104W 48. A cell of e.m.f 1.5V is connected in series
with a resistor of resistance 3Ω. A high-
43. The main function of the mouth piece of a resistance voltmeter connected across the
telephone is to convert sound energy into cell registers only 0.9V. Calculate the
A. heat energy internal resistance of the cell.
B. light energy A. 1.8Ω
C. chemical energy B. 2.0Ω
D. electrical energy C. 2.4Ω
E. mechanical energy D. 4.5Ω
E. 5.0Ω
44. Which of the following is not a component of
a.d.c electric motor? 49. Soft iron is used in making the armature of
A. commutator an electric bell because it
B. field magnet A. retains its magnetism for a long time
C. slip rings B. losses its magnetism
D. Armature C. produces permanent magnets
E. carbon brushes D. is not easily magnetized
E. decreased the magnetic effect of a direct
45. The diagram below illustrates the current
conversion of a galvanometer of resistance
2Ω to an ammeter, the galvanometer gives 50. The diagram beside illustrates an isolated
a fill-scale deflection for a current of 10mA. metal sphere carrying charge W in a
Calculate the value of R. medium whose permittivity is so. The
A. 2.0 x 103Ω magnitude of the electric field intensity P
B. 2.0 x 102Ω can be expressed as
C. 2.0 x 10-1Ω
D. 2.0 x 10-2Ω A. Q2___
E. 2.0 x 10-3Ω 4 π ∈ or2

46. A wire of length 100cm and cross-sectional B. Q__


area 2.0 x 103cm2 has a resistance of 4 π ∈ or2
0.10Ω. Calculate its electrical conductivity.
A. 5.0 x 107Ω-1cm-1 C. Q__
B. 5.0 x 106Ω-1cm-1 4 π ∈ or
C. 2.0 x 106Ω-1cm-1
D. 5.0 x 105Ω-1cm-1 D. Q2___
E. 5.0 x 10-6Ω-1cm-1 4 π ∈ or

47. Lenz’s law of electromagnetic induction E. Q___


states that (4 π ∈ or)2
A. e.m.f is induced in a circuit whenever there
is a change in the magnetic flux linked with 51. The electric potential of a point P in the
the circuit diagram on Question 50 can be expressed
B. the induced current in a conductor is in such as
a direction as to oppose the change
producing it A. Q2___
C. the magnitude of the induced e.m.f in a 4 π ∈ or
circuit is proportional to the rate of change
of the number of lines of force linking the B. Q__
circuit 4 π ∈ or2
D. a force is exerted on a current carrying
conductor in a magnetic field C. Q2__
E. the induced charge is constant for a fixed 4 π ∈ or2
change of flux
D. Q___
4π ∈ or
E. Ex-ray
E. Q___
4 π ∈ 2or 58. Which of the following is used in a nuclear
reactor to slow sown fast-moving neutrons?
52. Calculate the amount of heat generated in A. Carbon dioxide gas
an external load of resistance 8Ω, if an B. liquid sodium metal
alternating current of peak value 5A is C. concrete shield
passed through it for 100s. D. Graphite rods
A. 20,000J E. Boron rods
B. 10,000J
C. 5,000J 59. Which of the following particles/rays do not
D. 4,000J originate from the nucleus of the atom?
E. 200J A. Alpha particle
B. Beta particles
53. In the diagram below the resistor has a C. Neutrons
resistance of 8Ω while the reactance of the D. Gamma rays
inductor and the capacitor are 10Ω and 16Ω E. X-ray
respectively. Calculate the current in the
circuit. 60. The count rate of an alpha-particle source is
A. 3.6A 400 per minute. If the half-life of the source
B. 3.8A is 5 days, what would be the count rate per
C. 9.2A minute after 15 days?
D. 10.0A A. 20
E. 13.0A B. 25
C. 50
54. The resistance in a series R-C circuit is 5Ω. If D. 200
the impendence of the circuit of 13Ω, E. 400
calculate the reactance of the capacitor.
A. 65Ω 1996
B. 18Ω SECTION B THEORY
C. 16Ω
D. 14Ω 1. (a) Define the apparent cubic
E. 12Ω expansibility of
a liquid
55. What is the decay constant of a radioactive
element whose half-life is 3 seconds. (b) (i) Describe with the aid of a
A. 0.132s-1 labeled
B. 0.23s-1 diagram, an experiment to
C. 0.347s-1 determine the apparent cubic
D. 0.693s-1 expansibility of a liquid.
E. 0.924s-1 (ii) State two precautions that
should be taken to ensure
56. Which of the following substance is most accurate result
viscous at room temperature?
A. Water (c) A density glass bottle contains
B. Alcohol 44.25g of a liquid at 0oC and 42.02g
C. Petrol at 50oC. Calculate the real cubic
D. Palm Oil expansibility of the liquid.
E. Kerosine (Linear expansivity of glass = 1.0 x
10-5K-1)
57. Which of the following is not part of the
electromagnetic spectrum? 2. (a) What is meant by dispersion of white
A. Radio wave light?
B. Gamma ray (b) State the colours in the spectrum of
C. Infra-red ray white light in ascending order of their
D. Alpha particle wave lengths.
(c) Which colour is deviated
(i) least (ii) most?
(d) Explain why white light is dispersed
when it passes through a glass
prism.
(e) Describe with the aid of a labeled Use the knife-edged provided to
diagram, how a pure spectrum of determine and record the centre of
white light can be produced on a gravity G of the metre rule. Push the
screen. steel needle firmly into the cork,
then clamp the cork on the retort
3. (a) Explain what is meant by the stand so that the needle is
statement horizontal. Measure and record the
“The capacitance of a parallel-plate distance d between G and the small
capacitor is 2μF” hole at the 5cm mark on the metre
(b) State: rule. Through this hole, suspend the
(i) three factors on which its metre rule freely on the needle as
capacitance depends shown in the diagram above slightly
(ii) three uses of capacitors displace the free end of the metre
(c) Derive a formula for the energy W rule sideways and release to set it
stored in a changed capacitor of into oscillations in the vertical plane.
capacitance C carrying a charge Q Determine the time t for 20 complete
either plate oscillations of the rule. Calculate the
(d) Two parallel-plate capacitors of period T of oscillation. Evaluate d2
capacitance 2μF and 3μF are and T2 d.
connected in parallel and the
combinations is connected to a 50V Repeat the experiment for the holes
d.c source at the 10, 15, 20 and 25cm marks on
Draw the circuit diagram of the the metre rule respectively. In each
arrangement and determine the case, determine the values of t, t,d2
(i) charge on either plate of each and T2d. Tabulate your readings.
capacitor
(ii) potential difference across Plot a graph of T2d on the vertical
each capacitor axis against d2 on the horizontal axis,
(iii) energy of the combined starting both axes from the origin (0,
capacitors 0). Evaluate (i) C = 4 π 2 (ii) K =√1
4. (a) Explain the following terms: s s
(i) viscosity State two precautions taken to
(ii) terminal velocity ensure accurate result.
(b) (i) Describe an experiment to
determine the terminal
velocity of a steel ball falling (b) (i) What is meant by the
through a jar of glycerine. statement the focal length of
(ii) State two precautions that a concave mirror is 20cm?
should be taken to ensure (ii) Draw a array diagram
accurate result. showing how a concave
(c) State two mirror is used as a shaving
(i) effects of viscosity mirror.
(ii) applications of viscosity (iii) If the illuminated object in the
experiment is placed at a
1996 distance 20cm from the pole
PAPER I PRACTICAL of the mirror, state two
characteristics of its image
1. (a) formed on the screen.

3. (a)
2. An object falls freely from a height of 25m
unto the roof of a building 5m high.
Set up the circuit as shown in the Calculate the velocity with which the object
diagram above. Close the key K, use strikes the object. [g = 10ms-1]
the jockey to make contact with the A. 17.3ms-1
potentiometer wire PC such that PB B. 20.0ms-1
= L = 10cm. Record the ammeter C. 24.5ms-1
reading I. Evaluate I-1, repeat the D. 125.0ms-1
experiment for L = 0, 30, 40, 50 and E. 200.0ms-1
60cm respectively. In each case,
3. A body of mass 20kg is set in motion by two
forces 3N and 4N, acting at right angles to
each other. Determine the magnitude of its
acceleration.
A. 400ms-2
B. 2.89ms-2
determine and record the C. 0.35ms-2
corresponding values of I and I-1. D. 0.25ms-2
Tabulate your readings. Plot a graph E. 0.05ms-2
of L on the vertical axis against I-1 on
the horizontal axis starting both axes 4. A ball of man 0.1kg approaching a tennis
from the origin (0, 0). Determine the player with a velocity of 10ms-1, is hit back
slope of the graph and the intercept in the direction opposite with a velocity of
on the vertical axis. Read and record 15ms-1, if the time of impact between the
the value of I-1 when L = 0. State two racked and the ball is 0.01s, calculate the
precautions taken to ensure accurate magnitude of the force with which the ball is
result. hit.
A. 5000N
(b) (i) Deduce from your graph, the B. 500N
value of the current I in the C. 250N
circuit when the jockey makes D. 50N
contact with the E. 25N
potentiometer wire at C.
(ii) State two reasons why the 5. Two blocks of the same dimension, one
key in the circuit should be steel and the other wooden are dropped
opened when readings are simultaneously from the same height. If
not taken. they fall freely, neglecting air resistance,
(iii) State two sources of e.m.f the
other than the chemical cell. A. two blocks hit the ground simultaneously
because they have the same acceleration
B. two blocks reach the ground at the same
1997 time because their dimensions are the same
SSCE PHYSICS C. wooden block, being lighter than the steel
PAPER 2 OBJECTIVE block, reaches the ground first
D. Steel block reaches the ground first because
1. An object of mass 0.40kg attached to the it is denser than the wooden block
end of a string is whirled round in a E. steel block takes half as much time as the
horizontal circle of radius 2.0m with a wooden block to reach the ground because
constant speed of 8ms-1. Calculate the it is more massive than the wooden block
angular velocity of the object.
A. 0.8rads-1 6. Two forces 3 N and 4 N act on a body in
B. 2.0rads-1 directions due north and due east
C. 4.0rads-1 respectively. Calculate their equilibrant.
D. 8.0rads-1 A. 5 N,53o east of north
E. 16.0rads-1 B. 5 N,53o west of south
C. 5 N,37o north of east D. 4.5K
D. 7 N,37o west of north E. 5.0K
E. 7 N,37o south of west
12. Steel bars, each of length 3m at 28oC, are to
7. A missile weighing 400 N on the earth’s be used for constructing a rail line. if the
surface is shot into the atmosphere to an linear expansivity of steel is 1.0 x 10-3K-1,
altitude of 6.40 x 106m. Taking the earth as what is the safety gap that must be left
a sphere of radius 6.40 x 606m and between successive bars if the highest
assuming the inverse-square law of temperature expected is 40oC?
universal gravitation, what would be the A. 1.2 x 10-1cm
weight of the missile at the altitude? B. 7.2 x 10-2cm
A. 100 N C. 6.0 x 10-2cm
B. 200 N D. 3.6 x 10-2cm
C. 400 N E. 1.8 x 10-2cm
D. 800 N
E. 1600 N 13. Which of the following diagrams correctly
represents the variation of the volume V of
8. In the diagram below XY represents a plank a fixed mass of water with temperature T?
used to lift a load from a point X on the
ground onto a horizontal platform YP. What
is the velocity ratio of the plank? A. B. C.
A. XY/ZY
B. XY/XZ
C. ZY/XZ
D. XZ/XY
E. ZY/XY
D. E.
9. A uniform bar 15m long, is balanced on a
pivot placed at its mid-point. A boy of mass
55kg sits on one arm of the bar at a point 14. In which of the following diagram is the
5m away from the pivot. What mass can be pressure of the entrapped air equal to the
placed 2m away from the end of the bar to atmospheric pressure?
keep the bar horizontal?
A. 5.0kg A.
B. 6.9kg B.
C. 50.0kg C.
D. 55.0kg D.
E. 70.0kg E.

10. A solid plastic cube of side 0.2m is 15. A solid material of volume 100cm3 is heated
submerged in a liquid of density 0.8kgm-3. through a temperature difference of 40oC.
Calculate the upthrust of the liquid on the Calculate the increase in the volume of the
cube. [g = 10ms-2] material if its linear expansivity is 2.0 x 10-
A. 0.064N 6 -1
K .
B. 0.025N A. 2.4 x 10-2cm3
C. 0.016N B. 1.6 x 10-2cm3
D. 0.008N C. 8.0 x 10-3cm3
E. 0.003N D. 5.0 x 10-6cm-3
E. 4.0 x 10-6cm3
11. A piece of copper of mass 30g loses 60J of
heat energy. If the specific heat capacity of 16. Which of the following explains why a thick
copper is 400 J kg-1 K-1. Calculate the glass cup cracks when boiling water is
change in temperature of the copper. poured into it?
A. 0.2K A. Large increase in the heat capacity of the
B. 0.8K cup
C. 2.0K B. High density of water
C. Unequal expansion of the interior and 21. Two rays from a very distant object in the
exterior walls of the cup diagram below P, F and C represents
D. Anomalous expansion fo water respectively the pole, focus and centre of
E. Greater specific heat capacity of water curvature of a concave mirror. If two rays
compared with that of glass from a very distant object strike the mirror
as shown, where will the image of the object
17. Calculate the quantity of heat released be located?
when 100g of steam at 100oC condenses to
water. [Take the specific latent heat of A. Between P and F
vaporization of water as 2.3 x 106 J kg-1] B. Between F and C
A. 2.3 x 101 J C. Beyond C
B. 2.3 x 102J D. At F
C. 2.3 x 104J E. At C
D. 2.3 x 105J
E. 3.3 x 107J 22. A conveying lens of focal length 15cm is
used to obtain a real image magnified 1½
18. A fixed mass of gas occupies a volume of times. Calculate the distance of the image
20cm3 at a pressure of 700mm Hg. from the lens
Assuming that the temperature remains A. 37.5cm
constant, what will be the volume of the gas B. 22.5cm
at 750mm Hg? C. 15.0cm
A. 2.5cm3 D. 7.5cm
B. 15.5cm3 E. 3.3cm
C. 18.7cm3
D. 2.14cm3 23. At what distance from a simple microscope
E. 72.5cm3 must an object be placed so that an image 5
times the size of the object is produced
19. A cup containing 100g of pure water at 20oC 20cm from the lens?
is placed in a refrigerator. If the refrigerator A. 2.0cm
extracts heat at the rate of 840 J per minute B. 3.3cm
calculate the time taken for the water to C. 4.0cm
freeze. D. 5.0cm
[Neglect the heat capacity of the materials E. 15.0cm
of the cup]
[Specific heat capacity of water = 4.2J g-1K-1] 24. What is the frequency of a radio wave of
[Specific latent heat of fusion of ice = 336Jg- wavelength 150m if the velocity of radio
1
] waves in free space is 3 x 108ms-1?
A. 15 minutes A. 4.5 x 1010Hz
B. 20 minutes B. 5.0 x 109Hz
C. 42 minutes C. 4.5 x 108Hz
D. 50 minutes D. 2.0 x 107Hz
E. 84 minutes E. 2.0 x 106Hz

20. When white light is incident on a glass prism 25. Whenever light waves are restricted to a
the spectrum produced on a screen placed specific plane, they are said to be
beyond the prism is due to A. diffracted
A. diffraction B. refracted
B. reflection C. diffused
C. refraction D. regularly reflected
D. polarization E. plane-polarized
E. interference
26. The magnification produced with a
converging lens is 5. If the object is a square
wire gauze of side 2cm, calculate the area
of the image.
A. 100cm2
B. 40cm2 C. 1.0 x 103Hz
C. 20cm2 D. 6.7 x 102Hz
D. 01cm2 E. 3.3 x 102Hz
E. 4cm2 32. Two identical waves traveling in the same
direction are superimposed, what should be
27. Which of the following is not true of the the phase difference between the waves for
similarities between a camera and the maximum destructive interference to occur?
human eye? A. 20o
A. Both the camera and the eye have light- B. 45o
proof interiors C. 180o
B. Both the camera and the eye, have light D. 225o
sensitive screens E. 270o
C. Inverted images are formed on the screen in
both 33. A pipe, open at both ends, produces a
D. The distance between the lens and the fundamental note. If the velocity of sound in
screen is fixed in both air is V and L the length of the pipe, which
E. The amount of light entering both the eye of the following expresses the frequency of
and camera can be adjusted the note? [Neglect end-correction]
A. 2V
/L
28. The diagram below shows waves passing B. V
/2L
through a slit S. Which wave phenomenon is C. V
/3L
illustrated by the diagram? D. V
/4L
A. Reflection E. V
/5L
B. Refraction
C. Polarization 34. Which of the following statements is not
D. Diffraction correct?
E. Dispersion A. The pitch of a note depends on the
frequency of the vibrating source
29. A sound note is produced by a ringing bell B. The loudness of a note depends on its
and the echo of the note from a nearby wall intensity
is received 0.5s later. If the frequency of the C. The velocity of sound in air does not depend
note is 400Hz and its wavelength 1m, on the intensity of the note producing it
calculate the distance between the bell and D. The overtones present in a note determine
the wall. its quality
A. 400m E. Wind speed has no effect on the speed of
B. 200m sound in air
C. 125m
D. 100m 35. A progressive wave has a wavelength of
E. 50m 50cm. Calculate the phase difference
between the points at a distance of 20cm
30. The diagram below represents the profile of apart.
10 π
transverse wave. Which of the following A. /3
points are in phase? B. 5
/2 π
4 π
C. /3
A. O and P D. 2
/3 π
B. O and Q E. π /3
C. O and R
D. O and S 36. An alternating current with a peak value of
E. O and T 5 A passes through a resistor of resistance
10.0Ω. Calculate the rate at which energy is
31. The diagram below illustrates the profile of dissipated in the resistor.
a progressive wave in which energy is A. 250.0W
transferred from P to Q in 3.0 x 10-3s. B. 125.0W
Calculate the frequency of the wave. C. 50.0W
A. 2.0 x 103Hz D. 35.4W
B. 1.3 x 103Hz E. 12.5W
A. electrical energy
37. The invisible part of the spectrum of white B. chemical energy
light consists of the following colours: C. heat energy
A. red, orange and yellow only D. mechanical energy
B. infra-red, ultra-violet and blue only E. kinetic energy
C. indigo and green only
D. violet and red only 43. When a resistor of resistance R is connected
E. ultra-violet and infra-red only across a cell the terminal p.d. of the cell is
reduced to three-quarters of its e.m.f. [The
38. A charge of 1.0 x 10-5C experiences a force internal resistance of the cell is expressed
of 40N at a certain point in space. What is as]
the magnitude of the electric field intensity A. R
/4
at the point in Newton per coulomb? B. R
/3
A. 8.0 x 106 C. R
/2
B. 4.0 x 10-6 D. 3R
/4
C. 3.0 x 10-4 E. 3R
D. 2.0 x 10-4
E. 5.0 x 10-7 44. A magnet is placed successively near rings
made of the materials indicated in the
39. An atom radiates 1.5 x 10-19J of energy when diagrams below. The polarity of the magnet
an electron from jumps from one energy in each case is also indicated. Inside which
level to another. What is the wavelength of of the rings will a magnetic field be
the emitted radiation? [Plank’s constant = observed?
6.6 x 10-34 Js; Speed of light in air = 3 x
108ms-1] A. I only
A. 1.32 x 10-6m B. III only
B. 2.98 x 10-6m C. I and III only
C. 3.32 x 106m D. II and III only
D. 6.82 x 10-6m E. I, II and III only
E. 8.01 x 10-6m
45. Calculate the terminal potential difference
40. A capacitor of capacitance 25μF is across a 20Ω resistor connected to a battery
connected to an a.c. power source of of e.m.f. 15V, and internal resistance 5Ω
frequency 200 Hz. Calculate A. 60.0V
π B. 15.0V
reactance of the capacitor. C. 12.0V
A. 0.01Ω D. 6.3V
B. 0.02Ω E. 0.5V
C. 50.00Ω
D. 100.00Ω 46. A magnet is being inserted into a coil of
E. 150.00Ω wire. On which of the following factors does
the induced e.m.f. in the coil depend?
41. The current in a series R-L-C circuit attains I. Numbers of turns in the coil
its maximum value when the II. Strength of the magnet
A. impedance is greater than the capacitive III. Speed with which the magnet is inserted
reactance into the coil
B. inductive reactance is equal to the A. I only
capacitive reactance B. II only
C. inductive reactance is greater than the C. III only
resistance D. I and II only
D. capacitive reactance is less than the E. I, II and III
resistance
E. inductive reactance is greater than the 47. During the electrolysis of copper (11)
capacitive reactance tetraoxosulphate (VI) solution, an ammeter
shows a steady current reading of 1.0A for
42. In a photocell, light energy is converted to 30 minutes while 6.6 x 10-4kg of copper is
liberated. Calculate the error in the 53. When a paint brush is removed from clean
ammeter reading. [The electrochemical water, the bristles of the brush are pulled
equivalent of copper is 3.30 x 10kg C-1] together because of
A. 0.001A A. the viscosity of the surrounding air
B. 0.011A B. the low density of water
C. 0.111A C. surface tension forces
D. 1.000A D. the weight of the brush bristles
E. 1.100A E. the mass of the brush bristles

48. The relationship between the directions of 54. Which of the following is a pair of isotopes?
the magnetic field, the current and the
motion of a current-carrying wire in the field A. 35
Ar and 35
S
is easily remembered using 18 16

A. octet rule
B. right-hand grip rule B. 35
CI and 37
CI
C. Maxwell’s screw rule 17 17

D. Ampere’s swimming rule


E. Fleming’s left-hand rule C. 15
N and 16
O
7 8

49. The current in the primary coil of a


transformer is 2.5A. If the coil has 50 turns D. 30
Si and 30
P
and the secondary 250 turns, calculate the 14 15

current in the secondary coil. (Neglect


energy losses in the transformer) E. 24
mg and 24
Na
A. 0.2A 12 11

B. 0.5A 55. Which of the following statements is not


C. 2.5A true of nuclear fission?
D. 5.0A A. A mother nucleus is broken into two
E. 12.5A daughter nuclei roughly of equal parts
B. Neutron is normally used in bombarding
50. Lenz’s law of electromagnetic induction is heavy nucleus
essentially a statement of C. Two light nuclei are made to combine to
A. inverse-square law of gravitation form a heavy nucleus
B. inverse-square law of magnetism D. very enormous energy is released
C. inverse-square law of electrostatics E. There is an apparent loss in mass
D. law of conservation of momentum
E. law of conservation of energy 56. A nucleus has a proton number of 84. It
emits an χ -particle and then a β -particle
51. The odour of a leaking gas is perceived at a to achieve stability. What is the proton
distance from the source. This is made number of the product?
possible by the process of A. 81
A. sublimation B. 82
B. diffusion C. 53
C. osmosis D. 86
D. evaporation E. 89
E. capillarity
57. In a nuclear reaction, the mass defect is 2.0
52. The half-life of a radioactive material is 6 x 10-6 g. Calculate the energy released,
hours. What quantity of 1kg of the material given that the velocity of light is 3.0 x
would decay in 24 hours? 108ms-1
A. 15/16kg A. 9.0 x 107J
B. 1/2kg B. 1.8 x 108J
C. 1/4kg C. 1.8 x 109J
D. 1/8kg D. 9.0 x 1010J
E. 1/16kg E. 3.6 x 1011J
58. The ratio of tensile stress to tensile strain is (vi) work output of the man
known as (vii) total power developed by
A. modulus of rigidity man given that the time
B. modulus of elasticity taken to raise the box onto
C. shear modulus the platform is 50s. [g =
D. bulk modulus 10ms-2]
E. Young’s modulus 2. (a) Explain the term resonance and give
two
59. Which of the following is not correct about examples
Rutherford’s model of the atom? (b) (i) Describe with the aid of a
A. the mass of an atom is concentrated in the labeled
nucleus diagram, an experiment to
B. The nucleus of an atom is positively charged show how the frequency of
C. The nucleus is surrounded by electron cloud the note emitted by a
D. The model is applicable to atoms with only vibrating string depends on
one electron in the outer shell the length of the string.
E. the diameter of the nucleus is of the order (ii) State two precaution
of 10-15 m necessary to obtain an
accurate result.
60. Which of the following statements about (c) A sonometer wire is plucked and it
kinetic theory of matter is not correct? vibrates emitting a fundamental
A. The molecules of matter are always in note. State the effect on the
motion frequency of the note if the
B. When a body is heated, the average kinetic (i) tension in the wire were made
energy of its molecules decreases nine times as large with no
C. Molecules of a liquid move more freely than change in the length of the
those of a solid wire;
D. Matter is made up of very tiny particles (ii) length of the wire were
called molecules doubled with no change in the
E. the molecules of a gas move more freely tension. [Show clearly how
than those of a liquid you arrive at your answer]

3. (a) Sketch the form of the magnetic flux


pattern due to a current flowing
1997 (i) in a long solenoid
SECTION B THEORY (ii) through two long straight
parallel wires when the
1. (a) Explain the term work directions of the current are
(b) Draw a diagram of a pulley system opposite. [Neglect the earth’s
with a velocity ratio of 5. magnetic field]
(c) A man pulls up a box of mass 70kg (b) Draw a labeled diagram of an electric
using an inclined plane of a effective bell and explain how it works.
length 5m onto a platform 2.5m high (c) An electric bell takes a current of
at uniform speed. If the frictional 0.2A from a battery of two dry cells
force between the box and plane is connected in series. Each cell has an
100N, draw the diagram of all the e.m.f. of 1.5V and an internal
forces acting on the box when in resistance of 1.0Ω.
motion and calculate the (i) Calculate the effective
(i) minimum effort applied in resistance of the bell
pulling (ii) What current would the bell
up the box take if the cells were
(ii) velocity ratio of the plane arranged in parallel?
(iii) mechanical advantage of the
plane 4. (a) Briefly explain what would happen to
(iv) efficiency of the plane a
(v) energy lost in the system
stable element if it is bombarded by the density and relative
α -particles. density of a body.
(b) Explain how the bombardment of (ii) State Archimedes’ principle
Uranium with neutrons could lead to (iii) An object weighs 2.7N in air
nuclear fission chain reaction and and 1.2N when completely
hence nuclear explosion. immersed in water. Calculate
(c) State three characteristics of nuclear its relative density.
activity 2. (a)
(d) State three applications of atomic
energy
(e) State two postulates of Bohr’s model
of the atom and two limitation of
such a model.
1997 Trace the outline ABC of the
PAPER I PRACTICAL equilateral triangular glass prism as
shown above. Remove the prism.
1. (a) Draw a line RN such that it makes an
angle i=20o with the normal. Erect
two pins at Q1 and Q2 on the line
RN. Replace the prism. Place the
reflecting surface of the plane mirror
in contact with the face AC of the
prism. Looking through the face BC
at the prism, fix one pin at Q3 and
another pin at Q4 such that they
appear to be in a straight line with
Pivot the meter rule, which has been the images of the pins at Q1 and Q2.
drilled t the 50cm mark. Suspend the Remove the prism, the mirror and
object marked m at the 10cm mark the pins. Draw a line to join points
of the metre rule. On the other side Q4 and Q3 Produce the line Q4 Q3
of the pivot, suspend the known and that of RN to meet at T.
mass m1 = 100g and adjust its Measure and record angles e and θ
position until the rule balances
horizontally. Read and record the Repeat the experiment for i=25o,
distances d and d1 of m and m1 30o, 40o and 50o respectively, using
respectively from the pivot. Repeat a different outline in each case.
the experiment with m suspended at Determine and record the
the 15, 20, 25 and 30cm marks corresponding values of e and θ for
respectively. In each case, adjust the each tracing. Tabulate your readings.
position of m and determine d and
d1. Also, repeat the entire Plot a graph of θ on the vertical axis
experiment with m completely and e on the horizontal axis, starting
immersed in a beaker of water and both axes from the origin (0, 0).
at 10, 15, 25 and 30cm marks Determine the slope s of the graph
respectively. In each case, read and and the intercept on the vertical
record the new distance d2 of mass axis. State two precautions taken to
m1 from the pivot. Evaluate d1 – d2. ensure accurate results. [Attach your
Tabulate your values d1 d2 and d1 – tracing to your answer booklet].
d2, on the horizontal axis. Determine
the slope s of the graph. State two (b) (i) State the conditions under
precautions taken to ensure accurate which
results. total internal reflection of
light occurs
(b) (i) State two differences (ii) Draw a ray diagram showing
between how a right angled isosceles
glass prism may be used to represents the point of
invert a beam of light. contact of the jockey with the
(iii) With the plane mirror bridge wire when the
removed in the experiment galvanometer shows null
above, a ray of light is deflection. What is the
incident normally on the face potential difference between
AB of the prism. Draw a points T and Q? Explain your
labeled ray diagram showing answer
the path of the ray as it (iii) Two equal lengths of wire
passes through and out of the made of the same material
prism. but of different diameters
3. have an effective resistance
of 0.8Ω when they are
connected in parallel. If the
cross-sectional are of one is
four times the other, calculate
the resistance of the thicker
(a) You are provided with a standard wire.
resistor of resistance R = 1Ω and a
closed loop of bare constants wire. 1998
W. Without straightening out the SSCE PHYSICS
loop, connect a length 1 = 25cm PAPER 2 OBJECTIVE
measured along the loop, between
the points A and B of the metre 1. Which of the following pairs of physical
bridge. Connect other components of quantities comprises vectors?
the circuit as shown in the diagram A. Capacitance and inductance
above. Now obtain a balance at Q on B. Force ratio and velocity ratio
the metre bridge wire PS with the C. Friction and momentum
jockey. Read and record the lengths D. Energy and power
PQ and QS. E. Electric field potential and electric field
Calculate the effective resistance r of intensity
the loop of the wire W given that r =
PO R 2. During the same time interval, it is observed
QS that a train travels the same distance as
Repeat the experiment for I = 35, does a lorry. The two vehicles therefore
45, 55, 65 and 75 respectively. In have the same
each case determine the A. uniform acceleration
corresponding values of PQ, QS and B. instantaneous velocity
r. Tabulate your readings. Now C. initial velocity
straighten out the loop. Measure and D. average velocity
record its total length L. Plot a graph E. average speed
of r on the vertical axis and I on the
horizontal axis. Draw a smooth curve 3. A piece of stone attached to one end of a
through your points. Determine the string is whirled round in a horizontal circle.
value of I for which r is maximum When the string is suddenly cut, the stone
from your graph. State two will
precautions taken to ensure accurate A. move towards the centre of the circle
result. B. stop moving immediately
C. move along a circular path of smaller radius
(b) (i) From the experiment above, D. fly off in a direction tangential to the circular
it can path
be shown that ‘max = 1/4KL. E. acquire greater centripetal force
Using your graph and value of
L deduce the value of K. 4. The term torque means
(ii) In the circuit diagram shown A. the moment of a couple about an axis
in question 3(a) above, Q
B. the resultant of several forces acting on a 9. A pilot records the atmosphere pressure
body in equilibrium outside his plan as 63cm of Hg while a
C. two equal and opposite forces whose lines ground observer records a reading of 75cm
of action do not coincide of Hg with his barometer. Assuming that the
D. two coplanar forces at right angles to each density of air is constant, calculate the
other height of the plane above the ground. (Take
E. the equilibrant of two concurrent forces the relative densities of air and mercury as
0.00136 and 13.6 respectively.
5. A rectangular block of wood floats in water A. 120m
with two-thirds of its volume immersed. B. 138m
When placed in another liquid, it floats with C. 274m
half of its volume immersed. Calculate the D. 1,200m
relative density of the liquid E. 12,000m
A. 1.43
B. 1.33 10. An object is projected with a velocity of
C. 1.00 100ms-1 at an angle of 60o to the vertical.
D. 0.83 Calculate the time taken by the object to
E. 0.75 reach the highest point. (Take g as 10ms-2)
A. 5.0s
The diagram below shows a box X of weight B. 8.7s
W resting on a plank PQ that has its lower C. 10.0s
end P hinged to a horizontal floor PR. The D. 17.3s
plank makes an angle θ with the floor. Use E. 20.0s
this information to answer questions 6 to 8.
11. The earth is not a perfect sphere because
its equatorial axis is longer than the polar
axis. Where on the earth’s surface would an
object have its greatest weight?
A. Halfway between the equator and the north
6. As the end Q of the plank is raised, the pole
component of W, normal to the plank will B. Near the equator
A. increase C. Halfway between the equator and the south
B. decrease pole
C. remain the same D. At either pole
D. be slightly greater than W E. At the equator
E. be independent of W
12. A ball of man 5.0kg hits a smooth vertical
7. If θ is equal to α at the time the box is just wall normally with a speed of 2 ms-1 and
about to slide down the plank, the rebounds with the same speed. Determine
coefficient of static friction between the the impulse experienced by the ball.
plank and the box is A. 20.0 kg ms-1
A. sin α B. 10.0 kg ms-1
B. cos α C. 5.0 kg ms-1
C. tan α D. 1.3 kg ms-1
D. sec α E. 0.0 kg ms-1
E. cot α
13. An object is heated from 30oC. The increase
8. If the box X is pulled up from P to Q, which in its temperature on the Kelvin scale is
of the following expresses the velocity ratio A. 17K
of the inclined plane? B. 27K
A. PR/PQ C. 81K
B. QR/PQ D. 246K
C. QR/PR E. 300K
D. PQ/PR
E. PQ/QR 14. The Lower and upper fixed points of a
mercury-in-glass thermometer are marked Y
and 180mm respectively. On a particular B. 2.5 atmospheres
day the mercury meniscus in the C. 3.0 atmospheres
thermometer rises to 45mm. If the D. 12.8 atmospheres
corresponding reading on a Celsius scale is E. 15.0 atmospheres
10oC, calculate the value of Y.
A. 4mm 20. The inside of a vacuum flask is usually
B. 18mm coasted with silver to reduce heat lost by
C. 30mm A. evaporation
D. 135mm B. condensation
E. 450mm C. radiation
D. conduction
15. A piece of brass of mass 170kg has its E. convection
temperature raised from OoC to 30oC.
Calculate its increase in volume, given the 21. Dry oxygen is trapped by a pellet of
density of brass at OoC as 8.5 x 103kgm-3 mercury in a uniform capillary tube which is
and its cubic expansivity as 5.7 x 10-5 K-1. sealed at one end. The length of the column
A. 3.4 x 10-5m3 of oxygen at 27oC is 50cm. If the pressure of
B. 4.3 x 10-5m3 the oxygen is constant, at what temperature
C. 3.4 x 10-4m3 will the length be 60cm?
D. 3.4 x 10-3m3 A. 360.0oC
E. 3.4 x 10-2m3 B. 240.6˚C
C. 237.0˚C
16. the average kinetic energy of the molecules D. 87.0˚C
of a perfect gas is directly proportional to E. 36.0˚C
the
A. pressure exerted by the gas 22. A waterfall is 1,260m high. Calculate the
B. volume of the gas molecules change in temperature of a quantity of
C. Kelvin temperature of the gas water that falls from the top to the bottom
D. volume of the gas container of the waterfall. (Neglect heat losses to the
E. Celsius temperature of the gas surroundings, take g as 10ms-2 and specific
heat capacity of water as 4200 J kg-1 K-1)
17. How much heat is emitted when a body of A. 0.3˚C
mass 200g cools from 37oC to 31oC? B. 3.0˚C
(Specific heat capacity of the body = 0.4 J g- C. 33.3˚C
1 -1
K ) D. 42.0˚C
A. 4800 J E. 100.0˚C
B. 1200 J
C. 480 J 23. A piece of metal is heated until it becomes
D. 202 J red-hot. It is then quickly transferred into a
E. 90 J beaker containing boiling water. What effect
would this have on the boiling water?
18. If the cubic expansivity of brass between A. The boiling point rises slightly
27oC and 100oC is 5.7 x 10-5K-1, what is its B. The boiling point rises sharply
linear expansivity? C. The rate of vaporization of the boiling water
A. 2.85 x 10-5K-1, increases
B. 1.90 x 10-5K-1 D. the boiling point first rises and then falls
C. 1.86 x 10-5K-1 E. Neither the boiling point nor the rate of
D. 1.70 x 10-5K-1 vaporization increases
E. 1.62 x 10-5K-1
24. A 90-W immersion heater is used to supply
19. Air at temperature 527oC and pressure 30 energy for five minutes. The energy
atmospheres is admitted into the cylinder of supplied is used to completely melt 180g of
an engine. Calculate the pressure of the gas a solid at its melting point. Neglecting
when it has expanded to 5 times its volume energy losses to the surroundings, calculate
and cooled to 127oC as it leaves the engine. the specific latent heat effusion of the solid.
A. 1.5 atmospheres A. 0.5 J g-1
B. 2.5 J g-1 A. diminished, virtual and inverted
C. 15.0Jg-1 B. diminished, virtual and erect
D. 150,0 J g-1 C. diminished, inverted and real
E. 450.0 J g-1 D. magnified, virtual and erect
E. magnified, real and inverted
25. If an ebonite rod is rubbed with fur
A. both the ebonite rod and fur will be 31. An astronomical telescope, having an
negatively charged objective of focal length 100cm and an
B. the ebonite rod will be positively charged eyepiece of focal length 10cm, is used in
while the fur will have no charge normal adjustment. Calculate the separation
C. there will be electron transfer only if the of the lenses.
ebonite rods is earthed A. 0.10cm
D. the ebonite rod will be negatively charged B. 0.90cm
while the fur will be positively charged C. 1.10m
E. the ebonite rod will be positively charged D. 1.80m
while the fur will be negatively charged E. 2.20m

26. An eclipse of the sun by the moon occurs 32. A lantern gives an image 3m square of a
when the sun, the moon and the earth are slide 7.62cm square on a screen. If the
all in a straight line and the screen is 10m from the projection lens of
A. earth casts a shadow on the moon the lantern, calculate the focal length of the
B. moon casts a shadow on the sun lens.
C. moon is between the sun and the earth A. 40.3cm
D. sun is between the moon and the earth B. 26.1cm
E. earth is between the sun and the moon C. 24.8cm
D. 0.7cm
27. The phenomenon which occurs when light E. 0.3cm
changes direction as it passes from one
medium to another is called 33. The intensity of light falling on the film in a
A. reflection camera depends on the
B. polarisation I. brightness of the object
C. interference II. diameter of the aperture stop
D. diffraction III. speed of the shutter
E. refraction Which of the statements above is/are
correct?
28. What is the image distance of an object A. I, II and III
placed at a distance of 2f from a converging B. I and II only
lens of focal length f? C. I and III only
A. f/4 D. II and III only
B. f/2 E. III only
C. f
D. 2f 34. The shortest length of the air column in a
E. 4f resonance tube at resonance is 0.12m and
the next resonant length is 0.37m. Calculate
29. A converging lens of focal length 5cm is the frequency of vibration given that the
used as a magnifying glass by a man whose speed of sound in air is 340ms-1.
near-point is 35cm. Calculate the A. 1360Hz
magnification given by the lens. B. 694Hz
A. 8 C. 680Hz
B. 7 D. 347Hz
C. 6 E. 340Hz
D. 5
E. 4 35. A sonometer wire under a tension of 10 N,
produces a frequency of 250Hz when
30. the images formed by diverging lenses are plucked. Keeping the length of the wire
always constant, the tension is adjusted to produce
a new frequency of 350Hz. Calculate the 41. in the electrical method of magnetization,
new tension. the polarity of the magnet depends on the
A. 39.2 N A. amount of current passed
B. 19.6 N B. direction of current
C. 14.2 N C. magnetic material used
D. 7.4 N D. size of the magnetic material
E. 5.1 N E. orientation of the magnetic material in
space
36. Which of the following instruments produces
sound by the vibration of air column? 42. A cell of e.m.f. 1.5V is connected in series
A. Drum with a resistor of resistance 3.2. A high
B. Violon resistance voltmeter connected across the
C. Guitar cell registers only 0.9V. Calculate the
D. Piano internal resistance of the cell.
E. Flute A. 240Ω
B. 4.5Ω
37. Calculate the wavelength of a note which is C. 2.4Ω
one octave lower than a note of 256Hz in a D. 2.0Ω
medium in which the speed of sound is 352 E. 1.8Ω
ms-1.
A. 0.69m 43. Calculate the resistance of the filament of a
B. 1.38m lamp rated at 240 V 60W.
C. 2.75m A. 240Ω
D. 5.50m B. 360Ω
E. 9.30m C. 960Ω
D. 11440Ω
38. Which of the following are both mechanical E. 2880Ω
and transverse?
A. Infra-red rays 44. The diagram below illustrates a freely
B. Gamma rays suspended bar magnet NS hanging from a
C. Sound waves point in a horizontal ceiling. The thread used
D. Water waves for suspending the magnet and the axis of
E. Micro waves the magnet are as indicated on the
diagram. The angle marked θ is called the
39. An electromagnetic wave of frequency 5.0 x angle of
1014Hz, is incident on the surface of water of
reflactive index. Taking the speed of the
wave in air as 3.0 x 108ms-1, calculate the A. depression
wave length of the wave in water. B. declination
A. 2.2 x 10-6m C. variation
B. 1.7 x 10-6m D. elevation
C. 8.0 x 10-7m E. dip
D. 6.0 x 10-7m
E. 4.5 x 10-7m 45. Lenz’s law of electromagnetic induction
states that
40. The gravitational force of the moon is one- A. the induced charge is constant for a fixed
sixth that of the earth. If a body weighs 6.0 change of flux
N on the moon, calculate its weight on the B. a force is exerted on a current-carrying
earth. conductor in a magnetic field
A. 36.0 N C. the magnitude of the induced e.m.f. in a
B. 12.0 N circuit is proportional to the rate of change
C. 6.0 N of the number of lines of force linking the
D. 2.0 N circuit
E. 1.0 N D. the induced current in a conductor is in such
a direction as to oppose the change
producing it
E. e.m.f. is induced in a circuit whenever there D. 0.75
is a change in the magnetic flux linked with E. 0.60
the circuit
50. Calculate the average power dissipated in
46. A proton of charge 1.6 x 10-19C is projected the circuit.
into a uniform magnetic field of flux density A. 100 W
5.0 x 10-5T. If the proton moves parallel to B. 80 W
the field with a constant speed of 1.6 x C. 60 W
106ms-1, calculate the magnitude of the D. 20 W
force exerted on it by the field. E. 10 W
A. o.0 N
B. 2.O x 10-21 N 51. use the following data to determine the
C. 1.3 x 10-17 N length L of a wire when a force of 30N is
D. 5.1 x 10-14 N applied, assuming Hooke’s law is obeyed
E. 2.3 x 10-13 N
Force applied/N 0 5 10 30
47. the direction of the magnetic field at a point Length of Wire/mm 500. 500. 510. L
in the vicinity of a bar magnet is 0 5 0
A. along the line joining the point to a neutral
point A. 3.0mm
B. always away from the south pole of the B. 3.5mm
magnet C. 503.0mm
C. opposite the direction of the resultant field D. 503.5mm
at that point E. 506.0mm
D. always towards the north pole of the
magnet 52. Which of the following explains the concave
E. the direction towards which the north pole meniscus of water in a clean glass tube?
of a compass needle would point The
A. adhesion between water and glass
48. In a series L – C circuit, the inductance and molecules is greater than the cohesion
the capacitance are 0.5 Hand 20μL, between water molecules
respectively. Calculate the resonant B. cohesion between water molecules is
frequency of the circuit. greater than the adhesion between glass
A. 24.2 Hz and water molecules
B. 36.7 Hz C. molecules of water near the glass move
C. 50.3 Hz faster than the molecules at the centre of
D. 60.5 Hz the tube
E. 80.0 Hz D. molecules of water at the water-air
boundary are often attracted to the centre
49. the diagram below illustrates an a.c. source of the tube
of 50V (r.m.s.), Hz connected in series with E. weight of the water pulls the central part of
an inductor of inductance L and a resistor of the surface down
resistance R. The current in the circuit is 2A
and the p.d. across L and R are 30V and 40V 53. An electron is accelerated from rest through
respectively. Use this information to answer a potential difference of 70kV in a vacuum.
questions 49 and 50. Calculate the maximum speed acquired by
the electron. (Electronic charge = -1.6 x 10-
19
C; mass of an electron = 9.1 x 10-31 kg)
A. 3.00 x 108ms-1
B. 2.46 x 108ms-1
C. 1.57 x 108ms-1
D. 1.32 x 108ms-1
Calculate the power of the circuit. E. 1.11 x 108ms-1
A. 1.33
B. 1.25 54. When light from a source is sent through a
C. 0.80 gas
I. certain wavelengths are absorbed E. 1: 4
II. an emission line spectrum results
III. a dark line is left 60. It is always not possible to determine
Which of the following is/are correct? exactly and simultaneously the position and
A. I only momentum of a particle. This statement is
B. III only known as the
C. I and II only A. De Broglie’s law
D. I and III only B. Heisenberg’s uncertainty principle
E. II and III only C. Compton effect
D. Frank-Hertz experimental law
55. Calculate the value of the half-life of a E. Wave-particle paradox
radioactive element whose decay constant
is 0.077s-1. 1998
A. 0.5s SECTION B THEORY
B. 3.0s
C. 5.1s 1. (a) Explain the term uniform
D. 9.0s acceleration
E. 12.5s
(b) (i) Sketch and describe the
56. The process of increasing the energy of an velocity-
atom via inelastic collision with an electron time graph for the motion of a
is known as ball from the time it is
A. ionization projected vertically upwards
B. excitation until it returns to the point of
C. field emission projection.
D. photoemission (ii) Neglecting air resistance and
E. thermionic emission using your sketch, explain
how the acceleration of free
57. A radioactive nuclide of proton number X fall due to gravity, g, and the
emits a β particle to form a new nuclide of maximum height attained
proton number Y. The correct equation when the ball is projected
relating X and Y is vertically upwards can be
A. X=Y–1 determined.
B. X=Y+1
C. X=1–Y (c) A stone is projected vertically
D. X = 1/y upwards with a velocity of 20ms-1.
E. X=Y Two seconds later, a second stone is
similarly projected with the same
58. The frictional effect between the layers of a velocity. When the two stones meet,
moving fluid is called the second one is rising at a velocity
A. capillarity of 10ms-1. Neglecting air resistance,
B. turbulence calculate the:
C. diffusion (i) length of time the second
D. osmosis stone is in motion before they
E. viscosity meet.
(ii) velocity of the first stone
59. Two radioactive elements X and Y have half- when they meet (Take g as
lives of 100 and 50 years respectively. 10ms-2)
Samples of X and Y initially contain equal
number of atoms. What is the ratio of the 2. (a) Distinguish between heat and
number of the remaining atoms of X to that temperature.
of Y after 200 years? (b) State two physical properties of
A. 4: 1 substances which may be used to
B. 3: 1 measure temperature
C. 1: 1
D. 1: 2
(c) State two reasons why mercury is (ii) maximum kinetic energy of
preferred to alcohol as a the photo electrons?
thermometric liquid.
(d) State one similarity and one
3. (a) (i) Describe, with the aid of a difference between photoemission
circuit and evaporation.
diagram, an experiment to
measure the resistance of a (e) Name two methods by which a
wire given an ammeter of low beam of free electrons may be
resistance, a battery, a key, a produced other than photoemission.
rheostat, a high-resistance
voltmeter and some (f) State two application to
connecting wires photoelectric effect.
(ii) State two precautions (g) a light of wavelength 5.0 x 10-7m is
necessary to obtain an incident on a metal resulting in
accurate result. photoemission of electrons if the
work function of the metal is 3.04 x
(b) Using the experimental result and 10-19J, calculate the:
any necessary measurements, (i) frequency of the light
explain how the resistivity of the wire (ii) energy of the incident photon
may be determined. (iii) maximum kinetic energy of
the photoelectrons
(c) Two cells each of e.m.f. 2V and Speed of light – 3.00 x 108ms-
internal resistance o.5Ω are 1

connected in series. They are made Plank’s constant – 6.6 x 10-34Js


to supply current to a combination of 1998
three resistors, one of resistance 2Ω PAPER I PRACTICAL
connected in series to a parallel
combination of two other resistors 1. (a)
each of resistance 3Ω.
Draw the circuit diagram and
calculate the:
(i) current in the circuit You are provided with a uniform
(ii) potential difference across metre rule, a knife edge and a body
the parallel combination of m of mass 50g. Suspend the given
the resistors, body M by means of a thread from
(iii) lost volts of the battery. 1.00cm mark of the metre rule.
Balance the loaded metre rule on the
4. (a) Explain the term photoelectric effect. knife edge as shown in the diagram
above. Determine and record the
value of X when the metre rule is in
horizontal equilibrium. Evaluate 1/x.
Repeat the experiment for the value
of m = 70, 90, 110, 130 and 150g
respectively. In each case determine
and record the corresponding values
of X and 1/x. Tabulate your readings.
(b) The diagram above represents a Plot a graph of m on the vertical axis
photocell with its associated electric and 1/x on the horizontal axis;
circuit. Identify each of the physical starting both axes from the origin (0,
quantities represented by the letters 0). Determine the slope of the graph
A, B, C, D, E and F. and the value of m for which 1/x = 0.
State two precautions taken to
(c) What factor determines the: ensure accurate result.
(i) current produced by the
photocell,
(b) (i) Using your graph, determine (iii) State two conditions
the necessary for total internal
value of x for which m = 0 reflection to occur in a
(ii) State two conditions medium.
necessary to maintain the
metre rule in the experiment 3. (a)
above in equilibrium.
(iii) Using your graph, determine
the value of h for which m =
100g.

2. (a)
Connect the circuit as shown above.
Set
the value of R = 30Ω. Close the key
and obtain a balance at point T on
Trace the outline PQRS of the glass the potentiometer wire PQ. Read and
block on a sheet paper as shown record the length TQ = L. Evaluate L-
above. Remove the block. Mark a 1
and R-1. Repeat the experiment for
position O very close to P. Draw the R = 20, 10, 5, 3 and 1Ω respectively.
normal NOG. From the point G, In each case, determine and record
measure and mark out points B1, B2, the corresponding values of L, L-1 and
B3, B4, and B5, along the GR at R-1. Remove the resistance box from
distances 1, 2, 3, 4 and 5m the circuit and then determine the
respectively from G. replace the length Lo corresponding to R = O.
glass block on the outline PQRS. Tabulate your readings. Plot a graph
Erect a pin at O and another at B1. of R-1 on the vertical axis and L-1 on
Now fix a pin at T1 such that the pin the horizontal axis, starting both
at T1 and B1 are in line with the pin at axes from the origin (0, 0).
O when viewed through the side SR Determine the slope s of the graph
of the glass block. Remove the glass and its intercept 1 on the vertical
block. Join the line OB1 and B1 T1. axis.
Measure and record that angles x Evaluate: (i) K = 1-1 (ii)
and y. Evaluate sin x and cos y. C = Lo/s State two precautions
Repeat the experiment with the pin taken to ensure accurate
at B1 now fixed at B2, B3, B4, and B5 result.
respectively while the pin at 0
remains unaltered. In each ‘case, (b) (i) using your graph, determine
measure and record the values of x, the
y, sin x and cos y. Tabulate your value of L for which R = 15Ω
readings. Plot a graph of sin x on the
vertical axis and cos y on the (ii) If the intercept 1 = 0.5 + y1,
horizontal axis, starting both axes use your graph to determine
from the origin (0, 0). Calculate slope the value of y.
s of the graph. Evaluate K = 1/s, (iii) Explain what is meant by the
state two precautions taken to e.m.f of a cell.
ensure accurate result. [attach your
tracings to your answer booklet].

(b) (i) State Snell’s law of refraction


and explain why refraction
occurs at the boundary
between two media
(ii) Differentiate refraction from
diffraction
1999
SSCE PHYSICS
PAPER 2 OBJECTIVE

1. The force between molecules of the same


substance is termed.
A. elastic force
B. repulsive force
C. cohersive
D. adhesive force

2. A net force of magnitude 0.6N acts on a


body of mass 40g, initially at rest. Calculate
the magnitude of the resulting acceleration
A. 90ms-2
B. 60ms-2
C. 30ms-2
D. 15ms-2

3. Palm oil from a bottle flows out more easily


after it has been heated because the
A. Molecules are given potential energy during shown in the diagram beside. Calculate the
the heating value of the tension T, [g = 10ms-1]
B. friction between oil layers is reduced
C. oil molecules force each other out A. 300.0N
D. adhesion between the oil molecules and B. 173.2N
those of the bottle is increased C. 30.0N
D. 17.3N
4. A body weighing 100N moves with a speed
of 5ms-1 in a horizontal circular part of 10. A pendulum bob executing simple harmonic
radius 5m. Calculate the magnitude of the motion has 2cm and 12Hz as amplitude and
centripetal force acting on the body. [g = frequency respectively. Calculate the period
10ms-2] of the motion.
A. 10N A. 2.00s
B. 50N B. 0.83s
C. 75N C. 0.08s
D. 100N D. 0.06s

5. A motor-cyclist, passing a road junction, 11. A trolley of mass 4kg moving on a smooth
moves due west for 8s at a uniform speed of horizontal platform with a speed of 1.0ms-1
5ms-1. He then moves due north for collides perfectly with a stationary trolley of
another 6s with the same speed. At the end the same mass on the same plat form.
of the 6s his displacement from the road Calculate the total momentum of the two
junction is 50m in the direction of trolleys immediately after the collision.
A. N53oE A. 0.5Ns
B. N37oE B. 1.0Ns
C. N53oW C. 4.0Ns
D. N37oW D. 8.0Ns

6. The time rate of change of displacement is 12. The ice point of an ungraduated mercury-in-
known as glass thermometer is X, while its steam
A. speed point is 90o. This thermometer reads 60o
B. velocity when the true temperature is 40oC.
C. impulse Calculate the value of X.
D. acceleration A. 60˚
B. 48˚
7. A store of mass 0.7kg is projected vertically C. 40˚
upwards with a speed of 5ms-1. Calculate D. 30˚
the maximum height reached. [Take g as
10ms-1 and neglect air resistance] 13. Iron at 30˚C feels cooler to the hand than
A. 1.00m wood at 28˚C because
B. 1.25m A. their linear expansivities are not equal
C. 1.50m B. the temperature of the iron higher than that
D. 3.75m of the wood
C. iron is a better radiator of heat than wood
8. If the force of attraction between the sun D. iron conducts heat more quickly from the
and the planets is removed, the planets will hand than wood
A. fall towards the sun
B. scatter and stop moving 14. An iron rod of length 50m and at a
C. continue to move at tangent to their original temperature of 60˚C is heated to 70˚C.
orbit Calculate its new length [Linear expansivity
D. continue to move perpendicular to their of iron = 1.2 x 10-5K1]
original orbits A. 50.006m
B. 50.060m
9. A 15kg mass suspended from a ceiling is C. 51.600m
pulled aside with a horizontal force, F as D. 51.200m
15. The heat capacity of a substance is the A. 1,050W
energy B. 16,800W
A. required to raise the temperature of a unit C. 18,060W
mass of the substance by one degree D. 20,160W
B. required to raise the temperature of the
substance by one degree 21. A ray of light is incident on a plane mirror at
C. absorbed by the substance at constant an angle of 20˚C. This mirror is rotated
temperature through, twice this angle. In this new
D. lost by a unit mass of the substance position, the angle between the incident ray
and the reflected ray is
16. The temperature of a piece of metal of mass A. 20˚
9g is raided from 10˚C to 110˚C when it B. 40˚
absorbs 108J of heat energy. Determine the C. 80˚
specific heat capacity of the metal in J kg-1K- D. 120˚
1
.
A. 1.2 22. The air column in a resonance tube is set
B. 12.0 into resonance by a vibrating turning fork.
C. 120.2 The resultant waves in the air column will
D. 1200.0 be
A. Mechanical and transverse
17. Which of the following statements is not B. progressive and longitudinal
correct? C. stationary and transverse
A. A sea breeze is due to convection in air D. stationary and longitudinal
B. Cotton materials are better than woolen
ones for use in both weather 23. The image of an object is located 6cm
C. Convectional currents play an important behind a convex mirror. If its magnification
role in the cooling of the engine of a motor is 0.6, calculate the focal length of the
car mirror
D. the vacuum space in a flask helps to reduce A. 3.75cm
heat loss by radiation B. 6.60cm
C. 10.00cm
18. Calculate the heat energy required to D. 15.00cm
change 0.1kg of ice at 0˚C to water boiling
at 100˚C. [Specific heat capacity of water 24. The refractive index of glass for yellow light
= 4200 J kg`k1] [Specific latent heat of is greater than that for red light because
fusion of ice = 336.000J kg-1] yellow light
A. 75,600 J A. has a greater amplitude than red light
B. 336,000 J B. is more intense than red light
C. 340,200 J C. travels more slowly than red light
D. 378,000 J D. deviates less than red light

19. Which of the following statements about 25. The real image of an object formed by a
evaporation is not correct? converging lens of focal length 15cm is
A. The rate of evaporation of a liquid varies three times the size of the object. Calculate
with temperature the object distance
B. The molecules of an evaporating liquid have A. 60cm
an average speed B. 30cm
C. Evaporation occurs when faster molecules C. 20cm
escape from the surface of a liquid D. 15cm
D. Evaporation takes place inside a liquid
26. Which of the following statements is not
20. Calculate the power rating of an immersion correct about long sight?
heater used for 10 minutes to increase the A. A long-sighted person can see distant
temperature of 10kg of water by 15K. objects clearly
[Specific heat capacity of water = 4200 J kg- B. light from a nearby object is focused behind
1
k-1] the retina
C. The eyeball is too short 33. The Parts of a bar magnet at which the
D. The defect is corrected by using a diverging magnetic effect is strongest are called the
lens A. poles
B. neutral points
27. The amplitude of a wave is the C. magnetic declination
A. distance between two successive through of D. magnetic meridians
the wave
B. separation of two adjacent particles 34. A resistance wire of length 2m and of
vibrating in phase uniform cross sectional area 5.0 x 107m2 has
C. maximum displacement of the wave particle a resistance of 2-2Ω. Calculate its resistivity.
from the equilibrium position A. 1.8 x 10-7Ωm
D. distance travelled by a wave in a complete B. 5.5 x 10-7Ωm
cycle of its motion C. 1.8 x 107Ωm
D. 5.5 x 107Ωm
28. In order to obtain a sound note of a high
pitch from a wire stretched by a constant 35. Magnetic flux density is defined as the
tension, the wire must be A. total number of magnetic lines of force
A. short and thick surrounding a magnet
B. short and thin B. number of magnetic lines of force per unit
C. long and thick area normal to the magnetic field
D. long and thin C. strength of the magnetic field surrounding a
current carrying conductor
29. If the position of resonance in a resonance D. magnetic force exerted on a unit magnetic
tube is 16.50cm from the open end of the pole
tube, calculate the distance from the open
end to the next position where resonance 36. The voltage and current in the primary of a
occurs. [Neglect end-correction] transformer are 200Y and 2V respectively. If
A. 24.7cm the transformer is used to light ten 12V,
B. 33.00cm 20W bulbs, calculate its efficiency.
C. 41.25cm A. 100%
D. 49.50cm B. 90%
C. 75%
30. Thunder is usually heard some seconds D. 50%
after lighting is observed because
A. the human eye is more sensitive to light 37. In a uniform electric field, the magnitude of
than the ear to sound the force on a charge of 0.2C is 4N.
B. sound and light travel in different media Calculate the electric field intensity.
C. thunder occurs after lightning A. 20Nc-1
D. light travels faster than sound B. 8NC-1
C. 5NC-1
31. Using the diagram beside, calculate the D. 2NC-1
effective capacitance of the circuit.
A. 1.56µF 38. A rocket of mass m is fired from the Earth’s
B. 3.00µF surface such that it just escapes from the
C. 3.7µF Earth’s gravitational field. If R is the radius
D. 9.00µF of the Earth and g the acceleration of free
fall due to gravity, the escape velocity of
32. Using the data on the diagram beside, the rocket is expressed as
calculate the potential difference across the A. 2gR
20Ω resistor. [Neglect the internal
resistance of the cell] B. 2 / Rg
A. 5V
B. 10V C. 2 / Rg
C. 20V
D. 60V D. R2 / 2g
39. A 10-Ω coil takes 21s to melt 10g of ice at 44. The following statements relate to atomic
0oC. Assuming no heat losses, determine spectrum
the current in the coil. [Specific latent heat I. All elements emit and absorb characteristic
of fusion of ice = 336 Jg-1] spectra
A. 16A II. Spectral analysis is an important method of
B. 5A identifying environmental pollutants
C. 4A III. The chemical composition of stars could be
D. 3A determined using spectral analysis
Which of the following statements above are
40. A point charge of magnitude 2C is moved correct?
through a distance of 0.20m against a A. I, II and III
uniform field of intensity 25Vm-1. Calculate B. II and III only
the work done on the charge. C. I and III only
A. 5.0 x 10-6 J D. I and II only
B. 1.0 x 10-5 J
C. 5.0 x 10-6 J 45. A metal has a work function of 4.375eV.
D. 1.0 x 10-2 J Calculate its threshold frequency. [h = 6.6 x
10-34Js, leV = 1.6 x 10-19J]
41. Bohr’s atomic model proves most successful A. 2.01 x 1015Hz
for the explanation of the: B. 1.06 x 1015Hz
I. structure of the hydrogen atom C. 6.30 x 1014Hz
II. line spectra of the hydrogen atom D. 1.60 x 1014Hz
III. multi-electron atoms
Which of the following statement is/are 46. The half-life of a radioactive substance is
correct? A. its average life time
A. I only B. one-half the value of its decay constant
B. II only C. the length of time needed by the substance
C. I and II only to decay completely
D. I and III only D. the length of time during which the activity
of the substance reduces by 50%

47. The half-life of a radioactive element is 5s.


Calculate its decay constant
A. 0.113s-1
B. 0.136s-1
C. 0.139s-1
Use the diagram above to answer Questions D. 0.193s-1
42 and 43.

42. The diagram above illustrates the energy 48. A sheet of paper is placed in the path of the
transitions of five electrons of an atom. following radiations
Which of the transitions will produce the I. Alpha particles
emission of longest wave length? II. Beta particles
A. I III. Gamma rays
B. II Which of the radiations will pass through the
C. III sheet of paper
D. IV A. I only
B. II only
43. Which of the transitions will produce C. I and II only
emission of highest frequency? D. II and III only
A. V
B. IV 49. Which Of the following are the essential
C. III parts of an atomic bomb?
D. II A. Uranium and neutrons
B. Radium and polonium
C. Nitrogen and neutrons
D. Uranium and a-particles

50. Which of the following is not a safety


precaution in a nuclear station?
A. Radioactive materials are stored in gas
casings
B. Radioactive materials are handled with
remote controlled tongs
C. Technicians in nuclear stations wear radio
sensitive badges which are checked
regularly
D. Radioactive materials are kept in thick-
walled lead container

1999
SECTION B THEORY PART I

1. Explain plane polarization of light.

2. A stone is projected horizontally from the


top of a tower with a speed of 5ms-1. It lands
on the ground level at a horizontal distance
of 20m from the foot of the tower. Calculate
the height of the tower. [g = 10 ms-2].

3. In an electrolysis experiment, the ammeter


records a steady current of 1A. The mass of
copper deposited is 0.66g in 30 minutes.
Calculate the error in the ammeter reading.
[Electrochemical equivalent of copper =
0.00033gC-1]

4. (a) What is surface tension?


(b) State two methods by which the (c) A bullet of mass 120g is fired
surface tension of a liquid can be horizontally into a fixed wooden
reduced. block with a-speed of 20ms-1. The
bullet is brought to rest in the block-
5. (a) Define Young’s modulus in, 0.1 s by a constant resistance.
(b) State the physical quantities one has Calculate the:
to measure in order to determine the (i) magnitude of the resistance
Young’s modulus of a wire. (ii) distance, moved by the bullet
in
6. (a) What is the principle upon which the the wood
lighting in a fluorescent tube
operate? 2. (a) Explain (i) Work (ii) power
(b) State two factors on which the colour (b) Show that the efficiency E, the force
of light from a fluorescent tube ratio
depends. M.A and the velocity ratio V.R of a
machine are related by the equation:
7. (a) List two types of waves, apart from M.A x 100%
light VR
that can be plane polarized. (c) An inclined plane of angle 10o is
(b) State two applications of plane used to
polarized light raise a load of 4500N through a
height of 2m. If the plane is 75%
8. (a) State Hooke’s law of elasticity. efficient, calculate
(b) A spiral spring, loaded with a piece of (i) velocity ratio of the plane
metal, extends by 10.5cm in air, (ii) work done on the load
when the metal is fully submerged in (iii) Explain Charles’ law using the
water, the spring extends by 6.8cm. kinetic theory of matter.
Calculate the relative density of the
metal. [Assume Hooke’s law is 3. (a) State three properties of waves
obeyed] (b) (i) Describe with the aid of a
labeled diagram, an
9. (a) Explain electrolysis experiment to show how the
(b) Classify the following substances as frequency of the note emitted
electrolytes and non-electrolytes, by a vibrating string depends
Sugar solution, paraffin, salt solution on the tension in the string.
and grape juice (ii) State two precautions
necessary to ensure accurate
10. (a) State Heisenberg’s uncertainty results.
principle (c) Draw a ray diagram showing how a
(b) Mention two phenomena that can virtual image of an object is formed
only be explained terms of the by a concave mirror.
particulate nature of light.
PART II 4. (a) Sketch the magnetic flux pattern
around a
1. (a) State the conditions for the long, straight, current carrying wire.
equilibrium of (b) State two methods by which the
a rigid body acted upon by parallel sensitivity of a moving coil
forces galvanometer can be increased.
(b) (i) Describe an experiment,
using the
Principle of moments to
determine the mass of a (c) A series RLC circuit comprises a 100-
metre rule. Ω resistor a 3-H inductor and a 4-µF
(ii) State two precautions capacitor. The a.c source of the
necessary to ensure accurate circuit has an e.m.f of 100V and
results frequency of 160 Hz
π
(i) Draw the circuit diagram of
the arrangement.
Calculate the:
(ii) Capacitance reactance
(iii) Inductive reactance
(iv) Impedance of the, circuit
(v) Current in the circuit
(vi) Average power dissipated in
the circuit

5. (a) Explain
(i) fusion (ii) fission
(b) State three advantages of fusion
over fission in the generation of
power.
(c) Calculate, in joules, the binding
energy for 59 Co.
27
[Atomic mass of 59 Co = 58.9332u]
27
[Mass of proton = 1.00783u]
[Mass of neutron = 1.00867u
[Unified atomic mass unit, U = 931 M
ev]
[1 eV = 1.6 x 10-19J]

1999
PAPER I PRACTICAL
ray such that the angle of incidence i
= 30o. Fix two pins at points P and Q
on the incident ray.
1. (a) In the diagram above, a thread AC, Replace the block and fix two other
fixed pins at points U and V such that the
at pulley A passes over pulley C on a pins appear to be in a straight line
force board and carries an unknown with V such that the pins appear to
mass mo. Retain this mass mo be in a straight line with the images
throughout the experiment. of the pins at P and Q when viewed
through the block. Remove the block
Draw a line along the direction of AC and join the points at V and U,
on the paper held behind the thread. producing the line to meet DC at T.
Locate the mid-point B of AC and Join OT with O as centre and using
mark its position on this line. any convenient radius, draw a circle
to cut the incident and refracted rays
Draw PB at right angles to AC. at R and S respectively. Draw the
perpendiculars RN and MS. Measure
By means of a loop of thread, and record RN and MS.
suspend a mass M = 50g from AC Repeat the experiment for i = 40o,
and adjust the position of the loop so 50o, 60o and 70o respectively. In
that the line of action of the weight each case; determine and record the
of M lies along BP. Ensure that M corresponding values of RN and MS.
and mo hang off the force board. Tabulate your readings. Plot a graph
Measure BO = y and AO. Evaluate of RN on the vertical axis and MS on
y/AO. the horizontal axis. Determine the
slope s of the graph.
Repeat the experiment for M = 70,
90, 110 and 130g respectively. In State two precautions taken to
each case, determine the ensure accurate results.
corresponding values of y, AO and
y/AO. Tabulate your readings. (b) (i) Explain refraction.
(ii) Draw a diagram showing why
Plot a graph of y/AO on the vertical a metre rule, partly immersed
axis and M on the horizontal axis. in water and place obliquely
Determine the slope s of the graph. to the surface, appears bent
at the surface.
State two precautions taken to
ensure accurate results. 3. (a)

(b) (i) Distinguish between the


resultant and the
equilibrant of forces.
(ii) State two conditions
necessary for
the equilibrium of three non-
parallel co-planar forces.
Measure and record the e.m.f of the
2. (a) accumulator provided. Connect the
circuit as shown in the diagram
above. With zero resistance in the
resistance box, adjust the rheostat to
obtain the maximum possible
Trace the outline ABCD of the glass reading on the ammeter. Do not
block on the sheet of paper as shown adjust the rheostat again throughout
above. Remove the block and draw the experiment. Open the key.
the normal at O. Draw an incident
With R = 1Ω, close the key, read and
record the reading of the ammeter
IA. Calculate I-1A.

Repeat the experiment for R = 2, 3,


4 and 5Ω respectively. In each case,
read and record the value of IA, and
calculate the corresponding value of
I-1A.

Now connect the resistor Q in series


into the circuit and without altering
the setting of the rheostat, record
the new ammeter readings IB for R =
1, 2, 3, 4, and 5Ω.
Calculate the corresponding value of
I-1B, in each case. Tabulate your
readings.
On the same graph and using the
same axes and scales, plot a graph
of:
(i) IA-1 on the vertical axis and R
on the horizontal axis;
(ii) IB-1 on the vertical axis and R
on the horizontal axis.

(b) Calculate the slopes SA in (i) and SB


in (ii) above. Determine the
difference D between the intercepts
of the graphs on the vertical axis.

State two precautions taken to


ensure accurate results.
(i) Explain why a battery of eight
dry Leclanche cells, each of
e.m.f 1.5V, is not normally
used in place of a motor-car
battery of 12 V to start a car.
(ii) State two sources of e.m.f
other than the chemical cell.
2000 6. A wooden block of mass 1.6kg rests on a
SSCE PHYSICS rough horizontal surface. If the limiting
PAPER 2 OBJECTIVE frictional force between the block and the
surface is 8N. Calculate the coefficient of
1. When the surface of a piece of chalk is friction. [g = 10ms-2]
scrapped the tiny particles that flake off are A. 0.6
known as B. 0.5
A. matter C. 0.3
B. molecules D. 0.2
C. elements
D. atom 7. A ball bearing is gently released from rest
and allowed to fall through a viscous fluid.
2. The diagram below represents a section of a Which of the following statements about the
pair of vernier calipers. The reading on the motion is correct?
instrument is A. Its acceleration decreases before terminal
velocity is attained
B. When terminal velocity is attained the
acceleration of the fluid becomes zero
C. Its velocity increases before terminal
A. 5.22cm velocity is attained
B. 5.24cm D. There is no resultant force on the ball before
C. 5.25cm it attains terminal velocity
D. 5.26cm
8. The driver of a car moving with a uniform
3. An iron rod is moved from the earth to the speed of 40ms-1 observes a truck
moon. Which of the following properties of approaching in the opposite direction with a
the rod would remain unchanged? speed of 20ms-1. Calculate the speed of the
I. Mass car relative to that of the truck.
II. Weight A. 0.5ms-1
III. Relative density B. 2.0ms-1
A. I only C. 20.0ms-1
B. II only D. 60.0ms-1
C. III only
D. I and III only 9. A bus traveling at 15ms-1 accelerates
uniformly at 4ms-2. What is the distance
4. A uniform cylindrical hydrometer of mass covered in 10s?
20g and cross sectional area 0.54cm2 floats A. 150m
upright in a liquid. If 25cm of its length is B. 170m
submerged, calculate the relative density of C. 350m
the liquid. [Density of water = 1 gcm-3] D. 600m
A. 1.54
B. 1.48 10. Which of the following sketches represents
C. 1.25 the velocity-time graph of the motion of a
D. 0.80 stone projected vertically upwards and
allowed to return to the point of projection?
5. The apparent weight of a body wholly
immersed in water is 32 N and its weight in A. V B. V C. V
air is 96 N. Calculate the volume of the
body. [Density of water = 1000kgm-3 g =
10ms-2] t t
A. 8.9 x 10-3m3 t
B. 6.4 x 103m3
C. 3.2 x 10-3m3
D. 3.0 x 103m3 D. V
t can be determined from the information
11. The diagram above illustrates three forces given
T1 T2 and 100N in equilibrium. Determine A. Speed of each of the ball after collision
the magnitude of T1. B. Kinetic energy of each ball after collision
A. 100 tan 30o C. Total momentum of the two balls after
collision
B. 100__ D. Mutual forces exerted by the balls
Cos 30o
17. Using the forces-displacement diagram
C. 200 sin 30o shown below, calculate the work done.

D. 200__ A. 2000J
Tan 30o B. 1000J
C. 20J
12. A uniform metre rule of mass 90g is pivoted D. 5J
at the 40cm mark. If the rule is in
equilibrium with an unknown mass m placed 18. A machine of efficiency 80% is used to raise
at the 10cm mark and a 72g mass at the a body of mass 75kg through a vertical
70cm mark, determine m height of 3m in 30s. Calculate the power
A. 72g input. [g = 10ms2]
B. 1012g A. 9.4W
C. 198g B. 60.0W
D. 504g C. 75.0W
D. 93.8W
13. The resultant of two forces acting on an
object is maximum when the angle between 19. Two bodies, P and Q are in thermal
them is equilibrium. Which of the following
A. 180o statements about the bodies is correct?
B. 90o A. The temperature of Q is higher than that of
C. 45o P
D. 0o B. P and Q have the same heat capacity
C. P and Q have the same mass
14. The bob of a simple pendulum takes 0.25s D. P and Q are the same temperature
to swing from its equilibrium position to one
extreme end. Calculate its period. 20. The cubic expansivity of mercury is 1.8 x 10 -
A. 0.25s 4
k-1 and the linear expansivity of glass is 8.0
B. 0.50s x 10-6 k-1, calculate the apparent expansivity
C. 0.75s of mercury in a glass container.
D. 1.00s A. 1.00 x 10-4 k-1
B. 1.56 x 10-4 k-1
15. The period of oscillation of a particle C. 1.72 x 10-4 k-1
executing simple harmonic motion is 4 π D. 2.04 x 10-4 k-1
seconds. If the amplitude of oscillation is
30.0m. Calculate the maximum speed of the 21. The diagram below shows the variation of
particle. volume V of a glass with temperature in a
A. 1.5ms-1 Charle’s law experiment. The value of the
B. 3.0ms-1 temperature at the point x.
C. 4.5ms-1
D. 6.0ms-1 A. –32oC
B. –100oC
16. An inelastic collision takes place between C. –273oC
balls of known masses. Just before the D. –373oC
collision, one of the balls is moving with a
known velocity while the other is stationary. 22. The volume and pressure of a given mass of
Which of the following physical quantities gas at 27oC are 76cm3 and 80cm of mecury
respectively. Calculate its volume at s.t.p
A. 36.2cm3 B. 80o
B. 72.8cm3 C. 100o
C. 100.0cm3 D. 120o
D. 808.9cm3

23. A piece of metal of mass 50g is cooled from 29. A concave mirror can be used to produce a
80oC to 20oC. Calculate the amount of heat parallel beam of light if a lighted bulb is
lost. [Specific heat capacity of the material placed.
of metal = 450 Jkg-1 k-1] A. between its focus and the pole
A. 4.50 x 103 J B. at its focus
B. 2.25 x 103 J C. at its centre of curvature
C. 1.80 x 103 J D. between its focus and centre of curvature
D. 1.35 x 103 J
30. A simple microscope forms an image 10cm
24. The mass of water vapour in a given volume from an eye close to the lens. If the object is
of air is 0.05g at 20oC, while the mass of 6cm from the eye, calculate the focal length
water vapour required to saturate it at the of the lens.
same temperature is 0.15g. Calculate the A. 3.75cm
relative humidity of the air. B. 4.00cm
A. 3.33% C. 15.00cm
B. 5.55% D. 16.00cm
C. 33.33%
D. 55.55% 31. When white light passes through a
triangular glass prism, there is dispersion
25. The temperature at which the saturated because of
vapour pressure of a liquid is equal to the A. diffraction of light
external atmospheric pressure is known as B. polarization of light
its C. the difference in speed of the components
A. dew point of light
B. boiling point D. the interference of light waves in glass
C. lower fixed point
D. triple point 32. In which of the following media does sound
travel faster?
26. A periodic pulse travels a distance of 20.0m A. Water
in 1.00s. If the frequency is 2.0 x 102 Hz. B. Brass
Calculate the wave length C. Air
A. 1.0 x 10-3m D. Wood
B. 1.0 x 10-2m
C. 2.0 x 10-2m 33. A tuning fork off frequency 600Hz is
D. 1.0 x 102m sounded over a closed resonance tube. If
the first and second resonant positions are
27. A wave has an amplitude equal to 4.0m, 0.130m and 0.413m respectively, calculate
angular speed 1/3 π rad. The displacement the speed of sound in air.
y of the wave particle is given as A. 509.4ms-1
A. y = 4 sin π /3 (t+2) B. 480.0ms-1
B. y = 4 sin π /3 (t+2/3) C. 339.6ms-1
y = 4 sin π /3 (2t + 1)
D. 169.8ms-1
C.
D. y = 4 sin π /3 (t+2) 34. Loudness increases as a vibrating tuning
fork is brought=nearer the end of a pipe
28. In the diagram below, an incident ray AY containing air column due to
makes an angle anticlockwise about Y A. resonance
through an angle of 200, while AY is fixed, B. echo
what angle will the reflected ray now make C. reverberation
with the incident ray? D. difraction
A. 70o
35. The factor which enables the ear to B. 60.0 minutes
distinguish between a note played on C. 52.5 minutes
different instruments is the D. 10.5 minutes
A. pitch
B. sound 41. The region around magnet in which the
C. harmonics magnetic influence is experienced is called
D. loudness A. magnetic flux
B. magnetic field
36. An object is situated within the earth’s C. magnetic meridian
gravitational field. Which of the following D. magnetic declination
factors does not affect the acceleration of
free fall g? 42. If a bar magnet is accidentally broken into
A. The distance of the object from the centre three pieces as shown in the diagram below
of the Earth the polarities of P and Q respectively are
B. The Latitude of the Earth on which the
object is situated
C. the mass of the object
D. The rotation of the Earth A. S and S
B. N and S
37. The capacitance of a parallel plate capacitor C. N and N
is increased by making the area of the D. S and N
plates
A. small and their separation large 43. A device used to prevent wearing away of
B. large and their separation small the make-and-break contacts of an
C. and their separation small induction coil is called a/an
D. and their separation equal A. fuse
B. electroscope
Use the diagram below to answer Questions C. resistor
38 and 39 D. capacitor

44. An inductor of inductance 10H carries a


current of 0.2A. Calculate the energy stored
in the inductor.
A. 0.11
38. In the diagram above the current passing B. 0.2J
through the 6Ω resistor is 1.5A. Calculate C. 1.10J
the current in the 3Ω resistor. D. 2.0J
A. 1.30A
B. 0.90A 45. Calculate the inductance of an inductor
C. 0.75A whose reactance is one ohm at 50 Hz
D. 0.60A A. 2.00 x 10-2H
B. 6.36 x 10-3H
39. Calculate the terminal p.d of the battery in C. 3.18 x 10-3H
the diagram D. 4.55 x 10-4H
A. 7.50V
B. 9.00V 46. Bohr’s theory provides evidence for the
C. 10.80V A. structure of the atom
D. 11.25V B. positive charge of an election
C. existence of energy level in the atom
40. An immersion heater is rated 120W. How D. positive charge on a proton
long does it take the heater to raise the
temperature of 1.2kg water by 15oC. 47. A nuclide20284Y emits in succession an α –
[Assume heat lost to the surrounding is particle and β -particle. The atomic number
negligible. Specific heat capacity of water = of the resulting nuclide is
4200 J Kg-1 K-1] A. 198
A. 65.0 minutes B. 83
C. 82
D. 80

48. The half-life of radioactive substance is 14


days. If 48g of this substance is stored, after
how many days will 1.5g of the original
substance remain?
A. 84 days
B. 70 days
C. 56 days
D. 40 days

49. A material of mass 1.0 x 10-3kg undergoes a


fission process which decreases its mass by
0.02 percent. Calculate the amount of
energy released in the process. [c = 3.0 x
108 ms-1]
A. 1.8 x 1020J
B. 1.8 x 1013J
C. 1.8 x 1011J
D. 1.8 x 1010J

50. Absorption line spectra exhibited by atoms


is a result of
A. change in the kinetic energy of a moving
atom
B. instability of the nucleons
C. excitation of an electron in the atom
D. transition of an electron from a higher to a
lower energy level
10. An electron of mass 9.1 x 10-41kg moves
with a velocity of 4.2 x 107ms-1 between the
cathode and anode of an x-ray tube.
Calculate the wavelength. [Take plank-s
constant h = 6.6 x 1034 Js]

PART II

2000 11. (a) (i) Define the term linear


SECTION B THEORY PART I momentum
(ii) State the law of conservation
1. A body of mass 0.6kg is thrown vertically of linear momentum
upward from the ground with a speed of (b) A ball P of mass 0.25kg losses one-
20ms-1. Calculate its third of its velocity when it makes a
(i) potential energy at the maximum head on collision with an identical
height reached ball Q at rest. After the collision, Q
(ii) kinetic energy just before it hits the moves off with a speed of 2 ms-1 in
ground the original direction of P. Calculate
the initial velocity of P.
2. What is a projectile?
(a) Define the term surface tension (c) (i) State Newton’s second law of
(b) Calculate the force required to lift a motion
needle 4cm long off the surface (ii) Show that F = ma where F is
tension of water is 7.3 x 10-2 Nm-1 the magnitude of the force
acting on a body of mass m to
4. State any three properties of matter which give it an acceleration of
are common to all substances magnitude a.
(iii) the engine of a vehicle moves
5. Define (i) Elasticity (ii) Young’s modulus it forward with a force of 9600
(iii) Force constant N against a resistive force of
2200 N. If the mass of the
6. A force of 40 N is applied at the end of a vehicle 3400kg, calculate the
wire 4m long and produces an extension of acceleration produced
0.24mm. If the diameter of the wire is
2.00mm, calculate the 12. (a) (i) Mention two modes of heat
(i) stress on the wire (ii) strain in the wire transfer other than
convection.
7. (a) Differentiate between interference (ii) Explain land and sea breezes.
and (b) An iron rod of length 30cm is heated
polarization as applied to waves. through 50 kelvin. Calculate its
(b) Mention two uses of polaroids increase in length.
[Linear expansivity of iron = 1.2 x
8. (a) Define angle of contact 105 k1J
(b) Draw sketches to show angles of (c) An electric heater immersed in some
contact water raises the temperature of the
for a capillary tube dipped vertically water from 40oC to 100oC in 6
in (i) water (ii) mercury minutes. After another 25 minutes, it
is noticed that half the water had
9. During the electrolysis of copper (II) boiled away. Neglecting heat losses
teraoxosulphate (VI) solutions, a steady to the surrounding. Calculate the
current of 4.0 x 102 A flowing for one hour specific latent heat of vaporization of
liberated 0.48g of copper. Calculate the water.
mass of copper liberated by one coulomb of
charge 13. (a) (i) State the laws of refraction of
light
(ii) Describe an experiment to which take place during the
determine the refractive operation of a modern x-ray
index, n, of the material of an tube.
equilateral triangular glass (ii) Distinguish between hard and
prism using the minimum soft x-rays.
deviation method. (iii) State three uses of x-rays
(b) A rectangular glass prism of (iv) Mention one hazard of over
thickness 12cm is placed on a mark exposure to x-rays in a
on a piece of paper resting on a radiological laboratory, and
horizontal bench; indicate any two safety
(i) Draw a ray diagram to show precautions.
the apparent position of the (b) A possible fusion reaction is
mark in the glass prism. 2H + 2H → 3H + 1H + Q
(ii) If the refractive index of the 1 1 1 1
material of the prism is 1.5, where Q is the energy released as a
calculate the apparent result of the reaction.
displacement of the mark.

14. (a) Define the following terms:


(i) Electric field intensity; (ii) Electric If Q = 4.03 Me V, calculate the
potential atomic mass of 3H in atomic mass
(b) The diagram below illustrates two units.
collinear electric charges of 1
magnitudes + Q and – Q. The
[2H = 2.01410 U; 1H = 1.00783 U;
charges are equidistant from a point
P at which a rest charge is placed. 1 1
1U = 931 MeV]

Copy the diagram and use arrows to


indicate, from the point P, the
direction of the (i) electric force F1
due to + Q (ii) electric force f2 due to
– Q; (iii) electric field intensity E
(c) (i) What is meant by dielectric
substance?
(ii) List the factors which
determine the capacitance of
a parallel plate capacitor and
state the effect each of them
has on the capacitance.
(iii) The diagram below
represents a section of a
circuit. Calculate the effective
capacitance in the section.

15. (a) (i) State the energy


transformations
5. The apparent weight of a body fully
immersed in water is 32N and its weight in
air is 96 N. Calculate the volume of the body
[Density of water = 1000 kg m-3, g = 10ms-
2
]
A. 8.9 x 10-3m3
B. 6.4 x 10-3m3
C. 3.2 x 10-3m3
D. 3.0 x 10-3m3

6. A block weighing 15 N rests on a flat surface


and a horizontal force of 3 N is exerted on it.
Determine the frictional force on the block.
A. 0.3 N
B. 0.5 N
C. 3.0 N
D. 5.0 N
2001
SSCE PHYSICS 7. which of the following equations represents
PAPER 2 OBJECTIVE the distance x, traveled by a body moving
on a straight road with a constant speed?
1. Which of the following physical processes [The other symbols have their usual
cannot be explained by the molecular meanings]
theory of matter. A. x = ut + ½ at2
A. Evaporation B. x = ut
B. Thermal conduction C. x = v2 – u2
C. Radiation of heat 2a
D. Convectional currents in fluids D. x = ½ at2

2. A spring balance, which is suspended from 8. The amplitude of the motion of a body
the roof of a lift carries a mass of 1 kg at its performing simple harmonic motion
free end. If the lift acceleration upwards at decreases with time because
2.5ms-2, determine the reading on the A. frictional forces dissipate the energy of
spring balance. [g = 10ms-2] motion
A. 25.0 N B. the frequency of oscillation varies with time
B. 12.5 N C. the period of oscillation varies with time
C. 7.5 N D. energy is supplied by some external
D. 4.0 N agencies

3. An object of weight 10N immersed in a 9. The amplitude of a particle executing simple


liquid displaces a quantity of the liquid. If harmonic motion is 5cm while its angular
the liquid displaced weighs 6N, determine frequency is 10 rad S-1. Calculate the
the upthrust on the object magnitude of the maximum acceleration of
A. 20 N the particle
B. 10 N A. 0.25m s-2
C. 6N B. 0.50m s-2
D. 4N C. 2.00m s-2
D. 5.00m s-2
4. Which of the following statements about
pressure is not correct? Pressure. 10. The time rate of change of momentum is
A. increases with an increase in surface area A. impulse
B. decreases with an increase in surface area B. force
C. increases with a decrease in surface area C. power
D. increases with an increase in the applied D. pressure
force
11. Which of the following statements about A. 21.0N
elastic collision is correct? B. 60.6oC
A. Momentum is lost due to the sound C. 80.0oC
produced D. 84.7oC
B. Loss in momentum is equal to loss in kinetic
energy 17. A given mass of an ideal gas has a pressure
C. Both kinetic energy and momentum are of 500 Nm-2 at –13oC. If its volume remains
conserved constant, calculate its pressure at 247oC.
D. Kinetic energy is lost while momentum is A. 25 N m-2
conserved B. 50 N m-2
C. 500 N m-2
12. Electrical resistance is a property of an D. 1000 N m-2
electrical conductor that causes electrical
energy to be converted into 18. The volume of heat given out or absorbed
A. mechanical energy when a substance changes its state at a
B. heat energy constant temperature is known as
C. magnetic energy A. 411.5cm-3
D. chemical energy B. 267.1cm-3
C. 254.2cm-3
13. A simple machine with an efficiency of 75% D. 112.7cm-3
lifts a load of 5000 N when a force of 500 N
is applied to it. Calculate the velocity ratio 19. The amount of heat given out or absorbed
of the machine. when a substance changes its state at a
A. 10.0 constant temperature is known as0
B. 13.3 A. latent heat
C. 17.5 B. heat capacity
D. 25.0 C. specific latent heat
D. specific heat capacity
14. A screw jack with a tommy bar of length
12cm is used to raise a car through a 20. Which of the following physical quantities
vertical height of 25cm by turning the affects the saturated vapour pressure of a
tommy bar through 50 revolutions. liquid?
Calculate the approximate velocity ratio of A. Temperature
the jack [ π = 3.14]. B. Volume
A. 21 C. Mass
B. 38 D. Density
C. 48
D. 151 21. Humidity is used to describe the amount of
A. air in water vapour
15. A beam PQ pivoted at P carries a load of 80 B. cloud available in the atmosphere
N as shown below. Calculate the effort, E, C. air in equal volumes of cloud
required to keep it horizontal. [Neglect the D. water vapour in the atmosphere
weight of beam]
22. Surface waves traveling in deep water at 15
ms-1 are incident at a shallow water
boundary. If the angles of incidence and
refraction are 45o and 30o respectively,
A. 32.0N calculate the speed of the waves in shallow
B. 26.7N water.
C. 16.0N A. 8.1 m s-1
D. 13.3N B. 10.0 m s-1
C. 10.6 m s-1
16. A resistance thermometer has a resistance D. 22.5 m s-1
of 20Ω at 0oC and 85Ω at 100oC. If its
resistance is 52Ω in a medium, calculate the 23. An incident ray is reflected normally by a
corresponding temperature plane mirror on to a screen where it forms a
bright spot. The mirror and screen are
parallel and 1 m apart. If the mirror is Use the information below to answer
rotated through 5o, calculate the Questions 29 and 31.
displacement of the spot
A. 8.7 cm
B. 10.0cm
C. 15.4cm
D. 17.6cm

24. Convex spherical mirrors are preferred to The diagram above shows the wire of a
plane mirrors as driving mirrors because sonometer box set into resonance by a
A. the image produced is upright and clearly vibrating tuning fork of frequency, f, when
visible the wire is stretched by a tension. T. The
B. it provides a wider field of view resonating length of the wire is l while the
C. the image produced is erect and diminished wave length of the sound produced is λ .
D. the image produced is not laterally inverted
29. Which of the following statements is not
25. An object is placed 20cm from a lens. If an correct? The wave produced on the wire is
image is formed on a screen 260cm away A. stationary and longitudinal
from the lens, calculate the magnification of B. stationary and mechanical
the image. C. mechanical and longitudinal
A. 28 D. progressive and transverse
B. 26
C. 24 30. Which of the following statements about the
D. 13 frequency of vibration of the wire is not
correct? It is
26. A piece of cloth appears green in sunlight. A. proportional to the mass per unit length of
When held in red light, it will appear the wire
A. green B. equal to the frequency of the tuning fork
B. blue C. inversely proportional to the length of the
C. red wire
D. black D. proportional to the square root of the
tension
27. dispersion of white light by a glass prism
occurs because 31. If the frequency of the tuning fork at
A. white light consists of a mixture of seven resonance is 256 Hz, calculate the
different colours frequency of vibration of the wire when the
B. the refractive index of glass is different for tension in the wire is increased to 196N
each constituent colour of white light A. 130.6 Hz
C. the speed of each colour of light in the glass B. 182.9 Hz
is proportional to the refractive index of C. 358.4 Hz
glass for each colour D. 501.8 Hz
D. the speed of each colour of light in the glass
is proportional to the angle of retraction in 32. the magnitude of the gravitational force
the prism between two particles 0.10m apart is 10N. If
the distance between them is increased to
28. Which of the following statements about 0.20m, calculate the magnitude of the new
sound is correct? force.
A. When sound travels from air to a denser A. 40.0 N
medium, it is refracted away from the B. 5.0 N
normal at the point of incidence C. 20.0 N
B. Sound travels faster in air than in water D. 2.5 N
C. The wavelength of sound changes when it
travels from air to water 33. As the plates of a charge variable capacitor
D. The frequency of sound changes when it are moved closer together, the potential
travels from air to water difference between them
A. increases C. The presence of magnetic lines of force in a
B. decreases region indicates the presence of a magnetic
C. remains the same field
D. is doubled D. Magnetic lines of force are closely-packed
together at neutral points
Use the diagram shown beside to answer
Questions 34 and 35 40. The material used for constructing the core
of an electromagnet is
34. What is the effective capacitance in the A. iron
circuit B. carbon
A. 2µF C. copper
B. 6F D. steel
C. 18µF
D. 216µF 41. In the circuit diagram below, calculate the
energy stored in the inductor at resonance
35. What is the total energy stored by the
capacitors A. 0.100 J
A. 2.0 x 10-4J B. 0.05 J
B. 1.0 x 10-4J C. 0.010 J
C. 9.0 x 10-2J D. 0.005 J
D. 1.0 x 10-2J
42. Calculate the reactance of the inductor in
36. Three identical lamps each of power 100W, the circuit diagram shown [ π = 22/7]
are connected in parallel across a potential A. 132Ω
difference of 250V: calculate the current in B. 214Ω
the circuit. C. 264Ω
A. 7.5 A D. 269Ω
B. 2.5 A
C. 1.2 A 43. In which of the following transitions is the
D. 0.8 A largest quantum of energy liberated by an
hydrogen atom when the electron changes
37. An electric bulb is rated 60W, 220V. energy levels?[ n is the quantum number]
Calculate the resistance of its filament when A. n = 2 to n = 1
it is operating normally. B. n = 1 to n = 2
A. 296.7Ω C. n = 2 to n =3
B. 400.0Ω D. n = 3 to n = 2
C. 512.2Ω
D. 806.7Ω 44. A photon of wavelength λ 0 is emitted when
an electron in an atom makes a transition
38. The Earth’s magnetic equator passes atom a level of energy 2Ek to that of energy
through Jos in Nigeria. At Jos, the Ek. If the electron transits from 2Ek to Ek
A. angle of variation is zero level, determine the wavelength of the
B. magnetic declination is 90o photon that would be emitted
C. angle of dip is zero
A. 3 λo
D. horizontal component of the Earth’s
magnetic field is zero B. 2 λo
C. ¾ λo
39. Which of the following statements about D. 1
/3 λ o
magnetic lines of force is not correct?
A. A magnetic line of force is an imaginary line 45. Production of x-rays in an x-ry tube begins
which the north pole of a magnet would with
describe if it is free to move A. photo electric emission
B. Magnetic lines of force do not cross one B. collision of electrons
another C. thermionic emission
D. field emission of electrons
46. The nucleon number and the proton number
of a neutral atom of an element are 23 and
11 respectively. How many neutrons are
present in the atom?
A. 11
B. 12
C. 23
D. 34

47. A nitrogen nucleus bombarded with an


alpha particle produces an oxygen nucleus
and a proton. The nuclear reaction for this
process is
4He + 14N → 17O + 1H + Q
2 7 8 1
Which of the following statements about the
reaction is not correct?
A. the bombardment results into a nucleus
with greater proton number
B. It is an induced nuclear reaction
C. It is a natural radioactive decay
D. The sum of the initial nucleon numbers is
equal to the sum of the final nucleon
numbers

48. In a nuclear reactor, electricity can be


generated through the following processes.
Arrange the processes in the correct order.
I. The steam is used to drive turbines
II. The heat energy released is removed by
passing water through the reactor
III. The turbines in turn generate electricity
IV. The water then passes through some form
of heat exchanger to produce steam
A. II, IV, I and III
B. I, II, III and IV
C. III, I, IV and II
D. IV, I, III and II

49. Electrons passing through crystals are


diffracted because they
A. are repelled by the atoms in the crystal
B. are attracted by the atoms in the crystal
C. possess wave properties
D. are particles

50. The duality of matter implies that matter


A. exists as a particle of dual composition
B. has momentum and energy
C. has both wave and particle properties
D. is made up of dual materials
4. The value of the e.m.f of a voltaic cell which
has dilute tetraoxosulphate (VI) acid as its
electrolyte and copper and zinc as its
electrode becomes less with use, “Explain
this observation and state how it can be
corrected.

5. (a) What is meant by a beam of


palarised
light?
(b) with the aid of well labeled diagrams,
illustrate the action of a Polaroid
spectacle on a beam of sunlight.

6. (a) Explain how a gas can be made to


Conduct electricity
(b) name the electric charge carriers in
gases.

7. (a) State two properties of cathode rays


(b) Explain how the intensity and energy
of cathode rays may be increased

8. The mass and wavelength of a moving


electron are 9.0 x 10-31kg and 1.0 x 10-10m
2001 respectively. Calculate the kinetic energy of
SECTION B THEORY the electron
[h = 6.6 x 10-34 J s]
1. A ball thrown vertically upward reaches a
maximum height of 50 m above the level of
projection. Calculate the 9. The diagram below represents the graph of
(i) time taken to reach the electron energy against the frequency of
maximum height the radiation incident on a metal surface.
(ii) speed of the throw [g = 10 Interpret the
ms-2] (i) slope of the graph;
(ii) intercept, OC;
2. A lead shot is projected from the ground (iii) intercept, OK.
level with a velocity u at an angle θ to the
horizontal. Given the time, for the lead shot
to reach its maximum height as t = u2 sin2
θ
g where “g” is the
acceleration of free fall due to gravity, show
that the greatest height reached by the 10. (a) distinguish between stress and strain
as
body is h max = u2 sin2 θ
used in elasticity
2g (b) when a force of 40N is applied to the
free
3. (a) What is electrolysis? end of an elastic cord, an extension
(b) A current of 2A is passed through a of 5cm is produced in the cord.
copper volta-meter for 5 minutes. If Calculate the work done on the cord.
the electrochemical equivalent of
copper is 3.27 x 10-7 kg C-1, PART II
determine the mass of the copper
deposited. 11. (a) In his first attempt, a long jumper
took off
From the spring board with a speed (b) (i) Explain timbre and overtones
of 8 ms-1 at 30o to the horizontal. He (ii) What is reasonance?
makes a second attempt with the (c) As a ship approaches a cliff, its siren
same speed at 45o to the horizontal. is sounded and echo was heard 8
Given that the expression for the seconds later. If the speed of sound
horizontal range of a project tile is v2 in air is 340 ms-1, calculate the
sin 0 velocity at which the ship was
g where all the approaching the cliff.
symbols have their usual meanings,
show that he gains a distance of 14. (a) Explain the statement ‘the
0.8576 m in his second attempt capacitance of
(b) (i) State Hooke’s law of elasticity a capacitor is 5µF’
(ii) Describe an experiment to (b) (i) State the factors upon which
verify Hooke’s law the
(iii) State two precautions you capacitance of a parallel plate
would take if you were to capacitor depend.
perform this experiment in (ii) State how the capacitance
the laboratory depends on each of these
(c) A spiral spring of natural length factors state in (b)(i)
20.00cm has a scale pan hanging (d) A series arrangement of three
freely in its lower end. When an capacitors of values 8µF, 12µF and
object of mass 40g is placed in the 24µF is connected in series with a
pan, its length becomes 21.80cm. 90-V battery.
When another object of mass 60g is
placed in the pan, the length
(iii) Draw an open-circuit diagram
for this arrangement.
becomes 22.05cm. Calculate the
mass of the scale pan. [g = 10 ms-1] (iv) Determine the p.d across 8µF
capacitor.
12. (a) Define specific heat capacity.
(b) (i) With the aid of a labeled 15. (a) When nitrogen (atomic mass = 14,
diagram, atomic
Describe an experiment to Number = 7) is bombarded with
determine the specific heat neutrons, the collisions result in
capacity of copper using a disintegrations in which alpha
copper ball. particles are produced. Represent
this transmutation in a symbolic
(ii) State two precautions
equation.
necessary to obtain accurate
(b) (i) How does a radioactive atom
results.
differ from a stable one?
(c) A piece of copper block of mass 24g
at 230oC is placed in a copper (ii) Explain ‘half life’
calorimeter of mass 60g containing (iii) A sample of radioactive
54g of water at 31oC. Assuming heat material has a half life of 35
losses are negligible, calculate the days. Calculate the fraction of
final steady temperature of the the original quantity that will
mixture. remain after 105 days.
[specific heat capacity of water = (c) Light of wavelength 5.00 x 10-7m is
4200J kg-1 K-1] incident on a material of work
[specific heat capacity of copper = function 1.90 eV. Calculate the
400 J kg-1 K-1] (i) photon energy
(ii) kinetic energy of the most
13. (a) (i) What is an echo? energetic photo electron.
(ii) State two useful applications (iv) Stopping potential
of echoes [plancks constant h = 6.6x1-34
(iii) Why are the walls, floors and Js]
ceilings of a recording studio [c = 3.0 x 108 ms-2, JeV = 1.6
heavily padded? x 1019 J]
10.0cm from the 0 cm mark
of the meter rule.
(iii) Balance the whole
arrangement horizontally on a
knife edge as shown in the
diagram above.
(iv) Measure and record the
distance, U, of K from the
0cm mark of the metre rule.
(v) Repeat the procedure for five
other values of V = 15.0;
20.0, 25.0, 30.0 and 35.0cm.
(vi) In each case, measure and
record the corresponding
values of U. Tabulate your
readings.
(vii) Plot a graph of U on the
vertical axis against V on the
horizontal axis.
(viii) Determine the:
(i) slope; s, of the graph
(ii) intercept c, on the
vertical axis.
(ix) Evaluate (i) K1 = I – 2s 100
s

(ii) K2 = 26
/s – 160.

(x) State two precautions taken


to ensure accurate results.

(b) (i) State two conditions under


which
a rigid body at rest remains in
equilibrium when acted upon
by three non-parallel coplanar
forces.
(ii) Explain how the position of
the centre of gravity of a
body affects the equilibrium
of the body.
2001
PAPER I PRACTICAL 2. (a) Using the diagram beside
as a guide, carry out
1. (a) the following instructions
(i) Place the pin, O, horizontally
inside the cylinder provided.
Pour some water on the pin in
the cylinder such that the
(i) Fix the 100g mass, marked P, length of the water column, l
at B, the 80cm mark of the = SO = 10.0cm, where S
uniform metre rule, using an represents the water
adhesive. meniscus.
(ii) Suspend another 100g mass
marked Q at A, a distance V –
(ii) Insert another pin, P, in the (v) Repeat the procedure for four
cork held by the boss of the other values of L = 90, 80, 70
retort stand. and 60cm. In each case,
(iii) Adjust the position of P obtain and record the values
vertically upward or of ls and lp and evaluate R1 =
downward until it coincides (lp/h)Rs.
with the image I of O formed (vi) Repeat the experiment with
by refraction at S. the second wire, Q. Obtain
(iv) Read and record the distance values of Is and Ig for equal
h = PO lengths of wire as used in
(v) Repeat the procedure for four wire P.
other values of l= 15, 20, 25
and 30 cm
(vii) Evaluate R2 = l0 Rs in each
case.
(vi) In each case measure and
ls
record the corresponding
Tabulate your readings.
value of h. Tabulate your
readings. (viii) Plot a graph of R2 on the
(vii) Plot a graph of h on the vertical axis against R1 on the
vertical axis against l on the horizontal axis.
horizontal axis. (ix) Determine the slope, s, of the
(viii) Determine the slope, s, of the graph.
graph. (x) Evaluate k = s
(xi) State two precautions taken
(b) Explain total internal reflection of
to ensure accurate results.
light.
(b) (i) Define resistivity of the
(ii) A rectangular glass prism of
material of
thickness 6cm and refractive
a wire.
index 1.5 is placed on the
(ii) A galvanometer with a full-
page of a book. The prints on
scale-deflection of 1.5 x 10-3.
the book are viewed vertically
A has a resistance of 50Ω.
down wards from above.
Determine the resistance
Determine the apparent
required to convert it into a
upward displacement of the
voltmeter, reading up to 1.5V.
prints.

3. (a) You are provided with two wires


marked P
and Q, a resistor Rs = 1Ω and other
necessary apparatus.

(i) connect Rs in the left hand


gap of the metre bridge, a
length L = 100cm of wire P in
the right hand gap and the
other apparatus as shown in
the diagram above.
(ii) Determine the balance point,
B, on the bridge wire AC
(iii) Measure and record AB = ls
and BC = lp
(iv) Evaluate R1 = (lp/h)Rs.
PAPER 2 OBJECTIVE

1. One common characteristics of solids,


liquids and gases is that
A. all the three have fixed volume
B. their molecules have the same size
C. their molecules are always in motion
D. all the three have the same inter molecular
forces

2. The force between the molecules of a liquid


in contact with that of a solid is
A. adhesive
B. cohesive
C. magnetic
D. repulsive

3. The energy needed to separate the


nucleons against the forces which hold them
together is
A. nuclear energy
B. mechanical energy
C. ionization energy
D. binding energy

4. A reservoir is filled with a liquid of density


2000 kgm-1. Calculates the depth at which
the pressure in the liquid will be equal to
9100Nm-2. [g=10ms-2]
A. 0.262m
B. 0.455m
C. 0.664m
D. 0.819m

5. A solid weighs 45 N and 15 N respectively in


air and water. Determine the relative
density of the solid.
A. 0.33
B. 0.50
C. 1.50
D. 3.00

6. A boy travels 8km eastward to a point B and


then 6km northward to another point C.
Determine the difference between the
magnitude of displacement of the boy and
distance traveled by him.
A. 2.0km
B. 4.0km
C. 10.0km
D. 14.0km

2002
SSCE PHYSICS
7. The magnitude of the force required to C. All the parallel forces must be equal in
make an object of mass, m move with magnitude and direction
speed, v in a circular path of radius, r is D. The sum of the forces in one direction must
given by the expression. be equal to the sum of the forces in
A. mr opposite direction
v
12. An object, A, is held in equilibrium as
B. mr2 illustrated in the diagram below. Using the
v data on the diagram, determine the
magnitude, W, of the weight of A.
C. mv2
r

D. mv
R2

8. A man was moving round a circular path A. 23 N


continuously for 4 minutes and covered the B. 17 N
following distances in the times stated C. 7N
below: D. 5N
Distance (in metres); 200 400 600
800 13. A spiral spring of natural length 30.0cm and
Time (in minutes); 1 2 3 4 force constant of 20 Nm-1 is compressed to
Which of the following statements is correct 20.0cm. Calculate the energy stored in the
about the motion of the man? spring.
A. The man was moving with uniform speed A. 0.1 J
but non-uniform velocity B. 1.0 J
B. The speed was changing constantly C. 10.0 J
C. The velocity was uniform D. 100.0 J
D. It was an instance of motion in which both
speed and velocity are the same 14. In the diagram below, AB represents a
uniform rod of length 1.50m which is in
9. A car takes off from rest and covers a equilibrium on a pivot at P. If AP = 40cm,
distance of 80m on a straight road in 10s. calculate the mass of the rod. [g = 10 ms-2]
Calculate the magnitude of its acceleration. A. -.280 kg
A. 1.25 ms-2 B. 0.400 kg
B. 1.60 ms-2 C. 0.613 kg
C. 4.00 ms-2 D. 0.800 kg
D. 8.00 ms-2
15. Which of the following statements correctly
10. An object is released from rest at a height of defines quantitatively, the work done by a
25m. Calculate the time it takes to fall to force? The product of the force and the
the ground. [g = 10 ms-2] distance moved.
A. 25.00s A. parallel to the line of action of the force
B. 10.00s from a reference point
C. 2.50s B. perpendicular to the line of action of the
D. 2.24s force from a reference point
C. on a line through the reference point
11. An object is acted upon by a system of D. by the object
parallel forces causing the object to be in
static equilibrium. Which of the following 16. A load of mass 120kg is raised vertically
statements is not correct? through a height of 2m in 20s by a machine
A. The resultant of the parallel forces is zero whose efficiency is 100%. Calculate the
B. The algebraic sum of all the moments of the power generated by the machine. [g =
forces is zero 10ms-2]
A. 60 W
B. 80 W
C. 100 W 23. What type of motion does the skin of a
D. 120 W ‘talking’ drum perform when it is struck with
17. A block and tackle system has 6 pulleys. If a drum stick?
the efficiency of the machine is 60%, A. Random
determine its mechanical advantage. B. Rotational
A. 12.0 C. Translational
B. 10.0 D. Vibratory
C. 3.6
D. 1.8 24. A slinky spring fixed at one end is placed
horizontally on a table. The free end is
18. Which of the following actions will improve displaced parallel to the table and then
the efficiency of a pulley system? released. The resulting wave form is
A. Reducing the mass of the pulley A. transverse
B. Increasing the frictional force between the B. longitudinal
string and the pulley C. stationary
C. Increasing the mass per unit length of the D. electromagnetic
string of the pulley
D. Increasing the mass of the pulley 25. A radio wave has a wavelength of 150m. If
the velocity of radio waves in free space is
19. The ice and steam points of a certain 3x108ms-1, calculate the frequency of the
thermometer are 20o and 100o respectively. radio wave.
Calculate the Celsius temperature A. 4.5 x 1010Hz
corresponding to 70o on the thermometers. B. 5.0 x 109Hz
A. 84.0oC C. 4.5 x 109Hz
B. 75.0oC D. 2.0 x 106Hz
C. 64.0oC
D. 58.0oC 26. In the wave equation y=E0sin(200t – π x), E0
represents the
20. the silver coating on the inside of a vacuum A. amplitude
flask reduces heat loss by B. frequency
A. conduction C. period
B. radiation D. wavelength
C. condensation
D. convection 27. The size of the hole of a pin hole camera is
increased. The resultant image formed
21. Water of mass 120g at 50oC is add to 200g becomes
of water at 10oC and the mixture is well A. sharper
stirred. Calculate the temperature of the B. reduced in size
mixture. C. enlarged
[Neglect heat losses to the surrounding] D. blurred
A. 60oC
B. 40oC 28. The refractive index of a medium relative to
C. 25oC air 1s 1.8, calculate, to the nearest degree,
D. 10oC the critical angle for the medium.
A. 68º
22. Calculate the energy required to vaporize B. 56º
50g of water initially at 80oC.
C. 34º
[specific heat capacity of water = 4.2-1 k-1]
[specific latent heat of vaporization of water D. 18º
= 2260 Jg-1]
A. 113,000J 29. A converging lens of focal length 15cm
B. 117,200J forms a virtual image at a point 10cm from
C. 234,400J the lens. Calculate the distance of the
D. 420,000J object from the lens.
A. 10.00cm
B. 6.00cm
C. 5.00cm 36. Three capacitors each of capacitance 18µF
D. 1.50cm are connected in series. Calculate the
effective capacitance of the capacitors.
30. In a compound microscope, the image A. 54.00 µF
formed by the objective lens is at a distance B. 18.00 µF
of 3.0cm from the eye lens. If the final C. 6.00 µF
image is at 25.0cm from the eye lens, D. 0.17 µF
calculate the focal length of the eyes lens.
A. 0.3cm 37. Two spheres of masses 100kg and 90kg
B. 2.7cm respectively have their centres separated
C. 3.4cm by a distance of 1.0m. Calculate the
D. 8.3cm magnitude of the force of attraction
between them. [G =6.70x10-11 Nm2 kg-2]
31. A pulse of a sound is transmitted from a A. 6.70 x 10-11 N
ship and the reflection from the sea bed is B. 6.70 x 10-10 N
recorded after 0.2s. Calculate the depth of C. 6.03 x 10-10 N
the sea. D. 6.03 x 10-7 N
A. 312.0m
B. 156.0m 38. The magnitude of the gravitational
C. 78.0m attraction between the earth and a particle
D. 31.2m is 40 N. If the mass of the particle is 4kg,
calculate the magnitude of the gravitational
32. When two notes of nearly equal frequencies field intensity of the earth on the particle.
are sounded together, a periodic rise and A. 10.0 N kg-1
fall in sound intensity can be heard. This B. 12.6 N kg-1
phenomenon is called C. 25.0 N kg-1
A. beats D. 160.0 N kg-1
B. diffraction
C. overtones 39. Local action is normally prevented in a
D. resonance simple electric cell by
A. introducing an oxidizing agent in the acid
33. Which of the following instruments is not a B. not drawing current continuously for long
wind instrument? periods
A. Clarinet C. amalgamating the surface of the zinc plate
B. Saxophone D. using a porous pot to isolate the zinc from
C. Trumpet the acid
D. Xylophone
40. The lead-acid accumulator consists of
34. Which of the following actions will not A. lead as the positive electrode
cause an increase in the frequency of a B. lead oxide as the negative electrode
vibrating string? C. hydrochloric acid as the electrolyte
A. increasing the tension in the string D. tetraoxosulphate (VI) acid as the electrolyte
B. Decreasing the mass per unit length of the
string 41. Which of the following actions will not led
C. Decreasing the length of the string to an increase in the induced e.m.f. in a coil
D. Increasing the cross sectional area of the of wire rotating between the poles of a
string magnet? Increasing the
A. strength of the magnet
35. Which of the following items can be used to B. number of turns in the coil
compare the relative magnitudes of electric C. speed of rotation of the coil
charges on two bodies? D. gap between the poles of the magnet
A. Ebonite rod
B. Gold leaf electroscope 42. A transformer has 400 turns and 200 turns
C. Proof planes in the primary and secondary windings
D. The electrophorus respectively. If the current in the primary
and secondary windings are 3A and 5A
respectively, calculate the efficiency of the B. frequency of the light
transformer. C. intensity of the light
A. 85.0% D. time of exposure of the metallic to light
B. 83.3%
C. 37.5% 48. Which of the following materials is used to
D. 30.0% control the rate of neutron production in a
43. A freely-suspended magnet swinging in a nuclear reactor?
horizontal plane comes to rest with its axis A. Boron rods
pointing approximately. B. Concrete shield
A. north – south C. Graphite rods
B. south – west D. Uranium rods
C. north – east
D. south – east 49. Which of the following is usually used to
cause fission in an atomic reactor?
44. Calculate the inductance, L, of the coil in the A. Alpha particles
circuit diagram shown below. B. Beta particles
C. Electrons
D. Neutrons

50. A nuclide X is produced by bombarding a


nitrogen (N) nucleus with an alpha ( α )
A. 3.8 H particle with the release of heavy hydrogen
B. 0.6 H (D) nucleus as shown by the following
C. 0.4 H nuclear equation.
D. 0.2 H α + 14N → Px + 2D
7 q 1
45. In a model of the hydrogen atom, the Determine the values of p and q in the
energy levels Wn are given by the formula equation.
Wn = –R A. 12 and 6
n2 B. 14 and 7
Where n is an integer and R is a constant. C. 16 and 8
Determine the energy released in the D. 19 and 8
transition from n = 3 t0 n = 2.
A. R
5

B. –R
4

C. 5R
36

D. –5R
36

46. If a radioactive atom emits a beta particle,


its mass number
A. increases by one
B. remains the same
C. decreases by one
D. increases by two

47. When light falls on a metallic surface, the


number of electrons that may be emitted
would depend solely on the
A. area of the metallic surface
2002
SECTION B THEORY PART I

1. Explain The rise of water in a glass capillary


tube the kinetic theory.

2. (a) What is diffusion?


(b) State two factors which affect the
rate of diffusion.

3. (a) Explain elasticity of a substance.


(b) Define elastic constant.

4. A wire is gradually stretched until it snaps.


Sketch a load-extension graph for the wire
and on the graph indicate the
(a) elastic limit;
(b) yield point;
(c) maximum load;
(d) breaking point.

5. (a) Define horizontal range of a


projectile.
(b) The horizontal range, R, of a
projectile is given by the equation R
= u2 sin 2Θ
g
(i) What does Θ in the equation
represent?
(ii) For what value of Θ would R
be maximum?

6. Explain the action of direct electric current


in an electrolytic process with a dilute
solution of sodium chloride as electrolyte.

7. (a) What is meant by


(i) anode;
(ii) cathode; as used in
electrolysis?
(b) List the ions involved in an
electrolytic process with dilute
tetraoxoosulphate (VI) acid as
electrolyte.

8. (a) Explain how neon sings can be


produced.
(b) State two factors on which the colour
of light from a fluorescent tube
depend.

9. (a) State Heisenberg’s uncertainty


principle.
(b) Mention two phenomena that can Connected to a 220V source
only be explained in term of the is used to boil a certain
particulate nature of light. quantity of water in a
container of heat capacity
10. Explain why a direct electric current is used 100 J kg-1 for 2 minutes. If the
instead of an alternating current in initial temperature of the
electroplating. water is 40oC, calculate the
mass of the water in the
container.
PART II [specific heat capacity of
water = 4.2 x 103 J kg-1 K-1]
11. (a) (i) What is meant by a machine? [assume boiling point of water
(ii) List two examples of a simple = 100oC
machine.
(iii) Explain the statement the 13. (a) Define the following terms as applied
velocity ratio of a machine is to a
5, convex mirror:
(b) (i) Define the efficiency of a (i) principal focus;
machine (ii) pole;
(ii) Explain why a machine can (iii) radius of curvature.
never be 100% efficient. (b) State one advantage and one
(c) A screw jack, 25% efficient and disadvantage of using a convex
having a screw of pitch 0.4cm is mirror as a driving mirror
used to raise a load through a (c) Draw a clearly labeled diagram to
certain height. If in the process, the illustrate how two converging lenses
handle turns through a circle of may be arranged to form a
radius 40.0cm, calculate the compound microscope.
(i) velocity ratio of the machine; (d) An object 2.5mm long is viewed
(ii) mechanical advantage of the through a converging lens of focal
Machine; length 10.0cm held close to the eye.
(iii) effort required to raise a load A magnified image of the object is
of 1000 N with the machine. formed 30.0cm from the lens.
[Take π = 3.14]. Calculate the:
(i) distance of the object from
12. (a) (i) Define relative density. the
(ii) List three characteristics of Lens;
pressure in a liquid (ii) size of the image;
(b) The horizontal door of a submarine (iii) power of the lens.
at a depth of 500m has an area of
0.4m-2. Calculate the force exerted 14. (a) Define:
by the sea water on the door at this (i) reactance;
depth. (ii) impedance.
[Relative density of sea water = (b) (i) Explain reasonant frequency
1.03] of an
[Atmospheric pressure = 1.01x105 RLC circuit.
Nm-2] (ii) Explain the statement the
[Density of pure water = 1000 kg m- power supply voltage of a
3
] source is 230V.
[g = 10 ms-2] (c) A source of e.m.f. 240V and
(c) (i) List three effects of heat frequency 50 Hz is connected to a
other series arrangement of a resistor; an
than expansion. inductor and a capacitor. When the
(ii) Explain saturated vapour current in the capacitor is 10A, the
pressure potential difference across the
(iii) A heating coil of resistance resistor is 140V and that across the
20Ω inductor is 50V. Calculate the:
(i) potential difference across
the capacitor;
(ii) capacitance of the capacitor;
(iii) inductance of the inductor.
(d) Draw and label one vector diagram
for the potential differences across
the inductor, the capacitor and the
resistor in (c) above.

15. (a) Explain:


(i) nuclear fission;
(ii) nuclear fusion.
(b) (i) State three applications of
atomic
energy.
(ii) Define half life.
(iii) Give the expression that
relates the half life, T, and the
decay constant, λ , of a
radioactive material.
(c) A radioactive element X with atomic
number 88 and mass number 226 2002
emits in succession: PAPER I PRACTICAL
(i) an alpha particle,
(ii) a beta particle and 1. Using the diagrams below as a guide, carry
(iii) gamma radiation. out the following instructions.
Explain, using equations where necessary, (i) Pivot the metre rule, which
the changes that take place in the atomic has
structure of the element at each stage. been drilled at the 50cm
mark, horizontally on the
knife edge.

Ia.

II.

(ii) Suspend the object marked m


at the 10cm mark of the
centre rule.
(iii) Suspend a 100g mass on the
other side of the knife edge
and adjust the position of the
mass until the metre rule
balances horizontally as
shown in diagram 1 above.
(vi) Read and record the
distances, x; of m from the
pivot and y1, of the 100g (ii) Place the rectangular glass
mass from the pivot. prism on the paper and trace
(v) Repeat the procedure for its outline, ABCD. Remove
four other positions of m at the prism.
the 15, 20, 25 and 30cm (iii) Draw a normal NMP to meet
marks. In each case kept he AB and DC at M and P
position of the knife edge respectively such that |
fixed and adjust the position AM| = |DP| = 2.0CM.
of the 100g mass until the
metre rule balances
(iv) Trace the ray PQ with two
pins, P1 and P2, at P and Q
horizontally.
respectively such that angle
(vi) Also repeat the procedure
MPQ = i = 5o.
with m completely immersed
in water contained in a (v) Replace the prism on its
beaker and suspended at the outline. Trace the emergent
10, 15, 20, 25 and 30cm ray with two other pins P3 and
marks of the metre rule P4 such that they lie in a
respectively. In each case, straight line with P2 and the
adjust the position of the image of P1 viewed through
100g mass until the metre the glass prism.
rule balances horizontally, (vi) Measure and record θ , the
then read and record the new angle between the emergent
distance y2 of the 100g mass ray and face AB of the glass
from the pivot. prism.
(vii) Evaluate y1 – y2 in each case. (vii) Evaluate cos θ and sin i.
Tabulate your readings. (viii) Repeat the procedure for four
(viii) Plot a graph of y1 on the other values of i = 10o, 15o,
vertical axis against y1 – y2 on 20o and 25o. Tabulated your
the horizontal axis. readings.
(ix) Determine the slope of the (ix) Plot a graph of cos θ on the
graph. vertical axis against sin c on
(x) State two precautions taken the horizontal axis.
to ensure accurate results. (x) Determine the slope of the
graph.
(b) (i) Explain what is meant by the (xi) State two precautions taken
centre of gravity of a body to ensure accurate results.
and state how it is related to [Attach your traces to your
the stability of the body. answer booklet].
(ii) State the conditions of
equilibrium for a body acted (b) (i) State the laws of refraction of
upon by a number of co- light.
planar parallel forces. (ii) Explain what is meant by the
statement the refractive
2. (a) index of a material is 1.65.

3. (a)

Using the diagram above as a guide,


carry out the following instructions. (i) Measure and record the
(i) Fix a plane sheet of paper on length XY
the of the resistance wire
drawing board. provided.
(ii) Connect the circuit shown in
the diagram above.
(iii) With R = 0Ω, close the key, K.
Read and record current, I
and the corresponding
voltage drop, V across the
wire.
(iv) Setting R = 1Ω, close the key.
Read and record the current, I
and the corresponding
voltage drop, V across the
wire.
(v) Repeat the procedure for five
other values of R = 5, 10, 20,
40 and 60Ω. Tabulate your
readings.
(vi) Plot a graph of V on the
vertical axis against I on the
horizontal axis.
(vii) Determine the slope of the
graph.
(viii) State two precautions taken
to ensure accurate results.

(b) (i) Mention and state the law on


which the experiment in 3(a)
is based.
(ii) A piece of resistance wire of
diameter 0.2mm and
resistance 7Ω has resistivity
of 8.8 x 10-7Ω m, calculate
the length of the wire [ π =
22
/7]

2003
SSCE PHYSICS
PAPER 2 OBJECTIVE

1. A tuber of cassava can be processed into


powder. This explains the fact that
A. matter can change spontaneously
from one state to another
B. matter cannot be destroyed
C. force is required to change matter from one
state to another
D. matter is made up of tiny particles

2. A piece of metal of relative density 5.0


weighs 60 N in air. Calculate its weight
when fully immersed in water.
A. 4N
B. 5N
C. 48 N
D. 60 N D. time taken to move from one extreme
position to the other
3. A net force of 15 N acts upon a body of
mass 3kg for 5s, calculate the change in the 9. The mass on a loaded spiral spring oscillates
speed of the body. vertically between two extreme positions P
A. 25.0ms-1 and R equidistant from the equilibrium
B. 9.0ms-1 position Q. Which of the following
C. 2.5ms-1 statements about the system is not correct?
D. 1.0ms-1 A. The momentum of the mass is maximum at
Q
4. The tendency of a body to remain at rest B. The elastic potential energy of the spring is
when a force is applied to it is called maximum at Q
A. impulse C. The kinetic energy of the mass is maximum
B. momentum at P
C. inertia D. The total energy of the system is always
D. friction constant

5. A force acts on a body for 0.5s changing its 10. A ball of mass 5.0kg hits a smooth vertical
momentum from 16.0kgms-1 to 21.0kgms-1 wall normally with a speed of 2 ms -1.
calculate the magnitude of the force. Determine the magnitude of the resulting
A. 42.0 N impulse.
B. 37.0 N A. 20.0kg ms-1
C. 32.0 N B. 10.0kg ms-1
D. 10.0 N C. 5.0kg ms-1
D. 2.5kg ms-1
6. Which of the graphs below shows the
motion of a body with uniform retardation? 11. Which of the following statements correctly
defines a simple machine? A device
A. Vel B. Vel A. that can provide electric current
m/s m/s B. which can only carry people from one place
to another
0 0
C. with which work can be done easily
Time Time
D. which changes the state of rest or of
uniform motion of an object along a straight
line

C. Vel D. Vel
12. A platinum resistance thermometer has a
m/s m/s resistance of 4Ω at 0ºC and 10Ω at 100ºC.
Assuming the resistance changes uniformly
0 Time 0 Time with temperature, calculate the resistance
of the thermometer when the temperature
is 45oC.
7. A moving object is said to have uniform A. 14.0 Ω
acceleration if its B. 6.7 Ω
A. displacement decreases at a constant rate C. 3.2 Ω
B. speed is directly proportional to time D. 2.7 Ω
C. velocity increases by equal amount in equal
time intervals 13. A metal rod of length 100cm is heated
D. velocity varies inversely with time through 100oC, calculate the change in
length of the rod. [linear expansitivity of the
8. the period of an oscillatory motion is defined material of the rod is 3 x 10-5 K-1]
as the A. 4 mm
A. average of the times used in completing B. 3 mm
different numbers of oscillations C. 2 mm
B. time to complete a number of oscillations D. 1 mm
C. time to complete one oscillation
14. The heat capacity of a calorimeter is the 19. A wave of wavelength 0.30m travels 900m
amount of energy required to in 3.0s, calculate its frequency.
A. change the temperature of 1kg of the A. 68.0 Hz
calorimeter by IK B. 225.0 Hz
B. change 1kg mass of the calorimeter to C. 270.0 Hz
liquid at the same temperature D. 750.0 Hz
C. change the temperature of the calorimeter
by IK 20. Which of the following conditions is
D. melt the calorimeter into liquid at a necessary for the occurrence of total
constant temperature internal reflection of light?
A. Light must travel from an optically less
15. Which of the following statements about dense to a denser medium
latent heat of vaporization is correct. It B. The angle of incidence must be equal to the
A. weakens the adhesive forces between critical angle
molecules of a liquid and those of its C. The angle of incidence must be greater than
container the critical angle
B. breaks completely the force of attraction D. The angle of refraction must be 90o
between the molecules of a liquid
C. adds to the heat content of a liquid 21. The image which cannot be formed on a
D. increases the cohesive forces between screen is said to be
liquid molecules A. inverted
B. erect
16. When the pressure of a fixed mass of gas is C. real
doubled at constant temperature, the D. virtual
volume of the gas is
A. increased four times 22. An object is placed in the principal axis and
B. doubled at the centre of curvature of a concave
C. unchanged mirror, the image of the object formed by
D. halved the mirror is
A. real and magnified
17. The volume of a given mass of an ideal gas B. real and inverted
at 327K and 9.52 x 104 Pa is 40cm3. C. erect and magnified
Calculate the volume of the gas at 273 K D. erect and virtual
and 1.034 x 105 Pa.
A. 68.23cm3 23. A converging lens produces an image four
B. 47.91cm3 times as large as an object placed 25cm
C. 44.60cm3 from the lens. Calculate its focal length.
D. 30.75cm3 A. 100cm
B. 33cm
C. 29cm
D. 20cm
18.
24. The horizontal floor of a water reservoir
appears to be 1.0m deep when viewed
vertically from above. If the refractive index
of water is 1.35, calculate the real depth of
the reservoir.
The diagram above represents the A. 2.35m
displacement D versus time, t, graph of a B. 1.35m
progressive wave. Deduce the frequency of C. 1.00m
the wave. D. 0.35m
A. 20 Hz
B. 10 Hz 25. A converging lens has a focal length of 5cm.
C. 5 Hz Determine its power.
D. 4 Hz A. + 20.0D
B. + 0.2 D
C. - 20.0 D D. Guitar
D. -0.2 D
32. Which of the following features of an
26. Which of the following pairs of light rays electrostatic line of force is not correct?
shows the widest separation in the A. It is n imaginary line which a positive charge
spectrum of white light? would describe if it is free to move
A. Green and blue B. The tangent drawn at any point on a curved
B. Orange and indigo line of force shows the direction of the
C. Blue and violet electric field intensity at that point
D. Red and yellow C. It is sometimes curved
D. It can cross another line of force in a region
27. Which of the following statements about the of intense electric field
human eye and the camera is correct?
A. Both form virtual images of objects 33. Which of the following statements about a
B. The image formed by the eye is upright in force field is correct?
the retina while that formed by the camera A. Electrostatic, gravitational and magnetic
is inverted on the film forces are always attractive
C. The inside of both of them is blackened to B. Electric, gravitational and magnetic fields
absorb unwanted light obey inverse square laws
D. The focal length of both the eye lens and C. Field lines are real but their corresponding
the camera lens can be varied fields are imaginary
D. Field lines and their corresponding fields are
28. The speed of sound in air is directly both real
proportional to
A. its temperature on the Celsius scale 34. A positively charged glass rod is placed near
B. its temperature on the thermodynamic scale the cap of a positively charge electroscope.
C. the cube root of its temperature on the The divergence of the leaf is observed to
Celsius scale A. decrease
D. the square root of its temperature on the B. increase
absolute scale C. remain the same
D. increase and collapse immediately
29. Which of the following statements about the
characteristics of sound waves is correct? 35. Two identical cells each of emf 2 V and
A. Loudness decreases with increase in internal resistance 1.0Ω are connected in
intensity parallel. The combination is connected to an
B. The pitch of a note is determined by its external load of 1.5Ω. Calculate the current
amplitude in the circuit.
C. The intensity of a sound wave is A. 0.57 A
proportional to its amplitude B. 1.00 A
D. The quality of a note depends on its C. 1.14 A
overtones D. 2.00 A

30. A sonometer wire of liner density 0.08kg m- 36. Which of the following factors does not
1 subjected to a tension of 800 N is plucked. affect the electric resistance of a wire?
Calculate the speed of a pulse which moves A. Length
from one end of the wire to the other. B. Mass
A. 179.0 ms-1 C. Temperature
B. 100.0 ms-1 D. Cross sectional area
C. 15.8 ms-1
D. 8.0 ms-1 37. A current of 10 A passes through a
conductor for 10s, calculate the charge
31. Which of the following instruments gives a flowing through the conductor
pure note when sounded? A. 100.0 C
A. Talking drum B. 10.0 C
B. Siren C. 1.0 C
C. Tuning fork D. 0.1 C
38. A lamp is rated 240 V 60 W, calculate the 43. The direction of induced current in a
resistance of its filament. straight wire placed in a magnetic field is
A. 240 Ω determined by using
B. 360 Ω A. Fleming’s right hand rule
C. 960 Ω B. Maxwell’s screw rule
D. 1440 Ω C. Faraday’s law
D. Lenz’s law
39. The maximum power dissipated by a 100 Ω
resistor in a circuit is 4 W, calculate the 44. Which of the following statements about an
voltage across the resistor. electromagnet is not correct?
A. 10 V A. It is a temporary magnet
B. 20 V B. Its strength depends on the current
C. 25 V C. Its strength depends on the number of units
D. 400 V in its coil
D. It has permanent poles
40. One difference between a reactance and a
resistance in an alternating current circuit is 45. Which of the following statements is true of
A. resistance is larger than reactance ultra violet radiations. It
B. energy is not dissipated in a reactance A. supports the sensation of hearing
C. energy is stored in a resistance but not in a B. does not cause the emission of electrons
reactance from metals
D. the current in a reactance is lower than in C. is an electromagnetic wave
the resistance D. is not capable of causing sun burn

41. 46. Which of the following statements is not


true of the isotopes of an element? They
A. are atoms of the same element
B. have the same chemical properties
C. have the same atomic number
D. have the same mass number

The diagram above shows the resultant 47. A radioactive element has a decay constant
electric field pattern due to two electric of 0.077 s-1, calculate its half life.
point charges P and S. Which of the A. 12.5 s
following statements is correct? B. 9.0 s
A. P is negatively charged while S is positively C. 5.1 s
charged D. 0.5 s
B. P is positively charged while S is negatively
charged 48. The minimum energy required to remove an
C. Both P and S are positively charged electron from an atom is known as
D. Both P and S are negatively charged A. excitation energy
B. ionization energy
42. C. binding energy
D. photon energy

49. Nuclear fission is preferred to nuclear fusion


in the generation of energy because
The bar magnet illustrated in the diagram A. very high temperature are required for
above picks up an unmagnetised steel nail fusion
X. In turn, X picks up a similar nail Y and so B. the raw materials for fusion are not easily
on. The nails are said to be magnetized by obtained
A. conduction C. energy obtained from fusion is relatively
B. single touch smaller
C. induction D. the by product of fusion are very dangerous
D. divided touch
50. Which of the following reactions represents
nuclear fusion?
A. 2H + 2H → 3He + 1n
1 1 1 0

B. 235u + 1n → 145Ba + 90Kr + 21n


92 0 56 36 0

C. 234Th → 0n + 234Pa
90 -1 91

D. 238U → 4He + 234Th


92 2 90

2003
SECTION B THEORY PART I

1. (a) State two objects each used in sports


and
warfare which may be considered as
projectiles
(b) the horizontal range R, of a projectile
is given by the expression R = U2 Sin

g
where θ is the angle of projection, g
the acceleration of free fall due to
gravity and u the initial velocity. At
what value of θ will R be maximum?

2. (a) What is surface tension?


(b) State two methods by which the (c) A tractor of mass 5.0 x 103kg is used
surface tension of a liquid can be to tow a car mass 2.5 x 103kg. The
reduced tractor moved with a speed of
3.0ms-1 just before the towing rope
3. (a) Explain diffusion. becomes taut. Calculate the:
(b) State one factor that can affect the (i) speed of the tractor
rate of diffusion. immediately the rope
becomes taut;
4. (a) List two types of waves, apart from (ii) loss in kinetic energy of the
light, system just after the car has
that can be plane polarized. started moving;
(b) State two applications of plane (iii) impulse in the rope when it
polarized light. jerks the car into motion

5. Explain the rise of water in a glass capillary 12. (a) Using the kinetic theory of matter,
tube using the kinetic theory. explain
why
6. (a) Explain the term electrolyte and give (i) evaporation causes cooling;
two (ii) boiling water changes to
examples. steam without any change in
(b) State the relationship between the temperature, although heat is
mass of a substance liberated during being supplied to the water.
the electrolysis and the charge (b) (i) State Boyle’s law
passed. (ii) With the aid of a labeled
diagram, describe an
7. (a) What is the principle upon which the experiment to illustrate the
lighting in fluorescent tubes operate? relationship between the
(b) State two factors which determine volume and pressure of a
the colour of light from a fluorescent given mass of gas at constant
tube temperature.
(iii) State two precautions
8. Define: (i) Elasticity necessary to obtain accurate
(ii) Young modulus results.
(iii) Force constant
13. (a) (i) Illustrate, using a ray
9. A force of 40 N applied at the end of a wire diagram,
of length 4 m and diameter 2.00mm how an image is formed by a
produces an extension of 0.24mm. Calculate convex mirror
the (ii) State one advantage and one
(a) stress on the wire; disadvantage of using a
(b) strain in the wire; ( π = 3.14) convex mirror as a driving
mirror.
10. (a) What is meant by the wave-particle (iii) Explain the action of a
quality of matter? compound microscope
(b) Mention one physical phenomenon, (b) Illustrate using labeled diagrams
in each case, that can be explained only, a sonometer wire of length, l,
in terms of the wave nature and vibrating at its
particle nature of light. (i) fundamental; (ii) first
overtone;
PART II (iii) second overtone
(c) A tuning fork vibrating at a
11. (a) Define: (i) linear momentum (ii) frequency of 512 Hz is held over the
impulse top of a jar filled with water and
(b) State the principle of conservation of fitted with a tap at the bottom. If the
linear momentum. jar is 60cm tall and the speed of
sound is 350ms-1, determine the
possible resonance position(s).
[neglect end corrections]

14. (a) (i) What is meant by neutral


point in
a magnetic field?
(ii) Draw and label a diagram to
show the pattern and
direction of the magnetic field
produced around a straight
current carrying wire.
(b) When is an ammeter said to be
(i) sensitive; (ii) accurate?
(c) (i) Explain, using a labeled
diagram,
how a delicate magnetic
material could be protected
from the Earth’s magnetic
field.
(d) State the laws of electromagnetic
induction

15. (a) (i) Explain photoelectric


emission
(ii) State four applications of
photoelectric emission
(b) Draw and label a diagram showing
the
structure of a photocell and explain
its mode of operation.
(c) In a photocell, no electrons are
emitted until the threshold frequency
of light is reached.
(i) Explain what happens to the
energy of the light before
emission of electrons begin.
(ii) State one factor that may
affect the number of emitted
electrons

2003
PAPER I PRACTICAL
1. (a)
Using the diagram above as a guide,
carry out the following instructions.
(i) Fix the drawing paper
provided on the drawing
You are provided with a metre rule, a board
knife edge, two pieces of thread and (ii) Place the mirror vertically
two masses m1 and m2. with its longer side resting on
(i) Record the values of m1 and the drawing paper. Trace the
m2 outline AB of the mirror.
(ii) Balance the metre rule Remove the mirror.
horizontally on the knife edge (iii) Draw a normal PQ to meet
and record the balance point the outline at the middle Q
G. (iv) Draw a straight line through A
(iii) With the knife edge at the to meet the outline of the
60cm mark of the metre rule, mirror at right angle.
suspend m1 at the 20cm mark (v) Trace the incident ray, CQ
and m2 at a suitable mark with pins P1 and P2 so that it
such that the rule balances meets the perpendicular line
horizontally as illustrated in through A at C such that CA =
the diagram above. x = 1.0cm.
(iv) Record the positions Y on m1 (vi) Replace the mirror on its
and Q of m2 outline. Locate the images of
(v) Evaluate l = P – Y and d = Q – P1 P2 through the mirror using
P two other pins P3 and P4, so
(vi) Repeat the procedure for four that P3 and P4 and the images
other position of m1 at 18, of P1 and P2 are in a straight
16, 14 and 12cm marks. line.
(vii) In each case, evaluate and (vii) Remove the mirror and pins
record l and d. P3 and P4. Draw a straight line
(viii) Tabulate your readings through the pin points to
meet AB at Q and CA
(ix) Plot a graph of l on the
produced at D.
vertical axis against d on the
horizontal axis. (viii) Measure and record angle
(x) Determine the slope of the ACQ as θ 1 and angle ADQ as
graph. θ 2. Also record the value of x.
(xi) State two precautions taken Evaluate θ = ½ ( θ 1 + θ 2), x-1
to ensure accurate results. and tan θ .
(b) (i) With the aid of a diagram,
indicate the forces acting on (ix) Repeat the procedure for four
the other values of x = 2.0, 3.0,
metre rule in the 4.0 and 5.0cm. Tabulate your
experimental set up above. readings.
(ii) Define moment of a force (x) Plot a graph of tan θ on the
about a point and state its S. vertical axis against x-1 on the
I. unit. horizontal axis.
(xi) Determine the slope, s, of the
2. (a) graph. Evaluate k = 2s.
(xii) State two precautions taken
to ensure accurate results.
[Attach your traces to your
answer booklet]
(b) (i) Distinguish between regular potential difference
and between two points in
diffused reflections. an electric circuit.
(ii) An object is situated 25cm in (ii) A piece of resistance
front of a plane mirror. wire of diameter 0.2m
Determine the distance of the and resistance 7Ω has
image from the object. What resistivity of 8.8 x 10-
is the size of the image 7Ωm, calculate the
relative to the object? length of the wire. [ π
= 22/7]

3. (a)

You are provided with a


potentiometer XY; a jockey, J; a
standard resistor, R and other
necessary apparatus.
(i) Connect a circuit as
shown in the diagram
above.
(ii) Close the key. Read
and record the current
Io when J is not in
contact with XY.
(iii) Let J make contact
with XY at C, such that
XC = l = 25cm. Close
the key. Read and
record the current I.
(iv) Evaluate 1o
1
(v) Repeat the procedure
for four other values of
l = 40, 55, 70 and
85cm. Tabulate your
readings.
(vi) Plot a graph of I on the
vertical axis against l-1
on the horizontal axis.
(vii) From your graph,
deduce the value of 1
when l-1 = 0. Evaluate
Io
(viii) State two precautions
taken to ensure
accurate results.

(b) (i) Explain what is meant


by
5. An object weighs 60.0 N in air, 48.2 in a
certain liquid X, and 44.9 N in water.
Calculate the relative density of X.
A. 3.300
B. 1.279
C. 0.932
D. 0.782

6. A steel needle floating on water, sinks when


kerosene is added to the water. This is
because the kerosene
A. increases the surface tension of the water
B. reduces the density of water
C. reduces the surface tension of the water
D. reduces the upthrust on the needle
2004
SSCE PHYSICS 7. Calculate the change in volume when 90g of
PAPER 2 OBJECTIVE ice is completely melted. [density of water
= 1gcm-3, density of ice = 0.9gcm-3]
1. The molecular structure of a liquid is such A. 0.00cm3
that B. 9.00 cm3
A. The molecules have unlimited freedom of C. 10.00 cm3
movement D. 90.00 cm3
B. molecular movement is restricted to
vibration only 8. On which of the following factors does the
C. intermolecular collisions take place viscosity of a liquid depend?
D. all molecules move with the same speed I. Nature of the liquid;
II. Temperature of the liquid;
2. Which of the following concepts is not an III. Area of the liquid surface.
evidence of the particle nature of matter? A. I and II only
A. Diffusion B. II and III only
B. Brownian motion C. I and III only
C. Diffraction D. I, II and III
D. Crystallization
9. An elastic string of force constant 200 N m-1
3. Which of the following statements about the is stretched through 0.8m within its elastic
mass and the weight of a body is not limit. Calculate the energy stored in the
correct? Its string
A. mass is a scalar quantity A. 64.0 J
B. weight is a function of the gravitational pull B. 80.0 J
on it C. 128.0 J
C. mass on earth and on the moon is the same D. 160.0 J
D. weight at the equator and at the poles is the
same 10. A machine of velocity ratio 5 is used in
lifting a load with an effort of 500 N. If the
4. A diver is 5.2m below the surface of water machine is 80% efficient, determine the
of density 1000 kgm-1. If the atmospheric magnitude of the load.
pressure is 1.02 x 105 Pa, calculate the A. 2500 N
pressure on the diver. [g = 10 ms-2] B. 2000 N
A. 6.20 x 104 Pa C. 900 N
B. 1.02 x 105 Pa D. 625 N
C. 1.54 x 105 Pa
D. 5.20 x 105 Pa 11. The speed of an object in rectilinear motion
can be determined from the
A. area under a velocity-time graph
B. area under a distance-time graph
C. slope of a distance-time graph 2, calculate the angular frequency of the
D. slope of a velocity-time graph motion
A. 2 rad s-1
12. A body is said to be moving with uniform B. 4 rad s-1
acceleration if it experiences equal C. 5 rad s-1
A. increases in velocity at equal time intervals D. 10 rad s-1
B. decreases in velocity at equal time intervals
C. increases in speed at equal time intervals 19. When compared with water, mercury is
D. decreases in speed at equal time intervals preferred as a thermometric liquid because
it
13. A body dropped from a certain height above A. has a higher density
the ground level, falls with uniform B. has a lower specific heat capacity
A. speed C. exhibits anomalous thermal expansion
B. velocity D. contains molecules which have very strong
C. acceleration adhesive forces
D. retardation
20. Which of the following values on the
14. A body of mass 8kg moving with a speed of absolute scale of temperature is the ice
30ms-1 is brought to rest in 2.5s by a point?
constant retarding force. Calculate the A. 0K
magnitude of the force B. 32 K
A. 240 N C. 100 K
B. 96 N D. 273 K
C. 20 N
D. 9N 21. On a cold day, a concrete floor feels colder
to the bare foot than a mat on the same
15. A force acting on a body causes a change in floor because the
the momentum of the body from 12kg ms-1 A. mat is a better conductor of heat than the
to 16kgms-1 in 0.2s. Calculate the foot
magnitude of the force B. mat loses heat to the bare foot at a faster
A. 140.0 N rate
B. 20.0 N C. mat is a better conductor of heat than the
C. 4.0 N floor
D. 0.8 N D. concrete floor is a better conductor of heat
than the mat
16. The bob of a simple pendulum has a mass
of 0.02kg. Determine the weight of the bob. 22. When a metal ball is heated through 30oC,
[g = 10ms-2] its volume becomes 1.0018 cm-3. If the
A. 0.02 N linear expansivity of the material of the ball
B. 0.20 N is 2.0 x 10-5 K-1, calculate its original volume
C. 20.00 N A. 1.0000 cm3
D. 200.00 N B. 1,0020cm3
C. 1.0036 cm3
17. A ball bearing is projected vertically D. 1.0180 cm3
upwards from the ground with a velocity of
15ms-1. Calculate the time taken by the ball 23. A faulty barometer reads 72.6cm Hg when
to return to the ground. [g = 10ms-2] the atmospheric pressure is 75.0cm Hg.
A. 1.5s Calculate the atmospheric pressure when
B. 3.0s this barometer reads 72.0cm Hg.
C. 5.0s A. 79.4 cm Hg
D. 7.5s B. 74.4 cm Hg
C. 74.2 cm Hg
18. A loaded spring performs simple harmonic D. 69.4 cm Hg
motion with an amplitude of 5cm. If the
maximum acceleration of the load is 20cms-
24. On the basis of the kinetic theory, an
increase in the temperature of a fixed 30. In an experiment using a converging lens to
volume of an ideal gas causes produce real images on a screen, the linear
A. an increase in the number of the gs magnification, m, is plotted against the
molecules image distance, v, as illustrated in the
B. a decrease in the density of the gas diagram below. The distance OP represents
C. an increase in the average speed of the gas the
molecules
D. a decrease in the pressure exerted by the m
gas

25. The temperature of 900cm3 of an ideal gas


at a pressure of 114cm Hg is 27oC. Calculate
0 p v
its volume at 76 cm Hg and 0oC.
A. 50.0 cm3
B. 546.0 cm3
C. 659.3 cm3 A. focal length of the lens
D. 1228.5 cm3 B. thickness of the lens
C. radius of curvature of the lens
26. The basic difference between a transverse D. diameter of the lens
wave and a longitudinal wave traveling in
the same direction in a medium is in the 31. A magnified and virtual image of a near
A. amplitude of the waves object is produced by
B. wave length of the waves A. prism binoculars
C. direction of vibration of the particles of the B. astronomical telescope
medium C. periscope
D. period of vibration of the particles of the D. simple microscope
medium
32. During a thunderstorm, lightning and
27. A wave traveling from water to glass suffers thunder occurred simultaneously in the
a change in its speed at the common atmosphere. An observer on the earth’s
boundary. Which of the following properties surface sees the lightning flash shortly
explains this observation? before hearing the thunder. This is because
A. Dispersion the
B. Refraction A. atmospheric pressure is higher at the
C. Interference surface than at the higher altitudes
D. Diffraction B. speed of sound is less than that of light
C. temperature of the atmosphere decreases
28. The image of an optical pin placed at the with altitude
principal focus of a concave mirror will be D. atmospheric density decreases with altitude
formed
A. between the principal focus and the pole 33. A girl stands 80m away from a tall cliff and
B. at the centre of curvature blows a whistle. If the speed of sound in air
C. between he principal focus and the centre is 330ms-1, how long would it take for her
of curvature to hear the echo of the sound?
D. at infinity A. 2.062 s
B. 0.970 s
29. A man 1.8m tall stands 3m away from a pin C. 0.485 s
hole camera. If the distance between the D. 0.242 s
pin hole and the screen of the camera is
0.3m, calculate the height of the image of 34. Which of the following reasons explains why
the man produced by the camera. the walls and ceilings of a standard concert
A. 0.18 m hall are usually covered with perforated
B. 1.50 m pads? To
C. 1.62 m A. increase the intensity of sound waves
D. 18.00 m B. increase the loudness of the sound waves
C. reduce the effect of reverberation of sound
waves 41. Calculate the energy stored in a 20µF
D. decrease the frequency of sound waves capacitor if the p.d. between he plates is 40
V.
35. The sound from a vibrating tuning fork is A. 3.2 x 10-2 J
louder when the tip of its stem is placed on B. 1.6 x 10-2 J
a wooden table than when in air because C. 8.0 x 10-4 J
A. the friction between the table and the D. 4.0 x 10-4 J
tuning fork generates additional energy
B. air molecules absorb sound waves but wood 42. Given that the gravitational constant is G,
transmits sound waves Newton’s universal law of gravitation states
C. the table increases the frequency of the that the force of attraction between two
tuning fork masses M1 and M2 separated by a distance r
D. a larger mass of air is set into vibration by is
the table A. Gr2 M2 M2

36. Charges on a conductor are B. GrM1 M2


A. evenly distributed over a surface of any
shape C. GM1 M2
B. concentrated on the inner surface of the r2
conductor
C. concentrated at the outer flat surface of the D. GM1 M2
conductor r
D. generally concentrated at sharply curved
surface 43. A charge of 1.6 x 10-10C is placed in a
uniform electric field of intensity 2.0 x 105
37. Which of the following functions is NC-1. Calculate the magnitude of the
performed by manganese IV oxide in the electric force exerted on the charge.
Leclanche’ cell? A. 3.2 x 105 N
A. Control of local action B. 1.8 x 105 N
B. Depolarisation of the cell C. 3.2 x 10-5 N
C. Regulating current flow in the cell D. 1.8 x 10-5 N
D. Reduction of the internal resistance of the 44. Which of the following statements is not
cell correct?
A. A magnetic field is a region in which a
38. A car fuse marked 3 A operates optimally on magnetic force may be detected
a 12 V battery, calculate the resistance of B. A magnetic line of force is a path along
the fuse which a magnetic north-pole would move if
A. 36.0 Ω it were free
B. 15.0 Ω C. Magnetic fields are scalar quantities
C. 9.0 Ω D. Neutral points are obtained where the
D. 4.0 Ω earth’s magnetic field is exactly equal and
opposite to that due to a magnet
39. Find the cost of running a 60 W lamp for 24
hours, if I kW hr costs 5 Naira. 45. The main function of the mouth piece of a
A. 14.4 Naira telephone is that it converts sound energy
B. 12.5 Naira to
C. 7.2 Naira A. mechanical energy
D. 2.0 Naira B. electrical energy
C. chemical energy
40. The quantity of electric charge stored in a D. thermal energy
capacitor is measured in
A. Farad 46. An atom is normally electrically neutral
B. Ampere because the
C. Coulomb A. number of electrons is equal to the number
D. Volt of protons
B. number of electrons is equal to the number
of neutrons
C. number of protons is equal to the number of
neutrons
D. electrons of the atom are buried deeply in
the nucleus

47. In which of the following devices is the


principle of photoelectric effect not
applicable?
A. Closed-circuit television camera
B. Burglar alarm
C. Solar cells
D. Periscope

48. Which of the following statements about X-


rays is not correct? They
A. are electromagnetic waves
B. can be used to study crystal lattice
C. can destroy living cells
D. are produced from the nucleus of an atom

49. A radio active substance has a half-life of 3


days. If a mass of 1.55g of this substance is
left after decaying for 15 days, determine
the original value of the mass
A. 49.6 g
B. 37.2 g
C. 24.8 g
D. 12.4 g

50. Which of the following representations is


correct for an atom X with 28 electrons and
30 neutrons?
A. 30X
28

B. 28X
30

C. 58X
30

D. 58X
28
the air at an angle above the horizontal.
Copy the diagram, and, using arrows,
indicate the relative magnitudes and
directions of the vertical and horizontal
components of the velocities of the bullet at
the points A, B and C.

3. (a) For a water voltameter, identify the


(i) electrolyte; (ii) electrodes;
(iii) substances deposited on the
electrodes:
(b) State the ratio of the volume of the
substances deposited in (a) (iii)
above.

4. A parallel beam of unpolarized light is


incident on a plane glass surface at an
angle of 58o to the normal. If the reflected
beam is completely polarized, calculate the
refractive index of the glass.

5. State three methods of polarizing an


unpolarised light.

6. (a) Define diffusion.


(b) State Grahams law of diffusion

7. A spiral spring of natural length 20.00 cm


has a scale pan hanging freely in its lower
end. When an object of mass 40g is placed
in the pan, its length becomes 21.80cm.
2004 When the object is replaced with another of
EASSY SECTION B PART I mass 60g, the length becomes 22.05 cm.
Calculate the mass of the scale pan. [g =
1. In his first attempt, a long jumper took off 10m s-2]
from the spring board with a speed of 8 ms-
1 at 30o to the horizontal. He makes a 8. (a) Define youngs modulus
second attempt with the same speed at 45o (b) When a force of 50 N is applied to
to the horizontal. Given that the expression the free end of an elastic cord, an
for the horizontal range of a projectile is u2 extension of 4 cm is produced in the
sin 2 θ cord. Calculate the work done on the
g, where all the symbols cord.
have their usual meanings, show that he
gains a distance of 0.8576m in his second 9. (a) List two properties of cathode rays
attempt. [g = 10 m s-2] (b) Explain how the intensity and energy
Z of cathode rays may be increased.

10. Give three observations in support of de


Broglie’s assumption that moving particles
behave like waves.

PART II
A C x
Answer any three questions from this part.

2. The diagram above illustrates the path ABC, 11. (a) Define gravitational field intensity.
in a vertical x – z plane, of a bullet shot into
(b) In an experiment to determine the 13. (a) Explain
acceleration of free-fall due to (i) wave motion;
gravity, g, using a simple pendulum (ii) stationary wave
of length l, six different values of l (b) (i) List four physical properties
were used to obtain six of a
corresponding values of period T. If a wave
graph of l along the vertical axis is (ii) Define amplitude and use it to
plotted against T2 on the horizontal distinguish between the node
axes; and antinode of a stationary
(i) make a sketch to show the wave.
nature (iii) List the factors on which the
of the graph; frequency of vibration in a
(ii) write down the equation that stretched string depends.
relates T, l and g hence (c) The equation, y = 5 sin (3x – 4t),
obtain an expression for the where y is in millimeters, x is in
slope of the graph; metres and t is in seconds
(iii) given that the slope of the represents a wave motion.
graph is 0.25, determine the Determine the
value for g. [Take π = 3,142] (i) frequency
(c) A stone, thrown horizontally from the (ii) period and
top of a vertical wall with a velocity (iii) speed of the wave.
of 15ms-1, hits the horizontal ground
at a point 45 m from the base of the 14. (a) (i) Explain electromotive force
wall. Calculate the (ii) List two sources of
(i) time of flight of the stone; electromotive force other
(ii) height of the wall. [g = 10m s- than a chemical cell.
2
] (b) A chemical cell of electromotive
force, E, and internal resistance, r, is
12. (a) Define upper fixed point and lower connected in series with an
fixed ammeter, a plug key and an external
Point as used in thermometry. load of resistance R. A voltmeter is
(b) The electrical resistances of the connected across the cell. Draw a
element circuit diagram to illustrate the
in a platinum resistance arrangement.
thermometer at 100oC, 0oC and room (c) For the arrangement in (b) above,
temperature are 75,000, 63,000 and with the key opened and closed, the
64.992Ω respectively. Use these voltmeter readings are Vo and V
data to determine the room respectively.
temperature. (i) Explain the physical meanings
(c) (i) State Boyle’s law; of Vo and V.
(ii) A uniform capillary tube, (ii) Find an expression for the
closed at one end contained (I) current passing through
dry air trapped by a thread of the
mercury 8.5 x 10-2 m long. circuit;
When the tube was held (II) maximum power
horizontally, the length of the dissipated
air column was 5.0 x 10-2m, in the cell and external
when it was held vertically load respectively.
with the closed end (III) efficiency of the cell.
downwards, the length was 15. (a) State two
4.5 x 10-2m. Determine the (i) Properties of x-rays;
value of the atmospheric (ii) reasons to show that x-rays
pressure. [g = 10 ms-2, are
density of mercury = 1.36 x waves;
104 kg m-3] (iii) uses of x-rays other than
those in medicine;
(iv) hazards of x-rays.
(b) The potential difference between the
cathode and target of an x-ray tube
is 5.00 x 104 V and the current in the
tube is 2.00 x 10-2 A. Given that only
one percent of the total energy
supplied is emitted as x-radiation,
determine the
(i) maximum frequency of the
emitted radiation;
(ii) rate at which heat is removed You have been provided with a retort stand,
from the target in order to clamp and boss; a metre rule; a spiral
keep it at a steady spring; a set of slotted weights and a
temperature. pointer.
[Planck’s constant, h = (i) Set up the apparatus as
6.63x10-34J s] shown in the diagram above.
[electronic charge e=1.60x (ii) Read and record the length, lo
10-19C] of the spiral spring.
(iii) Place a mass m = 50.0g on
the hanger
(iv) Read and record the new
length, l, of the spring.
(v) Determine the extension e = l
- lo
(vi) Set the system into small
vertical oscillations. Measure
and record the time, t for ten
complete oscillations.
(vii) Determine the period. T, of
the oscillations and evaluate
T2
(viii) Repeat the procedure for
four other values of m =
100.0 ←150.0
 ,200.0, and
250.0g. Tabulate your
readings.
(ix) Plot a graph of e on the
vertical axis against T2 on the
horizontal axis.
(x) Determine the slope, S, of the
graph
(xi) Evaluate b = 4 π 2 x S
(xii) State two precautions taken
to ensure accurate results
(b) (i) State Hooke’s Law for an
elastic
Material.
(ii) A spiral spring is compressed
by 0.03 m. Calculate the
2004 energy stored in the spring if
PAPER I PRACTICAL its force constant is 300 N m-1

1.
2. (a) (xi) Evaluate 1_
√S
(xii) State two precautions taken
to ensure accurate results.
(b) (i) Define the principal focus of a
converging lens.
(ii) An object and its real
image are located at
distances 25.0cm and
40.0cm from the two
principal foci of a
converging lens
respectively, calculate
the focal length of the
lens.
You are provided with a converging lens, a
plane mirror, a ray box, a screen and a 3. (a)
metre rule.
(i) Mount the plane mirror
behind the lens and place the
ray box in front of it as shown
in diagram I above.
(ii) Adjust the position of the ray
box until a sharp image of the
cross wire is obtained in front You have been provided with a resistance
of the box, beside the cross box, a voltmeter, a key and a battery.
wire. (i) Connect the circuit as shown
(iii) Measure and record the in the diagram above.
distance of the cross wire (ii) With the key, K closed, read
from the lens. This position of and record the voltmeter
the cross wire is F1 reading Vo
(iv) Repeat the procedure with (iii) Set the resistance R in the
the lens reversed to obtain resistance box equal to 1Ω.
F2. Close the key, read and
(v) Replace the mirror with a record the potential
screen. difference V on the voltmeter.
(vi) Move the ray box a distance d Evaluate π R-1 and V01.
= 10.0cm from F2 and adjust (iv) Repeat the procedure for five
the position of the screen other values of R = 2, 3, 4, 5
beyond F1 until a sharp image and 6Ω. Tabulate your
of the wire is formed on the readings.
screen as shown in diagram II.
(v) Plot a graph of V-1 on the
(vii) Measure and record the vertical axis against R-1 on
distance, l between F1 and the the horizontal axis
screen. Evaluate l-1
(vi) Determine the slope of graph
(viii) Repeat procedures (v), (vi) and the intercept I on the
and (vii) for four other values vertical axis.
of d = 15.0, 20.0, 25.0 and
30.0cm. Tabulate your
(vii) Evaluate l-1
readings. (viii) State two precautions taken
to ensure accurate results.
(ix) Plot a graph of l-1 on the
vertical axis against d on the
(b) (i) Define potential difference
readings.
between two points in an
(x) Determine the slope, s, of the
electric circuit.
graph.
(ii) Explain why the emf of a cell
is greater than the p.d across
the cell when it is supplying
current through an external
resistance.

2005
SSCE PHYSICS
PAPER 2 OBJECTIVE

1. Which of the following phenomena is the


practical evidence for the existence of the
continual motion of molecules?
A. Translational motion
B. Rotational motion
C. Brownian motion
D. Oscilatory motion

2. Each of the following physical quantities is


classified as a vector or a scalar quantity.
Which of the classifications is correct?
A. electric potential (Vector)
B. Momentum (Scalar)
C. Gravitational field intensity (Scalar)
D. Magnetic flux density (Vector)

3. Which of the following statements about a


spring balance and/or a chemical balance is
not correct?
A. The chemical balance operates on the
principle of moments
B. The spring balance operates on Hooke’s law
C. Either may be used to measure the weight
of a substance
D. The reading of a spring changes over the
surface of the earth, while that of the
chemical balance remains constant

4. The friction which exists between two layers


of liquid in relative motion is called
A. capillarity
B. surface tension
C. viscosity
D. cohesion

5. A body of mass 25kg, moving at 3ms-1 on a


rough horizontal floor is brought to rest
after sliding through a distance of 2.50m on
the floor. Calculate the coefficient of sliding
friction. [g = 10.0ms-2]
A. 0.09
B. 0.18
C. 0.36
D. 0.54 A. number of complete oscillations the
pendulum makes in one second
6. Which of the following simple machines is a B. number of amplitudes the bob makes in one
second class lever? second
A. Wheel barrow C. angle the bob swings through in one second
B. Crowbar D. distance the bob covers in one second
C. Claw hammer
D. Seesaw 13. Which of the following statements about an
7. A wheel and axle of radii 800mm and object performing simple harmonic motion
200mm respectively is used to raise a body is correct. Its acceleration
of weight 800 N by application of 250 N. A. is maximum at the extreme ends
Calculate the efficiency of machine. B. is constant and directed towards a fixed
A. 85% point
B. 80% C. is zero when it is displaced from the
C. 60% equilibrium position
D. 20% D. varies linearly with the displacement from a
fixed point and is directed towards the fixed
8. Messes m1 and m2 at the 20-cm and 65-cm point
marks respectively of a uniform metre rule
freely suspended at its centre of gravity. If 14. A body of mass 5kg moving with a velocity
the metre rule balances horizontally, of 10ms-2 collides with a stationary body of
determine the ratio m2: m1. mass 6kg. If the two bodies stick together
A. 2:1 and move in the same direction after the
B. 3:3 collision, calculate their common velocity.
C. 2:3 A. 4.55 ms-1
D. 1:2 B. 12.00 ms-1
C. 30.0 ms-1
9. Which of the following types of motion does D. 50.00 ms-1
a body undergo when acted upon by a
couple? 15. Fuel was consumed at a steady rate of 5.0 x
A. Vibrational 10-2 kg per second in a rocket engine and
B. Translational ejected as a gas with a speed of 4 x 103 ms -
C. Rotational 1
. Determine the trust on the rocket.
D. Random A. 20 n
B. 80 n
10. A body starts from rest and accelerates C. 200 n
uniformly at 5 ms-2 until it attains a velocity D. 800 n
of 25 ms-1. Calculate the time taken to
attain this velocity. 16. Which of the following quantities has the
A. 2.5 s same unit as energy?
B. 5.0 s A. Power
C. 10.0 s B. Work
D. 125.0 s C. Force
D. Momentum
11. A particle accelerates uniformly from rest at
6.0 ms-2 for 8s and then decelerates 17. Which of the following sources of energy is
uniformly to rest in the next 5s. Determine renewable?
the magnitude of the deceleration. A. Sun
A. 9.6 ms-2 B. Petroleum
B. 24.0 ms-2 C. Coal
C. 30.0 ms-2 D. Uranium
D. 48.0 ms-2
18. The lower and upper fixed points of a
12. The frequency of a swinging pendulum is thermometer are 30mm and 180mm
the respectively. Calculate the temperature in
degrees Celsius when the thermometer D. Boiling takes place at 100oC while
reads 45mm. evaporation may take place at temperature
A. 10.0oC lower than 100oC
B. 15.0oC
C. 20.0oC 24. Which of the following statements is
D. 30.0oC correct?
A. The density of a liquid decreases when it
19. A body of mass, m, has a specific heat expands
capacity s, and a heat capacity, c. If the B. The densities of liquids increase when the
temperature of the body changes by θ oC, liquids are heated
which of the following equations is correct? C. The real expansivity of a liquid is less than
A. ms θ = mc-1 its apparent expansivity
D. A liquid changes to solid when heated to a
B. ms = C
sufficiently high temperature
C. ms = s θ
D. ms = c θ 25. The pressure exerted by a given mass of
gas in a container
20. An immersion heater rated 400W,220V, is A. decreases if the container is heated
used to heat a liquid of mass 0.5kg. If the B. increases if the molecules of the gas move
temperature of the liquid increases faster
uniformly at the rate of 2.5oC per second, C. increases if the volume of the container is
calculate the specific heat capacity of the doubled
liquid. [Assume no heat is lost] D. decreases as the kinetic energy of the gas
A. 1100 Jkg-1K-1 molecules increases
B. 320 Jkg-1K-1
C. 200 Jkg-1K-1 26. Which of the following statements about a
D. 176 Jkg-1K-1 progressive mechanical wave is correct?
A. It can be plane polarised
21. Water falls through a height of 50m. B. Its energy is localized at specific points of its
Determine the temperature rise of the water profile
at the bottom of the fall. [Neglect energy C. It does not require a material medium for its
losses. Specific heat capacity of water = propagation
4200 Jkg-1K-1, g = 10 ms-2] D. Its frequency remains constant as it travels
A. 0.119oC between different media
B. 0.840oC
C. 1.190oC 27. The distance between two points in phase
D. 8.400oC on a progressive wave is 5cm. If the speed
of the wave is 0.20ms-1, calculate its
22. The energy required to change a solid to period.
liquid at constant temperature is called A. 4.00s
A. thermal capacity B. 2.50s
B. specific heat capacity C. 0.25s
C. latent heat of fusion D. 0.04s
D. latent heat of vaporisation
28. Radio waves travel in air at 3.0 x 108 ms-1. If
23. Which of the following statements about the waves enter water of refractive index
boiling and evaporation at atmospheric 4/3, calculate the speed of radio waves in
pressure is correct? water.
A. Evaporation and boiling of water take place A. 4.33 x 108 ms-1
at 100oC B. 4.00 x 108 ms-1
B. Evaporation of water takes place at a C. 2.25 x 108 ms-1
temperature greater than 100oC D. 1.33 x 108 ms-1
C. Evaporation of water takes place at 100oC
while boiling takes place at a higher 29. The amplitude of a wave is the
temperature A. distance traveled by the wave in a complete
cycle of its motion
B. maximum displacement of the wave particle D. 120000.0C
from the equilibrium position
C. separation of two adjacent particles 36. A chemical cell of internal resistance 1Ω
vibrating in a phase supplies electric current to an external
D. distance between two successive troughs of resistor of resistance 3Ω. Calculate the
the wave efficiency of the cell.
A. 75%
30. Which of the following optical instruments B. 50%
does not depend on the use of plane C. 33%
mirrors? D. 25%
A. Kaleidoscope
B. Simple microscope 37. An electric lamp is rated 240V, 40W. What is
C. Sextant the cost of running the lamp for 72 hours if
D. Simple periscope the electricity authority charges N2.50 per
kWh?
31. A ray of light is incident normally on a plane A. N7.20
mirror. If the incident ray is kept fixed while B. N18.00
the mirror is rotated through an angle of C. N180.00
30o, determine the initial and final angle of D. N720.00
reflection respectively.
A. 0o and 60o
B. 90o and 30o 38. Which of the following statements explains
C. 0o and 30o lost volt in an electric circuit? The
D. 90o and 60o A. total p.d from a source to maintain a flow of
current through the circuit as well as the
32. In which of the following media is the source
transmission of sound waves fastest? B. p.d. to drive current through the circuit
A. Vacuum components except the source
B. Air C. p.d. across all external components
C. Wood connected to the source of electricity
D. Iron D. p.d across the internal resistance of the cell

33. The ability of a material to store an electric 39. A constantan wire has a cross sectional area
charge when its plates are at different of 4 x 10-8 m2 and a resistivity of 1.1 x 10-
potentials is referred to as its 6
Ωm. If a resistor of resistance 11Ω is to be
A. conductance made from this wire, calculate the length of
B. inductance the wire required.
C. capacitance A. 0.4 m
D. resistance B. 0.8 m
C. 2.5 m
34. Two capacitors of capacitances 0.4µF and D. 5.0 m
0.5µF are connected in parallel and charged
to a p.d. of 50V. Determine the total charge 40. The mass weight of a body on earth are 8kg
acquired. and 80 N respectively. Determine the mass
A. 45µC and weight of the body respectively on a
B. 25µC planet where the pull of gravity is 1/8 that of
C. 20µC earth.
D. 10µC A. 8kg. 8 N
B. 1kg. 10N
35. A parallel plate capacitor of capacitance C. 64kg. 10N
600µF has a potential difference of 2000V D. 8kg. 10N
between its plate. Calculate the charge on
either plate of the capacitor. 41. Lenz’s law of electromagnetic induction
A. 1.2C states that
B. 3.3C A. a force is exerted on a current-carrying
C. 3330.0C conductor in a magnetic field
B. electromotive force is induced in a circuit A. natural radioactivity
whenever there is a change in the magnetic B. nuclear fission
flux linked with the circuit C. nuclear fusion
C. the magnitude of the induced electromotive D. chain reaction
force in a circuit is proportional to the rate
of change of flux linking the circuit 48. Light of energy 5.0eV falls on a metal of
D. the induced current in a coil is in such a work function 3.0eV and electrons are
direction that it sets up a magnetic field emitted, determine the stopping potential.
which opposes the change producing it [electronic charge, e = 1.60 x 10-19C]
A. 1.7V
42. A bar magnet is placed near and lying along B. 2.0V
the axis of a solenoid connected to a C. 8.0V
galvanometer. The pointer of the D. 15.0V
galvanometer shows no deflection when
A. the magnet is moved away from the 49. The mass of a proton is 1.0074µ and that of
stationary solenoid a neutron is 1.0089µ. Determine the energy
B. the solenoid is moved away from the evolved in stabilizing the nucleus of
stationary magnet nitrogen of a mass number 14 with 7
C. the magnet is moved towards the stationary protons and 7 neutrons.
solenoid [speed of light = 3.0 x 108 ms-1]
D. there is not relative motion between the [1µ = 1.67 x 10-27kg]
magnet and the solenoid

43. An electric motor is a machine that converts


A. mechanical energy into electrical energy 50. Uranium of atomic number 92 and mass
B. electrical energy into mechanical energy number 238 emits an alpha particle from its
C. high voltage to low voltage nucleus. The new nucleus formed has
D. low voltage to high voltage respectively, atomic number and mass
number.
44. Which of the following statements is not a A. 94 and 238
reason for using soft iron in making the core B. 90 and 236
of a transformer? It C. 88 and 234
A. retains its magnetism for a long time D. 90 and 234
B. is easily magnetised
C. reduces energy loss due to hysteresis
D. is easily demagnetised

45. Which of the following statements is an


advantage of nuclear fusion over nuclear
fission?
A. The resulting chain reaction which produces
large amount of energy
B. No radioactive waste is produced as by-
product
C. Very high temperatures required in the
process
D. The end products is lead

46. Beta particles are fast-moving


A. protons
B. electrons
C. neutrons
D. photons

47. A process by which two light atoms join to


form a new atom of higher atomic mass is
2005
ESSAY SECTION B PART I

1. A stone thrown horizontally from the top of


a vertical wall with a velocity of 15ms-1, hits
the horizontal ground at a point 45 m from
the base of the wall. Calculate the height of
the wall, [g = 10ms-2]

2. A ball is projected horizontally from a height


of 20m above the ground with an initial
velocity of 0.4ms-1. Calculate the horizontal
distance moved by the ball before hitting
the ground. [g= 10ms-2]

3. Describe, with the aid of a diagram, how a


wave can be plane polarized?

4. (a) Explain cations


(b) Draw and label an electrolytic cell.

5. State
(a) the principle upon which the
lightning in
fluorescent tubes operate;
(b) two factors on which the colour of
light from a fluorescent tube depend.

6. (a) Define Young modulus of elasticity


(b) A spiral spring extends from a length
of 10.00cm to 10.0Icm when a force
of 20N is applied on it. Calculate the
force constant of the spring.

7. (a) Define elastic limit


(b) State Hooke’s law of elasticity heat of fusion of ice can be
determined by the method of
8. (a) Define surface tension mixtures.
(b) State two methods by which the (ii) State two precautions to be
surface tension of a liquid can be taken to ensure accurate
reduced. results.
(c) Steam, at 100oC, is passed into a
9. Using the kinetic theory of matter, explain container of negligible heat capacity,
the definite structure of solids. containing 20g of ice and 100g of
water at 0oC, until the ice is
10. Explain wave-particle paradox. completely melted. Determine the
total mass of water in the container.
PART II [specific latent heat of steam = 2.3 x
103 Jg-1]
11. (a) Sketch a diagram of a simple [specific latent heat of ice = 3.4 x
pendulum 102Jg-1]
performing simple harmonic motion [specific heat capacity of water =
and indicate positions of maximum 4.2Jg-1 K-1]
potential energy and kinetic energy.
(b) A body moving with simple harmonic 13. (a) (i) Explain refraction of a wave;
motion in a straight line has velocity, (ii) critical angle
v and acceleration, a, when the (b) State two conditions necessary for
instantaneous displacement, x in cm, (i) total internal reflection of a
from its maximum position is given wave to occur;
by x = 2.5 sin 0.4 π t, where t is in (ii) interference wave patterns to
seconds. be formed.
Determine the magnitude of the (c) The distance between two
maximum successive crests of a water wave
(i) velocity traveling at 3.6ms-1 is 0.45m,
(ii) acceleration calculate the frequency of the wave.
(d) A ray of light is incident at an angle
(c) A mas m attached to a light spiral of 30o at an air-glass interface.
spring is caused to perform simple (i) Draw a ray diagram to show
harmonic motion of frequency f = ½ the deviation of the ray in the
π k / m , where k is the force glass.
constant of the spring. (ii) Determine the angle of
(i) Explain the physical deviation [Refractive index of
glass = 1.50]
significance of k / m
(ii) If m = 0.30kg, k = 30N m-1 14. (a) (i) Explain
and the maximum (I) electric potential
displacement of the mass (II) electric potential
from the equilibrium position energy
is 0.015m, calculate the (ii) State the SI unit of each of
maximum the terms in (a) (i) above.
(I) kinetic energy of the (b) An isolated electrically-charged
system; sphere of radius, r, and charge. Q, is
(II) tension in the spring supported on an insulator in air of
during the motion. permittivity Eo. Write down
[g = 10ms-2, π = (i) an expression for the electric
3.142] field intensity on the surface
of the sphere;
12. (a) Explain specific latent heat. (ii) an expression for the electric
(b) (i) Describe how the specific potential at the surface of the
latent sphere;
(iii) a relationship between the
electric field intensity and the
electric potential at the
surface of the sphere.
(c) The plates of a parallel plate
capacitor, 5.0 x 10-3m apart are
maintained at a potential difference
of 5.0 x 104V. Calculate the
magnitude of the
(i) electric field intensity
between the plates;
(ii) force on the electron;
(iii) acceleration of the electron;
[electronic charge = 1.60x10-
19
C]
[mass of electron = 9.1x10-
31
kg]

15. (a) State two


(i) differences between nuclear
fusion and nuclear fission;
(ii) peaceful uses of atomic
energy
(b) (i) Explain chain reaction
(ii) State
(I) one condition
necessary or chain
reaction to occur
(II) two components in a
nuclear reactor used
to control chain
reaction.

2005
(c) (i) A nuclear reaction is given by PAPER I PRACTICAL
2H + 3H = 4He + 1n + energy
1 1 2 0 1. You have been provided with a retort stand,
What type of nuclear reaction clamp and boss, a set of masses, a spiral
is spring, stop watch, split cork and other
it? necessary apparatus.
(ii) The isotope of a nuclide has a
half life of 5.40 x 103g. Use the diagram below as a guide, carry out
Calculate its decay constant. the following instructions
(i) Suspend the spiral spring
vertically as shown in the
diagram.
(ii) Suspend a man hanger on the
free end of the spiral spring
and add a mass, m = 50.0g to
the hanger.
(iii) Pull the hanger gently
downwards and release to set
it into vertical oscillations.
(iv) Determine the time, t, for 20
complete oscillations.
(v) Evaluate the period, T, of the
oscillation. Also evaluate T2.
(vi) Repeat the procedure for four
other values of m = 70, 90,
110 and 130g. In each case,
determine t and evaluate T
and T2.
Tabulate your readings.
(vii) Plot a graph of T2 on the
vertical axis against m on the
horizontal axis.
(viii) Determine the slope, s, of the
graph and the intercept, I, on
the vertical axis.
(ix) Evaluate k = 4 π 2/S [Take π =
22
/7]

(b) (i) State four factors on which


the
resistance of a wire depend.

2006
(ii) A resistance wire of length
SSCE PHYSICS
100cm is connected in a
PAPER 2 OBJECTIVE
circuit. If the resistance per
unit length of the wire is
1. Which of the following phenomena causes
0.02Ωcm-1, how much heat
capillarity of liquids in tubes of narrow bore?
would be produced in the wire
A. Viscosity
if a voltmeter connected
B. Surface tension
across its ends indicates 1.5V
C. Osmosis
while the current runs for 1
D. Brownian motion
minute?
2. Which of the following physical quantities
are derived?
I. Area
II. Thrust
III. Pressure
IV. Mass
A. I, II, III, and IV
B. I, II, and III only
C. I, II, and IV only
D. I and III only D. 0.5 ms-1

3. A block of volume 3 x 105m3 and density 10. speed


2.5 x 103kg m-3 is suspended from a spring
v/ms-1
balance with 2/3 of its volume immersed in
a liquid of density 900kg m-3. Determine the
reading of the spring balance [g = 10 ms-2]
A. 0.18 N
B. 0.57 N
C. 0.75 N 0 5 20 25 t/s
D. 0.93 N
The diagram above shows the speed-time
4. A solid body will float in a liquid if its graph of a car. If the car covered a total
A. density is less than that of the liquid distance of 600m in 25s, calculate the
B. mass is equal to that of the liquid maximum speed.
C. density is greater than that of the liquid A. 30 ms-1
D. mass is less than that of the liquid B. 25 ms-1
C. 20 ms-1
5. An object is said to undergo oscillatory D. 15 ms-1
motion when it moves
A. in an erratic manner 11. Which of the following quantities is a
B. to and fro about a fixed point vector?
C. in a circular path A. Speed
D. along a continuous path from the starting B. Distance
point C. Energy
D. Momentum
6. A moving car of mass 800kg experiences a
frictional force of 200N. If it accelerates at
2ms-2, calculate the magnitude of the force
applied to the car.
A. 600 N
B. 1000 N
C. 1600 N
D. 1800 N

7. A body is rotating in a horizontal circle of


radius 2.5m with an angular speed of 5 and 12. P
s-1. Calculate the magnitude of the radial
acceleration of the body 30O
A. 62.50 ms-2
B. 31.25 ms-2 40N
C. 12.50 ms-2
20cm 30cm
D. 10.00 ms-2

8. Uniform speed occurs when there is equal


change of Determine the magnitude of P in the
A. distance in equal times diagram above.
B. displacement in equal times A. 16 N
C. velocity in equal times B. 20 N
D. acceleration in equal times C. 32 N
D. 40 N
9. A car accelerates uniformly from rest at 5
ms-2. Determine its speed after 10s. 13. Which of the following statements about
A. 50.0 ms-1 simple harmonic motion is correct? The
B. 25.0 ms-1 A. total mechanical energy is always
C. 2.0 ms-1 conserved
B. linear acceleration is directed to any 20. The velocity ratio of an inclined plane,
variable point inclined at an angle θ to the horizontal can
C. linear acceleration varies inversely with be expressed as
displacement A. (sin θ -1)
D. period of oscillation varies linearly as
B. sin θ
acceleration due to gravity
C. (cos θ )-1
14. A force on a body causes a change in the D. cos θ
momentum of the body from 12 kg ms-1 to
16 kg ms-1 in 0.2 s. Calculate the 21. A block and tackle system has six pulleys. A
magnitude of the impulse. force of 50 N applied to it lifts a load of
A. 28.0 Ns weight W. If the efficiency of the system is
B. 20.0 Ns 40%, calculate W.
C. 4.0 Ns A. 300 N
D. 0.8 Ns B. 200 N
C. 140 N
15. Which of the following sources of energy is D. 120 N
renewable?
A. Petroleum 22. The sagging of overhead electrical cables is
B. Charcoal the consequence of
C. Hydro A. linear expansivity
D. Nuclear B. superficial expansivity
C. cubic expansivity
16. A stone of mass 2.0kg is thrown vertically D. conductivity
upwards with a velocity of 2.0ms-1, calculate
the initial kinetic energy of the stone. 23. The silvered surface in a vacuum flask
A. 200 J reduces heat loss due to
B. 400 J A. conduction
C. 800 J B. convection
D. 1600 J C. radiation
D. evaporation
17. Which of the following statements about the
mouth piece of a telephone is correct? It 24. How long will it take to heat 2kg of water
converts sound energy into from 28oC to 88oC in an electric kettle taking
A. acoustic energy 6A from a 220V supply? [specific heat
B. mechanical energy capacity of water = 4180 J kg-1 k-1]
C. electrical energy A. 120 s
D. heat energy B. 570 s
C. 600 s
18. The potential energy in an elastic string of D. 836 s
force constant k which has been extended
by x metres is expressed as 25. The amount of energy required to change a
A. ½ kx2 kilogram of ice block into water without a
B. kx2 change in temperature is
C. ½ kx A. head capacity of ice
D. kx B. specific heat capacity
C. specific latent heat of vaporization of ice
19. The engine of a train produces a force of D. specific latent heat of fusion of ice
3000 N when moving at 30ms-1. Calculate
the power of the engine. 26. A quantity of stem at 100oC considenses to
A. 1.00 x 102 W water at the same temperature by releasing
B. 3.00 x 104 W 6.9 x 104J of energy. Calculate the mass of
C. 9.00 x 104 W the condensed steam. [specific latent heat
D. 3.00 x 105 W of vapourization of water = 2.3 x 106 J kg-1]
A. 3.0 x 10-1 kg
B. 3.0 x 10-2 kg
C. 3.0 x 10-3 kg is rotated through 30o, by how many
D. 3.0 x 10-4 kg degrees would the reflect ray rotate?
A. 60o
27. The temperature of a liquid falls after some B. 50o
of it has evaporated because C. 40o
A. the liquid left is smaller in volume and the D. 30o
breeze easily cools it
B. the more energetic molecules have escaped 34. Which of the following distances is usually
into the atmosphere adjustable in the camera? The distance
C. the liquid left has latent heat between the
D. after evaporation impurities are left behind A. Lens and the film
B. diaphragm and the shutter
28. The S.I. units of frequency, period and C. shutter and the film
amplitude of a wave are respectively D. diaphragm and the film
A. hertz, second and centimeter
B. second, metre and hertz 35. A traveling microscope is focused on a mark
C. metre, hertz and second on a table. If a glass block of thickness
D. hertz, second and metre 18.0cm is placed on the mark, by how many
centimeters will the microscope be moved
29. A note of frequency 2000 Hz has a velocity upwards so as to focus the mark once
of 400 ms-1. Calculate the wave length of again? [refractive index of glass = 1.5]
the note. A. 27 cm
A. 5.0 m B. 24 cm
B. 2.0 m C. 9 cm
C. 0.5 D. 6 cm
D. 0.2
36. In which of the following media would sound
30. The eclipse of the moon occurs when the wave travel faster?
A. moon comes exactly between the earth and A. wood
the sun B. Water
B. earth comes exactly between the moon and C. Iron
the sun D. Mercury
C. sun comes exactly between the earth and
the moon
D. moon reflects all the rays from the sun onto 37. Which of the following actions is necessary
the earth in order to tune a string to produce a note of
31. Which of the following observations cannot an octave higher than its fundamental?
be explained using the rectilinear A. Doubling the length of the string
propagation of light? B. Reducing the tension by half
A. Production of images by a pinhole camera C. Reducing the mass per unit length
B. Production of real images of objects by D. Increasing the tension four times
lenses
C. Formation of an annular eclipse 38. The acceleration due to gravity may be
D. Diffraction patterns of light defined as the force
A. of attraction of the sun on the earth
32. The distance between the successive crests B. with which the earth revolves around the
of a wave traveling at 20ms-1 is 25cm. sun
Calculate the frequency of the wave. C. with which the earth attracts one-kilogram
A. 0.8 Hz mass
B. 5.0 Hz D. of the moon on the earth
C. 50.0 Hz
D. 80.0 Hz 39. An electron of mass m and charge enters a
uniform electric field between two metal
33. A ray from a fixed object is incident on a plates P and Q separated by a distance d. P
plane mirror at an angle of 20o. If the mirror is maintained at a potential V while Q is
earthed. Determine an expression for the
magnitude of the acceleration of the I. moving coil ammeter
electron through the field. II. electric bell
A. eV/md III. electric motor
B. d/meV Which of the statements above are correct?
C. md/eV A. I and II only
D. e/Vmd B. II and III only
C. I and III only
40. Which of the following factors doe not D. I, II and III
affect the capacitance of a parallel plate
capacitor? 46. the breaking up of an atomic nucleus into
A. Area of the plates two fragments of nearly equal sizes is
B. Nature of the surface of the plates known as
C. Distance of separation between the plates A. nuclear fusion
D. Nature of the insulating material between B. nuclear fission
the plates C. thermonuclear reaction
D. natural radioactivity
41. The equivalent capacitance of a 3µF
capacitor and a 6µF capacitor connected in 47. A 500 kV is applied across an Ex-ray tube.
parallel is Calculate the maximum velocity of the
A. 2.0µF electrons produced. [Me = 9.1 x 10-31kg, e =
B. 3.0µF 1.6 x 10-19C]
C. 4.5µF A. 4.2 x 108 ms-1
D. 9.0µF B. 1.8 x 108 ms-1
C. 4.2 x 105 ms-1
42. A cell of e.m.f. 1.5V and internal resistance D. 1.8 x 105 ms-1
1.0Ω is connected to two resistors of
resistances 2.0Ω and 3.0Ω in series. 48. A radioactive substance of mass 768g has a
Calculate through the resistors. half life of 3 years. After how many years
A. 0.25 A does this substance leave only 6g
B. 0.30 A undecayed?
C. 0.35 A A. 21
D. 0.50 A B. 18
C. 15
43. An electric lamp rated 120W in used on a D. 12
240 Vrms, calculate the resistance of its
filament. 49. Fusion is not usually used for generating
A. 480Ω electric power because
B. 360Ω A. no energy is released when fusion takes
C. 60Ω place
D. 2Ω B. very high temperatures are required to
initiate fusion reaction
44. Which of the following statements about a C. the raw materials needed are not easily
bar magnet is correct? available
A. Iron filings cling uniformly along the length D. heavy nuclei are involved
of the bar magnet
B. The magnetic properties are more 50. An atom 234
P emits a gamma radiation. The
pronounced at the middle of a magnet 91

C. It is impossible to have the same poles at Resultant nuclide is


the ends of a magnet A. 234
Th
D. The magnetic properties are more 90
pronounced at points near the ends of the
bar
B. 234
Ac
89
45. The force experienced by a current-carrying
conductor moving in a magnetic field is
employed in the working of the C. 230
Th
90
D. 234
Pa
91

2006
THEORY SECTION B PART I

1. (a) Explain the statement the


acceleration of
free fall due to gravity on the
equator is 9.78 ms-2.

(b) State two factors that can affect the


value of the acceleration of free fall
at a place.

2. A particle is projected horizontally at 10ms-1


from a height of 45m. Calculate the
horizontal distance covered by the particle
before hitting the ground. [g = 10ms-2]
3. A particle dropped from a vertical height h 10. List three observations in support of the de-
falls freely for a time interval t. Sketch and Broglie’s assumptions that moving particles
explain a graph to show how h varies with behave like waves.
(a) t; (b) t2.
PART I FOR CANDIDATES IN GHANA ONLY
4. The diagram below represents the graph of
the force applied in stretching a spiral 1. State the dimensions of
spring against the corresponding extension (a) power (b) momentum (c) density
produced within its elastic limit.
2. (a) how many strokes are there in a
petrol car
engine?
(b) List the strokes

3. Sketch and label the cooling curve of liquid


naphthalene as it cools from 85oC to 30oC.

Using the notations on the graph, determine 4. (a) Write down the Van der Waals
the equation of
(a) force constant state.
(b) work done in stretching the spring (b) To what type of gas does the
from equation in 4(a) above apply?
10 x 10-2m to 20 x 10-2m (c) Re-write the equation in 4(a) above
for gas at very low pressure.
5. (a) Distinguish between the forces of
adhesion and cohesion 5. Explain how audible beats arise when two
(b) Give one example each of the forces tuning forks of slightly different frequencies
of adhesion and cohesion. f1 and f2 are sounded together

6. Explain why water in a narrow glass tube 6. (a) What is doopler effect?
has a concave meniscus while mercury, in (b) State one practical situation in which
the same tube, has a convex meniscus. doopler effect is experienced.

7. (a) State two applications of 7. (a) What is a loop in an electrical


electrolysis. network?
(b) Explain what is meant by the (b) State, in words, Kirchhoff’s second
electrochemical equivalent of copper law of electrical network.
is 3.3 x 10-7kgC-1. (c) Which of the conservation laws does
the law in 7(b) above express?
8. The uncertainty in determining the duration
during which an electron remains in a 8. The magnitude of the force of attraction
particular energy level before returning to between two bodies is directly proportional
the ground state is 2.0 x 10-9s. Calculate the to the product of their masses, M1 and M2,
uncertainty in determining its energy at and inversely proportional to the square of
that level. [Take h/2 π = h = 1.054 x 10-34 their distance, d, apart. Determine the S.I.
Js]. unit of the universal gravitational constant,
G using dimensional analysis.
9. A parallel beam of unpolarized light is
incident on a plane glass of refractive index 9. List three devices in everyday life which
1.60 at an angle to the normal. If the operate on the deflection of electric charges
reflected beam is completely polarized, in a magnetic field.
calculate the angle of incidence of the
beam. 10. Sketch a graph to show the I-V
characteristics of a p-n junction diode.

PART II FOR ALL CANDIDATES


image would be the same size
11. (a) State the conditions for the as the object.
equilibrium of
a rigid body acted upon by parallel 14. (a) (i) Name and explain the
forces common
(b) (i) Describe an experiment to Defects of a primary cell.
determine the mass of a (ii) State two advantages of a
metre rule using the principle secondary cell over a primary
of moments. cell.
(ii) State two precautions (b) Draw a labeled diagram to show the
necessary to obtain accurate essential parts of a dry leclanche
results in the experiment cell.
described in 11(b)(i) above. (c) (i) Explain why six accumulators
(c) A bullet of mass 120g is fired each of e.m.f.2V connected
horizontally into a fixed wooden in series can be used to start
block with a speed of 20ms-1. If the the engine of a car whereas
bullet is brought to rest in the block eight dry cells each of e.m.f.
in 0.1 s by a constant resistance, 1.5V connected in series
calculate the cannot be used.
(i) magnitude of the resistance (ii) Name the materials used for
(ii) distance moved by the the positive terminal, the
bullet in the wood. negative terminal and the
electrolyte in a
12. (a) State two (I) leclanche cell;
(i) laws of solid friction; (II) charged lead acid
(ii) advantages of friction accumulator
(iii) methods of reducing friction
(b) Draw and label a diagram of a pulley 15. (a) Define
system with a velocity ratio of 5. (i) proton number
(c) (i) Show that the efficiency ε, (ii) nucleon number
the (iii) isotopes
force ratio M.A. and velocity (b) A nuclide ZAX emits β -particles to
ratio V.R. of a machine are form a daughter nuclide Y. Write a
related by the equation ε = nuclear equation to illustrate the
M.A.
/V.R x 100% charge conservation.
(ii) The efficiency of a machine is (c) The radioactive nuclei 210 Po emits an
80%. Calculate the work done 84 α
particle to
by a person using the produce 206
Pb. Calculate the energy,
machine to raise a load of in MeV, 82
released in each
300kg through a height of disintegration.
4m. [g = 10ms-2]
Take the masses of 210Po = 209.936730 u;
13. (a) With the aid of ray diagrams, explain 84
total 206Pb = 205,929421 u;
82
internal reflection. 4He = 4,001504 u,
(b) Describe, with the aid of a laebelled 1
diagram, the essential features of an And that 1u = 931 MeV.
astronomical telescope in normal
adjustments.
(c) A converging lens forms a real image
of a real object. If the magnification
is 2 and the distance between the
image and the object is 90.0cm,
determine the 2006
(i) focal length of the lens; PAPER I PRACTICAL
(ii) object distance for which the
2.

1. You have been provided with a metre rule, a


clamp and set of masses.
(i) Clamp the metre rule to the
edge of the bench such that
90cm of the rule projects You have been provided with a rectangular
from the edges as shown in glass prism, optical pins and other
the diagram above. Ensure necessary apparatus. Using the above
that the rule is capable of diagram as a guide, carry out the following
performing oscillatory motion. instructions:
(ii) Fix a mass M = 50g at the (i) fix the drawing paper
free end of the rule. provided to the drawing
(iii) Deflect the rule slightly such board;
that it performs vertical (ii) place the glass prism on the
oscillation. drawing paper and trace the
(iv) Determine the time t for 10 outline, ABCD of the prism;
complete oscillations. (iii) remove the prism, mark a
(v) Calculate the period T of the point O and AB such that AO
oscillations and evaluate T2. is about one-quarter of AB;
(vi) Repeat the procedure for (iv) draw a normal through point
four other values of M = 100, O. Also draw an incident ray
150, 200 and 250g. In each to make an angle i= 25o with
case determine and record the normal at O. Fix two pins
the corresponding values of t, at P1 and P2 on the incident
T and T2. Tabulate your ray;
readings.
(v) replace the prism. Fix two
(vii) Plot a graph of T2 on the other pins at P3 and P4 such
vertical axis against M on the that the pins appear to be in a
horizontal axis, starting both straight line with the images
axes from the origin (0,0). of the pins at P1 and P2 when
(viii) Determine the slope s, of the viewed through the clock
graph and its intercept C on along DC;
the vertical axis.
(vi) remove the prism. Join points
(ix) Evaluate k = 4 π /s. [Take π P3 and P4 and produce it to
= 22/7]. meet DC at I. Also draw a line
(x) From your graph, determine to join OI;
the period T, when M = 180g. (vii) with O as centre and using
(xi) State two precautions taken any convenient radius, draw a
to ensure accurate results. circle to cut the incident ray
(b) (i) Explain simple harmonic and the refracted ray at E and
motion. H respectively. Maintain this
(ii) Define period and frequency, radius through-out the
with respect to a simple experiment.
harmonic motion. (viii) Draw the perpendiculars EF
and GH. Measure and record
d = EF and l = GH;
(ix) Repeat the procedure for four
other values of i=35o, 45o,
55o, and 65o respectively. In
each case measure and (viii) Determine the slope, s, of the
record d and l graph and its intercept, c on
(x) Plot a graph of d on the the vertical axis.
vertical axis against l on the (ix) Evaluate k = c/s
horizontal axis; (x) using your graph, determine
(xi) Determine the slope of the the current I when I = 55cm
graph; (xi) State two precautions taken
(xii) State two precautions taken to ensure accurate results.
to ensure accurate results.
[Attach your traces to your (b) (i) Explain what is meant by the
answer booklet] potential difference between two
(b) (i) State Snell’s law. points in an electric circuit.
(ii) Calculate the critical angle for (ii) State two factors on which the
a water-air interface. resistance of a resistance wire
[refractive index of water = depends.
4/3]

3. (a)

You are provided with a constantan wire, a


2Ω standard resistor, an accumulator E, and
ammeter A, a key K and other necessary
apparatus.
(i) Measure and record as shown
in the diagram above.
(ii) Connect a circuit as shown in
the diagram above.
(iii) Close the key, read and
record the ammeter reading
Io when the crocodile clip is
not in contact with the
constantan wire.
(iv) Open the key. With the clip
making contact with the wire,
when I = 90cm, close the key.
Read and record the ammeter
reading I, Evaluate I-1.
(v) Repeat the procedure for I =
80, 70, 60 and 50cm.
(vi) In each case, read and record
the ammeter reading and
evaluate
K-1. Tabulate your readings.
(vii) Plot a graph of I on the
vertical axis against I-1 on the
horizontal axis;
B. U + F – mg = 0
C. U – F = mg
D. F – U – mg = 0

6. A body moving at a constant speed


accelerates when it is in
A. rectilinear motion
B. translational motion
2007 C. circular motion
SSCE PHYSICS D. vibrational motion
PAPER 2 OBJECTIVE
7. An object of mass 2kg moves with a uniform
1. Which of the following observations is not speed of 10ms-1 for 5 s along a straight
an effect of surface tension? path. Determine the magnitude of its
A. Droplets of water dripping slowly from a tap acceleration.
B. Mercury spilled on a clean glass plate forms A. 0 ms-2
small spherical droplets B. 1 ms-2
C. An insect walking across the surface of a C. 3 ms-3
pond D. 4 ms-2
D. Water flowing out more easily than engine
oil from container 8. The term rectilinear acceleration means the
rate of increase of
2. Which of the physical quantities is correctly A. velocity along a circular path in a unit time
paired with its corresponding S.I. unit? B. distance along a rectangular path in a unit
A. Density [kg m-1] time
B. Power [J s-1] C. displacement along a straight-line path in a
C. Specific latent heat [J kg-1k-1] unit time
D. Pressure [N m-1] D. velocity along a straight path in a unit time

3. A liquid of volume 2.00m3 and density 1.00 9. A vehicle moving with a uniform speed is
x 103kg m-3 is mixed with 3.00m3 of another gradually brought to test. Which of the
liquid of density 8.00 x 103 kg m-1. Calculate following sketches describe the motion of
the density of the mixture the vehicle?
A. 5.20 x 103 kg m-3
B. 8.80 x 103 kg m-3 A. B. C.
C. 1.13 x 103 kg m-3 speed speed speed
D. 5.20 x 103 kg m-3

4. Which of the following statements about


solid friction are correct? It
I. is a force
time time time
II. occurs between the surfaces of two bodies
in contact
III. depends on the area of contact
A. I and II only
B. I and III only
C. II and III only D.
speed
D. I, II and III

5. A man of mass m, experiences a viscous


drag. F and an upthrust. U as he descends
towards the ground at a steady velocity, time
using a parachute. If the acceleration of free
fall is g, which of the following expression is
correct?
A. FU = mg
10. A body accelerates uniformly from rest at 2 14. A block of mass 4.0kg causes a spiral spring
ms-1. Calculate the magnitude of its velocity to extend by 0.16 m from its unstretched
after traveling 9 m. position. The block is removed and another
body of mass 0.50kg is hung from the same
A. s= v–u I spiral spring. If the spring is then stretched
2 and released, what is the angular frequency
of the subsequent motion?
B. s = v + 2u I [g = 10 ms-2]
2 A. 10 √ 5 rad s-1
B. 5 √ 2 rad s-1
C. 5 rad s-1
C. s= v+u I D. √5 rad s-1
2
15. An external force of magnitude 100N acts
on a particle of mass 0.15kg for 0.03s.
D. s = 2V + u I Calculate the change in the speed of the
2 particle.
A. 50 ms-1
B. 25 ms-1
11. A body accelerates uniformly from rest at 2 C. 20 ms-1
ms-2. Calculate the magnitude of its D. 5 ms-1
velocity after traveling 9 m.
A. 4.5 ms-1 16. A car of mass 800kg moves from rest on a
B. 6.0 ms-1 horizontal track and travels 60 m in 20s with
C. 18.0 ms-1 uniform acceleration. Assuming there were
D. 36.0 ms-1 no frictional forces, calculate the
accelerating force.
12. 10N A. 240.00 N
B. 800.00 N
C. 1600.00 N
D. 2400.00 N

17. A plane inclined at 30o to the horizontal has


an efficiency of 50%. Calculate the force
parallel to the plane required to push a load
30o 0 F of 120N uniformly up the plane.
A. 50.0 N
P
B. 120.0 N
C. 200.0 N
The diagram above illustrates three forces D. 240.0 N
acting on an object at point O. If the object
is in equilibrium, determine the magnitude 18. Mechanical energy can be either
of the force P. A. kinetic or electric
A. 10.5N B. chemical or potential
B. 11.0N C. potential or kinetic
C. 17.3N D. electric or heat
D. 20.0N
19. The ice and steam points on a mercury-in-
13. A swinging pendulum between the rest glass thermometer are 10cm and 30cm
position and its maximum displacement respectively. Calculate the temperature in
possesses degree celius when the mercury meniscus is
A. kinetic energy only at the 14cm mark.
B. potential energy only A. 20oC
C. gravitational energy only B. 30oC
D. both kinetic and potential energy C. 34oC
D. 70oC
following diagrams correctly illustrates the
20. Which of the following surfaces will radiate temperature ( θ ) versus time (t) graph for
heat energy best? this process?
A. Red surface
B. White surface A. θ /oC B. θ /oC
C. Black surface 150 150
D. Yellow surface
100 100
97 97
21. Which of the following statements
95 95
distinguishes thermal conduction from
convection? t/s t/s
I. Conduction requires a material medium
while convection does not
II. In convection, there is actual motion of hot
material, while in conduction, molecules
C. θ /oC D. θ /oC
vibrate faster about their mean positions
150 150
III. Conduction takes place in solids while
convection takes place in fluids 100 100
A. I and II only 97 97
95 95
B. II and III only
C. I and III only t/s t/s
D. I, II and III

22. A cube made of a metal of linear 26. According to the kinetic theory of gases,
expansivity R is heated through a which of the following properties of a gas
temperature θ . If the initial volume of the can be affected by the collision of the gas
cube is Vo, the correct expression for the molecules with the walls of its container?
increase in volume of the cube is A. Temperature
A. 1/3 Vo θ B. Energy
B. 1/3 Vo θ C. Viscosity
D. Pressure
C. 2 Vo θ
D. 3 Vo θ 27. The pressure of fixed mass of an ideal gas
at 27oC is 3 Pa. The gas is heated at a
23. A relative density bottle of volume 50cm3 is constant volume until its pressure is 5Pa.
completely filled with a liquid at 30oC. It is Determine the new temperature of the gas.
then heated to 80oC such that 0.75cm-3 of A. 100oC
the liquid is expelled. Calculate the B. 227oC
apparent cube expansivity of the liquid. C. 273oC
A. 0.00030 k-1 D. 500oC
B. 0.00032 k-1
C. 0.01970 k-1 28. Which of the following waves is
D. 0.02030 k-1 electromagnetic?
A. X-rays
24. A body of mass 200g and specific heat B. Sound waves
capacity 0.4Jg-1 k-1 cools from 37oC to 31oC. C. Water waves
Calculate the quantity of heat released by D. Tidal waves
the body.
A. 4800 J 29. The distance between two points P and Q
B. 1200 J along a wave is 0.05m. If the wave length of
C. 480 J the wave is 0.10m, determine the angle
D. 202 J between P and Q in radians.
A. 0.1 π
25. At standard atmospheric pressure, a B. π
solution of common salt is heated gradually C. 2π
until most of it boils away. Which of the D. 10 π
30. Which of the following phenomena is not a 36. A sounding tuning fork is brought near the
direct consequence of rectilinear open end of a pipe containing air and the
propagation of light? loudness of the sound is observed to
A. Lunar and solar eclipses increase. This observation is due to
B. Images of object in pinhole camera A. an echo
C. Diffraction of light B. resonance
D. Shadows of opaque objects C. interference
D. reverberation
31. An object is placed 10cm in front of a plane
mirror. If it is moved 8cm farther away from 37. Which of the following statement about
the mirror, determine the distance of the sound waves is not correct? Sound waves
final image from the mirror. can be
A. 2 cm A. reflected
B. 9 cm B. refracted
C. 16 cm C. diffracted
D. 18 cm D. polarized

32. In which of the following pieces of apparatus 38. Which of the following statements is correct
is the radius of curvature equal to twice the about resonance? Resonance occurs when
focal length? a body
A. Converging lens A. vibrates with its natural frequency
B. Diverging lens B. is forced to vibrate at its own natural
C. Converging mirror frequency by another body vibrating at the
D. Parabolic mirror same frequency
C. vibrates at the frequency of the body
33. When an object is placed at the centre of causing the vibration
curvature of a concave mirror, its image is D. vibrates with an irregular frequency
formed at?
A. the focus 39. A dry plastic comb used in combing hair was
B. infinity found to attract pieces of paper and dust.
C. the centre of curvature The most probable explanation for this
D. a distance twice the radius of curvature phenomenon is that the comb has been
given
34. A rectangular glass prism of thickness d and A. magnetization by induction
absolute retractive index n is placed on a B. electric charges by induction
point object, which is viewed vertically C. electric charges by conduction
downward from above the prism. Which of D. electric charges by friction
the following expressions correctly defines
the apparent upward displacement of the 40. Which of the following components is used
object? for storing electric charges?
A. d
/n A. Inductor
B. dn B. Resistor
C. d
/n-2 C. Capacitor
D. d(n – l) D. Electrometer
n
41. Which of the following operations does not
35. Which of the following factors can affect the represent an action of a force field?
speed of sound in air? A. Falling of a mango fruit from the tree
I. The temperature of the surrounding B. Picking of nails using a bar magnet
II. The direction of wind C. Repulsion of two like charges
III. The pitch of the sound D. Pushing of a wheel-barrow on a level ground
A. I and II only 42.
B. II and III only
C. I and III only
D. I, II and III
III. Potential energy of the target atoms
IV. Heat energy of the emitted electrons
A. IV, III, II, I
B. IV, II, III, I
Calculate the effective resistance between P C. II, IV, III, I
and Q in the diagram shown above. D. III, IV, II, I
A. 0.75 Ω
B. 4.00 Ω 49. In a nuclear reactor, chain reactions result
C. 5.00 Ω from the release of
D. 7.00 Ω A. electrons
B. photons
43. A bulb marked 240V, 40W is used for 30 C. neutrons
minutes. Calculate the heat generated. D. protons
A. 320 J
B. 400 J
C. 10800 J 50. There is always an uncertainty involved in
D. 72000 J any attempt to measure the position and
momentum of an electron simultaneously.
44. Five 80 – W and three 100 – W lamps are This statement is known as the
run for 8 hours. If the cost of energy is A. de Broglie’s law
N5.00 per unit, calculate the cost of running B. Heisenberg uncertainty principle
the lamps. C. Franck-Hertz experimental law
A. N280.00 D. Wave-particle paradox
B. N28.00
C. N7.20
D. N1.44

45. Which of the following electromagnetic


wave can be detected by its heating effect?
A. Ultraviolet radiation
B. X-rays
C. Gamma rays
D. Infrared radiation

46. If a gas is excited by high voltage to


produce a discharge, and the light is
examined in a spectrometer;
A. an explosion is observed
B. the rainbow colours are seen
C. an emission spectrum is observed
D. dark lines are observed

47. The number of protons in an element


increased by one after a radioactive decay.
The element must have decayed by
emitting
A. a beta particle
B. an alpha particle
C. a gamma ray
D. a neutron

48. the correction sequence of energy


transformation in the operation of an X-ray
tube is
I. X-rays
II. Kinetic energy of thermo electrons
(a) Tensile stress
(b) Tensile strain
(c) Yield point

8. A wire of length 5.0m and diameter 2.0mm


extends by 0.25mm when a force of 50N
was used to stretch it from its end.
Calculate the
(a) stress on the wire
(b) strain in the wire [ π = 3,142]

9. (a) List two factors that can affect the


2007 rate of
THEORY SECTION B PART I Diffusion.
(b) State two examples to illustrate the
1. A particle is projected at an angle of 30o to effects of surface tension.
the horizontal with a speed of 250ms-1. 10. State two effects to show
Calculate the (a) the existence of matter waves
(i) total time of flight of the (b) that radiation behaves like particles.
particle
(ii) speed of the particle at its PART II FOR ALL CANDIDATES
maximum height. [g = 10 ms-
1
] 11. (a) State the conditions of equilibrium
for a
2. A stone projected horizontally from the top number of coplanar parallel forces.
of a tower with a speed of 4 ms-1 land on the (b) A metre rule is found to balance
level ground at a horizontal distance 25m horizontally at the 48cm mark. When
from the foot of the tower. Calculate the a body of mass 60g is suspended at
height of the tower. [g = 10ms-1] the 6cm mark, the balance point is
found to be at the 30cm mark.
3. State: Calculate the
(a) the difference between plane (i) mass of the metre rule
polarized light and ordinary light. (ii) distance of the balance point
(b) two uses of polaroids. from
the zero end, if the body were
4. Define: moved to the 13cm mark.
(a) Electrolysis: (c) A man pulls up a box of mass 70kg
(b) Electrolyte; using an inclined plane of effective
(c) electrode: length 5m unto a platform 2.5m high
at a uniform speed. If the frictional
5. Copper of thickness d is plated on the force between the box and the plane
cathode of a copper voltameter. If the total is 100N.
surface area of the cathode is 60cm-2 and a (i) draw a diagram to illustrate
steady current of 5.0 A is maintained in the all the forces acting on the
voltameter for 1 hour, calculate the value of box while in motion.
polaroids. (ii) Calculate the:
[ density of copper = 8.9 x 103 kgm-3] I. minimum effort
[ electro chemical equivalent of copper = applied in pulling up
3.3 x 10-7 kgC-1] box;
II. velocity ratio of the
6. (a) Explain thermionic emission plane if it is included
(b) State two applications of electrical at 30o to the
conduction through gases. horizontal.
III. force ratio of the
7. Define: plane.
from a 4400V a.c. supply. Calculate
12. (a) Define the boiling point of a liquid. the
(b) With the aid of a sketch diagram, (i) ratio of the number of turns in
describe an experiment to determine the primary coil to the
the boiling point of a small quantity number of turns in the
of a liquid. secondary coil of the
(c) A piece of copper of mass 300g at a transformer.
temperature of 950oC is quickly (ii) current taken from the mains
transferred into a vessel of negligible circuit.
thermal capacity containing 250g of
water at 25oC. If the final steady 15. (a) (i) With the aid of a labeled
temperature of the mixture is 100oC, diagram,
calculate the mass of water that will describe the mode of
boil away. operation of a modern X-ray
[specific heat capacity of copper = tube.
4.0 x 10 J kg K] (ii) State the energy
[specific heat capacity of water = 4.2 transformations that take
x 10 J kg K] place during the operation of
[specific latent heat of vaporization the X-ray tube.
of steam = 2.26 x 10 J kg]. (b) Define, as applied to X-rays, the
following terms.
13. (a) (i) What is a wave motion? (i) Hardness
(ii) State two differences (ii) Intensity
between a radio wave and a (c) State:
sound wave. (i) Four uses of X-rays
(b) (i) Given that you are provided (ii) One hazard of over exposure to
with X-rays in a radio-logical
a tuning fork, a burette and laboratory
other necessary apparatus,
describe with the aid of a
diagram, an experiment to
determine the frequency of a
note emitted by a source
sound. [assume the velocity
of sound in air is known]
(ii) State two precautions
necessary to obtain accurate
result in the experiment
described in 13(b)(i) above.
(c) A pipe closed at one end is 100cm
long. If the pipe is set into vibration
and a fundamental note is produced,
calculate the frequency of the note.
[velocity of sound in air = 340 ms-1]

14. (a) With the aid of a simple diagram,


explain
How a step down transformer works.
(b) (i) State three ways by which
energy
is lost in a transformer
(ii) Mention how each of the
losses in (b)(i) above can be
minimized.
(c) A 95% efficient transformer is used
to operate a lamp rated 60 W; 220V
of suspension of the
pendulum.
(iii) Displace the pendulum
through a small angle and
release. Allow the pendulum
to oscillate freely.
(iv) Determine the time t for 20
complete oscillations;
(v) Also, determine the period T
of the oscillations;
(vi) Evaluate T2 and L = l – 30
(vii) Repeat the procedure for
four other values of l = 110,
90, 70 and 50cm
(viii) In each case determine t and
evaluate T, T2 and L. Tabulate
your readings;
(ix) Plot a graph of T2 on the
vertical axis against L on the
horizontal axis, starting both
axes from the origin (0,0)
(x) Determine the slope, s, of the
graph. Also determine the
intercept, C of the graph on
the T2 axis.
(xi) Evaluate:
I. k1 = 4 π 3
s
II. k2 = F/s [Take π =
22
/7]
(xii) State two precautions taken
2007
to ensure accurate results.
PAPER I PRACTICAL
(b) (i) What is meant by the period
1. (a)
of
oscillations of an oscillating
body?
(ii) Explain acceleration of free
fall due to gravity.

2. (a)

You are provided with a retort stand; clamp


and boss, a pendulum bob, a piece of thread
and other necessary apparatus. Carry out
the following experiment:
(i) set up the apparatus as
shown in the diagram above.
(ii) Measure and record the
Using the above diagram as a guide, carry
distance l=130cm from the
out the following experiment:
centre of the bob to the point
(i) place the equilateral
triangular glass prism on the
drawing paper. Trace the
outline ABC of the prism.
(ii) Remove the prism. Draw line
NO such that it makes an
angle i=25o with the normal
at point O on side AB.
(iii) Fix two pins R1 and R2
vertically on line no. Replace You have been provided with an
the prism on its outline: accumulator E, a standard resistor R3, two
resistance boxes RB1 and RB2, two keys K1
(iv) Place the reflecting surface of
and K2 and other necessary apparatus.
the plane mirror in contact
(i) Measure and record the e.m.f. of
with face AC of the prism.
the accumulator.
(v) Looking through face BC of (ii) Connect a circuit as shown
the prism, fix two other pins above.
at R3 and R4 such that the (iii) Set the resistance R, in the
pins appear to be in a straight
resistance boxes such that R in
line with the images of the
RB1 = R in RB2 = 1Ω
pins at R1 and R2;
(iv) With K open and K2 closed,
(vi) Remove the prism, the mirror
measure and record the potential
and the pins. Draw a line to
difference Vo across the standard
join points R3 and R4.
resistor Rx.
(vii) Produce R4R3 to meet line NO
(v) Close K1 and K2. Read and record
produced at T.
the potential difference V1 across
(viii) Measure and record the Rx
angles θ at T and e at D. (vi) Evaluate V1-1
(ix) Repeat the procedure for (vii) Repeat procedure (v) for four
four other values of i=30o, other values of R = 2, 3, 4 and
35o, 40o and 45o. In each case, 5Ω respectively. In each case,
measure and record the ensure that the value of R in RB1
corresponding values of θ and is equal to the value of R in RB2.
e. Tabulate your readings; (viii) Evaluate V1-1 in each case.
(x) Plot a graph of e on the Tabulate your readings.
vertical axis and θ on the
horizontal axis, starting both (ix) Plot a graph of V1-1 on the vertical
axes from the origin (0,0) axis against R on the horizontal
(xi) Determine the slope s, of the axis starting both axes from the
graph and the intercept on origin (0,0).
the horizontal axis.
(xii) Evaluate k = s-1. (x) Determine the slope, s, of the
(xiii) State two precautions taken graph and the intercept I on the
to ensure accurate results. vertical axis.
[Attach your traces to your
answer booklet]. (xi) Evaluate y = ¼.
(b) (i) State four characteristics of
the (xii) State two precautions taken to
image of an object formed by ensure accurate results.
a plane mirror.
(ii) State two conditions (b) (i) Explain what is meant by the
necessary for total internal potential
reflection to occur in a
medium. difference between two points in an
electric circuit.
3. (a)
(ii) A cell has an e.m.f. of 3V. When it is 2. Which of the following sets of quantities is
connected across a resistor of fundamental?
resistance 4Ω, a current of 0.5A A. Length, mass and time
passes through the circuit. Calculate B. Speed, length and time
the internal resistance of the cell. C. Speed, mass and distance
D. Distance, speed and time

3.

The diagram above illustrates a Hare’s


apparatus, e1,e2 represent densities and
h1, h2 heights of column of liquids. Which of
the following equations is correct?

A. h1 = h2 l 1
l 2

B. h1 = h2 l 2
l 2

C. h1 = l 1l 2

D. h1 = l 1
H2 l 2

4. Which of the statements about Archimedes’


principle is correct? The upthrust on a body
is a measure of the
A. mass of fluid displaced
B. weight of the body
C. volume of the body
D. weight of the fluid displaced
2008 5. Which of the following is/are in random
SSCE PHYSICS motion?
PAPER 2 OBJECTIVE I. Pollen grains in water
II. The molecules of hydrogen gas
1. If the air inside a rigid box is heated, the III. Fine chalk particles floating in air.
I. average speed of the molecules increases A. I and II only
II. pressure of the air increases B. I and III only
III. average separation of the molecules C. II and III only
increases D. I, II and III
Which of the statements above are correct?
A. I and II only 6. A car traveling with a uniform velocity of
B. II and III only 30ms-1 along a horizontal road overcomes a
C. I and III only constant frictional force of 600N. Calculate
D. I, II and III the power of the engine of the car
A. 18kW
B. 20kW
C. 180kW
D. 200kW
11. F3 F2
7. A rectangular block of dimensions 2.0m x
1.0m x 0.5m weighs 200N. Calculate the
maximum pressure exerted by the block on
a horizontal floor. A B
A. 100Nm-2
B. 200Nm-2
C. 300Nm-2
D. 400Nm-2 F1

8. d/m P The diagram above shows three forces F1, F2


and F3 which keep the bar AB in horizontal
equilibrium. Which of the following
equations is correct?
A. F3 = F1 + F2
B. F2 = F1 + F3
t/s C. F1 = F2 – F3
D. F1 = F2 + F3

The diagram above represents the 12. A mass attached to a string is moving in a
displacement – time graph of the motion of circular path. If the speed is doubled, the
an object thrown upwards. The velocity of tension in the string will be
the object at the point P is A. doubled
A. maximum B. halved
B. uniform C. four times as great
C. zero D. one-fourth as much
D. terminal
13. An object of mass 2kg moving with a
9. A very sensitive spring balance was used to velocity of 3cm-1 collides head-on with
determine the weight of an object at the another object of mass 1kg moving in the
north pole. When the same spring balance opposite direction with a velocity 4ms-1. If
was used to measure the weight of the the objects stick together after collision,
same object at the equator, it was found to calculate their common speed.
reduce. The explanation for this observation A. 0.60ms-1
is that B. 0.67ms-1
A. it is very hot at the equator C. 2.00ms-1
B. the spring balance has expanded D. 3.33ms-1
C. the acceleration of free fall due to gravity
varies with location 14. Which of the following devices converts
D. the mass of the body is reduce heat energy to electrical energy?
A. Transformer
10. A ship moving northward with speed VS is B. Dynamo
acted upon by a wind blowing the due west C. thermocouple
at a speed VW. Which of the following D. Thermostat
diagrams correctly indicates the velocity VR
of the wind relative to the ship? 15. A ball is dropped and it hits the floor at a
A. B. B. point A. It rebounds upwards to a point B.
While moving from A to B its
A. kinetic energy is increasing
B. potential energy is increasing
C. potential energy is decreasing
D. D. kinetic energy remains constant

16. An object of mass 0.25kg moves at a height


h above the ground with a speed of 4cm-1. If
its mechanical energy at this height is 12 J, A. 4.57 minutes
determine the value of h. [g = 10ms-1] B. 5.02 minutes
A. 0.8m C. 7.30 minutes
B. 4.0m D. 10.04 minutes
C. 4.8m
D. 5.6m 22. The working principle of a pressure cooker
is based on the
17. Which of the following equations for the A. increase in the volume of steam inside it
efficiency of a machine is correct? B. complete trapping of heat inside it
A. Efficiency = Velocity Ratio ___ x 100% C. increase in the pressure of the pot
Mechanical Advantage D. decrease in the pressure of the pot

B. Efficiency = Input__ x 100% 23. It takes a shorter time for a liquid to boil at
Output the top of a mountain than at the base
because at the top, the
C. Efficiency = Load distance__ x 100% A. temperature is higher
Effort distance B. pressure is lower
C. humidity is higher
D. Efficiency = Mechanical Advantage x D. temperature is constant
100%
Velocity Ratio 24. Light is considered as a transverse wave
because it travels
18. the design of the thermostat of an electric A. through materials without causing
iron is based on the disturbance of the medium
A. emission of electrons from metals when B. through space with constant speed
heated C. in a direction parallel to the plane
B. increase in size of metals when heated containing the electric and magnetic fields
C. increase in the density of metals when D. in a direction perpendicular to the plane
heated containing the electric and magnetic fields
D. change in the mass of metals when heated
25.
19. A metal rod of length 50cm is heated from
40oC to 80oC. If the linear expansivity of the
material is α , calculate the increase in
length of the rod (in metres) in terms of α
A. 20 α
B. 200 α
C. 2000 α The diagram above illustrates the sun,
D. 20000 α moon and earth in the same straight line
20. The cubic expansivity of a certain gas at during an eclipse. An observer at the point P
constant pressure is 1 K-1. If a given mass on earth observes
of the gas is A. annular eclipse
273 B. partial eclipse
Held at constant pressure and its volume at C. total eclipse
0oC is 273m3, determine the volume of the D. no eclipse
gas at 273oC.
A. 273m3 26. The letter F is placed in front of a plane
B. 546m3 mirror as illustrated in the diagram beside
C. 819m3
D. 1092m3

21. Water of mass 1.5kg is heated from 30oC to


80oC using an electric kettle which is rated Which of the following represents the image
5A, 230V. Calculate the time taken to reach observed?
the final temperature. [Specific heat
capacity of water = 4200 Jkg-1 K-1] A.
B. 33. A sound wave is produced from a source
and an echo is heard t seconds afterwards.
C. If d is the distance of the reflecting surface
from the source, v the speed, λ the
D. wavelength and T the period of the wave,
then

27. In the day time, it is possible to see under A. d=2t


shady areas such as under a tree, because λ
light has undergone
A. internal reflection
B. refraction
B. d= λt
C. diffraction 2T
D. diffuse reflection
C. d=2T
28. The distance between a concave mirror and λt
a object placed in front of it is 1.0m. If the
radius of curvature of the mirror is 4.0m, D. d= λ T;
the image formed will be 2t
A. 2.00m behind the mirror
B. 2.00m in front of the mirror 34. A negatively charged rod is held close to an
C. 1.25m in front of the mirror uncharged metal ball on an insulating stand.
D. 1.25m behind the mirror The ball will
A. be negatively charged
29. The image of an object formed by a convex B. be positively charged
mirror is C. have an excess of negative charges on the
A. real, erect and enlarged side nearest the rod
B. real, inverted and enlarged D. have an excess of positive charges on the
C. virtual, erect and diminished side nearest the rod
D. virtual, erect and enlarged
35. Two bodies each carrying a charge of 2.00
30. When viewed straight down, a fish in a pond x10-10C are 5cm apart. Calculate the
is magnitude of the force between the
A. farther below than it appears to be charges.
B. nearer than it appears to be [ 1_ = 9.00 x 109Nm2C-2]
C. found at exactly the same point it is seen 4π ε o
D. seen beside its image
A. 1.44 x 10-7 N
31. A converging lens and a screen are placed B. 7.20 x 10-9 N
20cm and 80cm respectively from an object C. 7.20 x 10-11 N
in a straight line so that a sharp image of D. 1.44 x 10-11 N
the object is formed on the screen. If the
object is 3cm high, calculate the height of 36. If the charge on an object is measured as
the image formed. 4.0 x 10-18C, how many excess electrons
A. 1cm does the object posses, given that the
B. 9cm charge of an electron is 1.6 x 10-19C?
C. 12cm A. 18
D. 15cm B. 19
C. 25
32. Which of the following defects of vision is as D. 37
a result of the eye ball being too long?
A. Long sight 37. Which of the following is a unit of the time
B. Astigmatism rate of flow of electric charges?
C. Short sight A. Coulomb
D. Loss of accommodation B. Ampere
C. Volt
D. Watt

38. A wire, 1.0m long and with cross-sectional


area 2.0 x 10-7m2 has a resistance of 0.1Ω. The diagram above illustrates a bar magnet
Calculate the electrical conductivity of the near a coil connected to a galvanometer.
wire When the magnet is rapidly moved towards
A. 2.0 x 107Ω-1m-1 the coil
B. 5.0 x 107Ω-1m-1 I. an e.m.f. is induced in the coil
C. 2.0 x 108Ω-1m-1 II. the galvanometer needle deflects
D. 5.0 x 108Ω-1m-1 III. the magnet is attracted by the coil
Which of the statements above are correct?
39. Three capacitors each of capacitance 1.5µF A. I and II only
are connected in series. The total B. II and III only
capacitance in the circuit is C. I and III only
A. 2.0µF D. I, II and III
B. 1.5µF
C. 0.6µF 45. A direct current of 5A flows through a 0.2H
D. 0.5µF inductor. Calculate the energy stored in the
inductor.
40. A voltmeter is a device that A. 0.5J
A. draws high current B. 1.0J
B. has a very low resistance C. 2.5J
C. has a very high resistance D. 5.0J
D. is able to measure both voltage and current
46. Both rays and X-rays
41. Which of the following actions will not I. can be produced by thermionic emission
increase the sensitivity of a moving coil II. have low frequencies
galvanometer? III. have short wavelengths
A. Using a strong temporary magnet Which of the statements above is/are
B. Increasing the area and umber of turns of correct?
the coil A. I only
C. Using a weak hair spring B. II only
D. Using a light pointer C. III only
D. I and III only
42. Fleming’s right hand rule is also called the
A. motor rule 47. When a radioactive substance undergoes a
B. dynamo rule beta decay, its
C. screw rule A. mass number decreases by 1
D. thumb rule B. atomic number decreases by 1
C. mass number increases by 1
43. A transformer is required to supply 12VFMS to D. atomic number increases by 1
operate a toy train set from a 240VFMS. If the
number of turns in the secondary coil is 48. Nuclear fission is preferred to nuclear fusion
100, calculate the number of turns required in the generation of energy because
in the primary coil. A. very high temperatures are required for
A. 5 fusion
B. 12 B. the raw materials for fusion are easily
C. 2000 obtained
D. 2400 C. energy obtained from fusion is relatively
smaller
44. D. the by-products of fusion are very
dangerous
49. Control in nuclear reactors is effected with
boron rods because the rods have the 2. In an electrolysis experiment, the ammeter
ability to records a steady current of 1A. The mass of
A. absorb electrons copper deposited in 30 minutes is 0.66g.
B. absorb neutrons Calculate the error in the ammeter reading.
C. accelerate electrons [Electrochemical equivalent of copper =
D. slow down neutrons 0.00033gC-1]

50. A radioactive element has a decay constant 3. The horizontal component of the initial
of 0.077s-1. Calculate its half-life. speed of a particle projected at 30o to the
A. 13.0s horizontal is
B. 9.0s 50ms-1. If the acceleration of free fall due to
C. 5.1s gravity is 10ms-2, determine its
D. 0.5s (a) initial speed;
(b) speed at the maximum height
reached.

4. Explain plane polarized light.

5. A ball bearing falls through a viscous liquid.


(a) Using a labeled diagram of a tall
vessel, show all the forces acting on
it.
(b) When will it attain terminal velocity?

Use the diagram below to answer question


6.

6. Camphor is a chemical that sublimes and


interacts with water reducing the surface
tension where it is put. Explain why the toy
boat illustrated above would move forward
with the camphor placed at the back of it
but would be stationary before the
placement of the camphor.

7. An electron of charge 1.60 x 10-19C is


2008 accelerated under a potential difference of
PAPER 2 THEORY SECTION B 1.0 x 105V. Calculate the energy of the
PART I electron in joules.

1. A wire is gradually stretched by loading it 8. (a) On what principle does lighting in a


until it snaps. fluorescent tube operate?
(a) Sketch a load-extension graph for (b) State two factors which determine
the wire. the colour of light produced in a
(b) Indicate on the graph the fluorescent tube.
(i) elastic limit (E);
(ii) yield point (Y); 9. (a) Define electrolysis.
(iii) breaking point (B).
(b) Classify each of the following
substances as an electrolyte or a
non-electrolyte: 13. (a) The equation y = a sin (wt – kx)
(i) sugar solution; represents a plane wave traveling in
(ii) kerosene; a medium along the x-direction, y
(iii) alkaline solution; being the displacement at the point
(iv) lemon fruit juice. x at time t.
(i) Given that x is in metres and
10. (a) State Heisenberg’s Uncertainty t is in seconds, state the units
Principle. of k and w.
(b) State one phenomenon that can only (ii) What physical quantity does
be explained in terms of the wave w
/k represent? Justify your
nature of light. answer.
(iii) State whether the wave is
PART II traveling in the positive or
negative x-direction.
11. (a) A particle moves on a straight path (b) (i) What are beats
with (ii) A sonometer wire has a
An initial speed u and final speed v frequency of 259Hz. It is
in time t, Show that the total sounded alongside a tuning
distance x covered by the particle is fork of frequency 256Hz.
given by = ut + ½ at2 where a is Calculate the beat frequency.
the magnitude of acceleration. (iii) The sonometer wire in 13(b)
(b) State: (ii) above is under a tension
(i) Newton s second law of of 1200N. If a metre of the
motion; wire has a mass of 0.03kg,
(ii) the principle of conservation calculate the length l of the
of wire when it is vibrating in the
energy; fundamental mode.
(iii) the law of floatation. (c) List two similarities between the
human eye and the photographic
12. (a) State two differences between camera.
boiling and
evaporation. 14. (a) What is a magnetic field?
(b) A closed inexpansible vessel contains (b) With the aid of a labeled diagram
air saturated with water vapour at describe an experiment to show that
77oC. The total pressure in the vessel a magnetic field exists around a
is 1007 mmHg. Calculate the new straight wire carrying current.
pressure in the vessel if the (c) A 40µF capacitor in series with a 40Ω
temperature is reduced to 27oC. resistor is connected to a 100V, 50Hz
[The s.v.p. of water at 77oC and a.c. supply.
27oC respectively are 314mmHg and (i) Draw a circuit diagram of the
27mmHg. Treat the air in the vessel arrangement.
as an ideal gas]. (ii) Calculate the:
(c) The length of a zinc rod at 23oC is (I) impedance in the
200m. Calculate the increase in circuit;
length of the rod when its (II) current in the circuit;
temperature rises to 33oC (III) potential difference
[Linear expansivity of zinc = 2.6 x10- across the capacitor.
5
K-1]
(d) Explain why there is no temperature 15. (a) (i) State Einstein’s equation of
change when a solid being heated photoelectric effect.
changes into liquid at its melting (ii) What conservation principle
point. does the equation represent?
(b) List three applications of photocells.
(c) A photo emissive surface has a
threshold frequency of 4.02 x 1014Hz.
If the surface is illuminated by light
of frequency 5.0 x 1015Hz, calculate 2008
the: PAPER I PRACTICAL
(i) threshold wavelength;
(ii) work function; 1. (a) 0cm 15cm G Y 100cm
(iii) kinetic energy of the emitted
photoelectrons.
[c = 3.0 x 108 ms-1, h = 6.63 x
D L
10-34Js]

W M

You are provided with a uniform metre rule,


a knife edge, masses and other necessary
apparatus.
(i) Suspend the metre rule
horizontally on the knife
edge. Read and record the
point of balance G of the
metre rule. Keep the knife
edge at this point throughout
the experiment.
(ii) Using the thread provided,
suspend the object labeled W
at the 15cm mark of the
metre rule.
(iii) Suspend a mass M = 20g on
the other side of G. Adjust the
position of the mass until the
metre rule balances
horizontally again.
(iv) Read and record the position
Y of the mass M on the metre
rule.
(v) Determine and record the
distance L between the mass
and G. Also determine and
record the distance D
between W and G.
(vi) Repeat the procedure for
four other values of M = 30,
40, 50, and 60g. In each case,
ensure that W is kept
constant at the 15cm mark
and the knife edge at G.
(vii) Evaluate L-1 in each case.
Tabulate your readings.
(viii) Plot a graph of M on the
vertical axis against L-1 on the
horizontal axis.
(ix) Determine the slope, s, of the (x) Plot a graph of D on the
graph. vertical axis against d on the
(x) Evaluate s horizontal axis.
D (xi) Determine the slope, s, of the
(xi) State two precautions taken graph.
to obtain accurate results. (xii) Evaluate k = 8/4.
(b) (i) State the principle of (xiii) State two precautions taken
moments. to obtain accurate results.
(ii) Define centre of gravity. (b) (i) Explain the statement the focal
length of a
2. (a) converging lens is 15cm.
(ii) Distinguish between a real image
and a virtual image.

3. (a)

Using the above diagram as a guide, carry


out the following instructions.
(i) Fix a metre rule on the bench
with its graduated side facing
up.
(ii) Place the illuminated object at You are provide d with two resistance wires
the 0cm end and the screen labeled A and B, a 1Ω standard resistor RX
at the 100cm end of the rule and other necessary apparatus.
such that the distance d (i) Connect RX in the left hand
between the illuminated gap of the metre bridge, a
object and screen is 100cm. length L = 100cm of wire A in
(iii) Record the distance d. also the right hand gap and the
evaluate and record d2. other apparatus as shown in
(iv) Place and move the the diagram above.
converging lens between the (ii) Determine and record the
illuminated object and the balance point P on the bridge
screen until a sharp wire NQ.
diminished image of the
object is formed on the
(iii) Measure and record lx = NP
screen. and ly = PQ.
(v) Read and record the position (iv) Evaluate R1 = ly RX.
l1 of the lens. lX
(vi) Now move the lens towards (v) Repeat the procedure for
the object until another sharp four other values of L = 95,
image of the object is formed 85, 75 and 65cm. In each
on the screen. case, determine and record
the balance point P and the
(vii) Read and record the new lengths lx and ly. Also
position l2 of the lens. evaluate R1.
(viii) Evaluate and record L = (l1 – (vi) Repeat the experiment with
l2) also evaluate L2 and D = the second wire B. Obtain the
d1 – L2. balance points P and the
(ix) Repeat the procedure for values of lx and ly.
four values of d = 85, 75, 65 (vii) Evaluate R2 = ly RX.
and 55cm. Tabulate your lX
readings.
(viii) Plot a graph of R2 on the
vertical axis against R1 on the
horizontal axis.
(ix) Determine the slope, s of the
graph.
(x) Evaluate k = S
(xi) State two advantages of
using a potentiometer over a
voltmeter for measuring
potential difference.

(b) (i) State two advantages of


using a
Potentiometer over a
voltmeter for measuring
potential difference.
(ii) Define internal resistance of a
cell.

2009
SSCE PHYSICS
PAPER 2 OBJECTIVE

1. In which of the following states of matter do


the molecules vibrate about their mean
positions?
A. Liquids only
B. Solids only
C. Liquids and gases only
D. Solids, liquids and gases

2. Which of the following properties of a steel


bar can be measured in terms of the
dimension of length only?
A. Weight
B. Density
C. Pressure
D. Volume

3. In a hydraulic press, a force of 40 N is


applied to the smaller piston of area 10cm2.
If the area of the larger piston is 200cm2,
calculate the force obtained.
A. 800 N
B. 500 N
C. 80 N
D. 50 N

4. A body of volume 0.046m3 is immersed in a


liquid of density 980kgm-3 with ¾ of its
volume submerged. Calculate the upthrust
on the body. [g = 10ms-2]
A. 11.27 N
B. 33.81 N
C. 112.70 N
D. 338.10 N

5. Which of the following types of motion will


be produced when a pair of equal and
opposite non-collinear parallel forces acts 11. Two forces 6 N and 8 N, act eastwards and
on a body. northwards respectively on a body. Calculate
A. Vibrational motion
B. Random motion
the magnitude of their equilibrant.
A. 2N
C. Translational motion
B. 7N
D. Rotational motion
C. 10 N
D. 14 N
6. A body moving in a circle at constant speed
has
12. The period of a simple pendulum of length
I. a velocity tangential of the circle
80.0cm was found to have doubled when
II. a constant kinetic energy
the length of the pendulum was increased
III. an acceleration directed towards the
by X. Calculate X.
circumference of the circle
A. 26.7 cm
Which of the statements above are correct?
B. 40.0 cm
A. I and II only
C. 160.0 cm
B. II and III only
D. 240.0 cm
C. I and III only
13. A body of mass 5kg moving with a velocity
D. I, II and III
of 30ms-1 due east is suddenly hit by
another body and changes its velocity to
7. A car moves with a speed of 20ms-1.
50ms-1 in the same direction. Calculate the
Calculate the distance traveled in 30s.
magnitude of the impulse received.
A. 30 m
A. 100 Ns
B. 60 m
B. 150 Ns
C. 450 m
C. 250 Ns
D. 900 m
D. 400 Ns
8. A moving body accelerates when it
14. Electrical energy is measured in
A. changes its direction at constant speed
A. ampere
B. maintains a constant linear speed
B. coulomb
C. covers equal distances in equal times in a
C. kilowatt - hour
straight line
D. kilowatt
D. is acted upon by balanced forces
15. A stone of mass 0.5kg is dropped from a
9. the following properties of a solid are
height of 12m, calculate its maximum
measured on earth:
kinetic energy. [g = 10ms-2]
I. weight
A. 3.0 J
II. Specific heat capacity
B. 6.0 J
III. Density
C. 30.0 J
A. I and II only
D. 60.0 J
B. II and III only
C. I and III only
16. A machine of efficiency 80% is used to lift a
D. II, I, and III
box. If the effort applied by the machine is
twice the weight of the box, calculate the
10. Which of the vector diagram below
velocity ratio of the machine.
represents the following position vectors?
A. 0.50
B. 0.63
A. B.
C. 0.80
D. 1.60

17. A faulty thermometer registers 102.5oC at


100oC. If the thermometer has no zero error,
C. D.
what will it register at 55oC?
A. 54.6oC B. Barometer
B. 55.0oC C. Hypsometer
C. 56.0oC D. Hygrometer
D. 56.4oC
24. The maxim displacement of a vibrating
18. A rod of initial length 2m at a temperature tuning fork is its
of 25oC is heated to 80oC. Calculate the A. amplitude
increase in length of the rod if its linear B. frequency
expansivity is 4.0 x 10-3K-1. C. period
A. 0.26 m D. wavelength
B. 0.44 m
C. 0.53 m 25. A boat is rocked by waves of speed 30ms-1
D. 0.84 m whose successive crests are 10m apart.
Calculate the rate at which the boat
19. As a bicycle tyre was being pumped up, it receives the waves.
was noticed that contrary to Boyle’s law the A. 30 s-1
volume increased as the pressure B. 10 s-1
increased. The best explanation of this is C. 3 s-1
that Boyle’s law is only true for D. 1 s-1
A. ideal gases 26. Which of the following waves is not
B. a fixed mass of gases transverse?
C. a mixture of gases A. Light waves
D. a gas at variable temperature B. Sound waves
C. Sea waves
20. It takes 4 minutes to boil a quantity of water D. Radio waves
using an electrical heating coil. How long
will it take to boil the same quantity of 27. Light traveling through a small pinhole
water using the same heating coil if the usually does not make a shadow with a
current is doubled? [Neglect any external distinct sharp edge because of
heat losses] A. diffraction
A. 8 minutes B. interference
B. 4 minutes C. reflection
C. 2 minutes D. refraction
D. 1 minute
28. The angle of incidence of a ray of light on a
21. When table salt is added to ice, the melting plane mirror is 55o. Determine the angle
point of the ice between the reflected ray and the mirror.
A. is raised A. 35o
B. is lowered B. 45o
C. remains unchanged C. 55o
D. is first raised, then lowered D. 110o

22. Which of the following factors affect the rate 29. A concave mirror forms a magnified and
of evaporation of a liquid? erect image only when the object is placed
I. Exposed surface area A. at the centre of curvature
II. Temperature of the surroundings B. at the focus
III. Thermal capacity of the liquid C. between the focus and the pole of the
IV. Relative humidity of the surrounding mirror
A. II and III only D. beyond the radius of curvature
B. II, III and IV only
C. I, III and IV only 30. A thin lens is placed 50cm from an
D. I, II and IV only illuminated object. The image produced has
linear magnification of ¼. Calculate the
23. Which of the following instruments is used power of the lens in dioptres.
to measure relative humidity? A. 2.5 D
A. Hydrometer B. 5.0 D
C. 10.0 D 4π ε 0 ε rR
D. 25.0 D
37. The magnitude of the electric field intensity
31. Which of the following characteristics of at P is given by the expression
light determines its colour?
A. Velocity A. Q___
B. Wavelength 4π ε 0 ε rr2
C. Amplitude
D. Intensity B. Q______
4π ε 0 ε r(R + r)2
32. Which of the following is not a part of
electromagnetic-spectrum? C. Q_______
A. X-rays 4π ε 0 ε r(R – r)2
B. Microwaves
C. Infrared radiation D. Q___
D. Alpha rays 4π ε 0 ε rR2
38. A resistor of resistance R is connected to a
33. Sound interference is necessary to produce battery of negligible internal resistance. If a
the phenomenon of similar resistor is connected in series with it
A. beats the
B. Droppler effect A. effective resistance of the circuit is halved
C. acoustic resonance B. total power dissipated is doubled
D. echo C. total current in the circuit is halved
D. terminal voltage halfed
34. A satellite in circular motion around the
earth does not have 39. A cell of e.m.f. 1.5V is connected in series
A. a gravitational forces acting on it with a resistor of resistance 3.0A voltmeter
B. a uniform velocity connected across the cell registers 0.9V.
C. an acceleration Calculate the internal resistance of the cell.
D. a centripetal force acting on it A. 2.0Ω
B. 3.0Ω
35. When a conductor mounted on an insulating C. 5.0Ω
stand is charged and left for sometime, the D. 6.0Ω
conductor eventually loses all its charges.
This is because the 40. A wire of resistivity 4.40 x 10-5Ω cm has a
A. electric charges evaporate from the surface cross sectional area of 7.50 x 10-4cm-2.
of the conductor Calculate the length of this wire that will be
B. charges are conducted to earth required to make a 4.0Ω resistor
C. charges on the conductor are neutralized by A. 82.50 cm
opposite charges from the surrounding air B. 68.18cmΩ
D. charges ionize the surrounding air C. 15.90cm
D. 11.94cm
36. The electric potential at P is given by the
expression 41. A battery of e.m.f. 12.0V and internal
resistance 0.5Ω and 4.0 series resistors.
A. Q___ Calculate the terminal voltage of the
4π ε 0 ε rr battery?
A. 13.0 V
B. Q______ B. 11.0 V
4π ε 0 ε r(R + r) C. 3.0 V
D. 1.0 V
C. Q_______
4π ε 0 ε r(R – r) 42. If the direction of the current in a straight
wire is reversed, the magnetic field
D. Q___ A. remains the same
B. becomes parallel to the wire 48. Two isotopes of uranium are designated as
C. ceases to exist 238U and 235U. The numbers 238 and 235
D. is oppositely directed represent their
A. atomic numbers
43. An ideal step-down transformer steps up B. nucleon numbers
A. power C. proton numbers
B. energy D. neutron numbers
C. current
D. voltage 49. The main difference between x-rays and y-
rays lies in their
44. An inductor of inductance 1.0H is connected A. ionizing ability
in series with a capacitor of capacitance B. absorption rate
2.0µF in an a.c. circuit. Calculate the value C. mode of production
of frequency that will make the circuit to D. mode of propagation
resonate.
A. 112.5Hz 50. An electron of mass 9.1 x 10-31kg moves
B. 225.0Hz with a speed of 107ms-1. Calculate the
C. 353.5 Hz wavelength of the associated wave [h = 6.6
D. 707.21 Hz x 1034 Js]
A. 9.10 x 10-24m
45. an ammeter connected to an a.c. circult B. 4.55 x 10-17m
records 5.5A, the peak current in the circuit C. 7.25 x 10-11m
is D. 6.20 x 10-8m
A. 7.8 A 2009
B. 7.1 A PAPER 2 THEORY
C. 3.9 A SECTION B PART I
D. 3.5 A
1. A particle is dropped from a vertical height
The table below shows the energy h and falls freely for a time t. With the aid
distribution for various levels of an atom. of a sketch, explain how h varies with:
Use it to answer 46 and 47. (a) t; (b) t2

Energy level 1 2 3 4
(n) 2. A particle is projected horizontally at 15 ms-1
Energy (eV) – –3.39 – – from a height of 20m. Calculate the
13.6 1.51 0.85 horizontal distance covered by the particle
just before hitting the ground. [g = 10 ms-2]
[h = 6.6 x 10-34 Js, e = 1.6 x 10-19C; c = 3.0 x
108 ms-1]
3. Explain why mercury does not wet glass
46. Calculate the first excitation energy of the while water does.
atom.
A. 1.60 x 10-19 J 4. (a) Explain what is meant by cations
B. 1.02 x 10-18 J (b) Draw and label an electrolytic cell
C. 1.60 x 10-18 J
D. 1.02 x 10 J 5. State three methods of polarizing and
unpolarized light.
47. If the atom de-excites from n = 2,
wavelength of the emitted radiation is 6. (a) State faraday’s second law of
A. 2.4 x 10-7 m electrolysis
B. 1.2 x 10-7 m (b) An electric charge of 9.6 x 104C
C. 2.4 x 10-9 m liberates 1 mole of a substance
D. 1.2 x 10-9 m containing 6.0 x 1023 atoms.
Determine the value of the electronic
charge.
7. Explain the following terms: (b) State the effect of an increase in
(a) tensile stress; (b) Young’s pressure on the (i) boiling point; and
modulus (ii) melting point of water.
(c) S.V.P (mmHg)
8. (a) Define diffusion
(b) State two applications of electrical S.V.P. (mmHg)
conduction through gases.
760

9. (a) List two properties of cathode rays


(b) Explain how the intensity and energy 500
of cathode rays may be increased.

10. Give three observation in support of de


Broglie’s assumption that moving particles 50 100 Temp(oC)
behave like waves.
The graph shown above is that of the
11. (a) Give a retort stand and clamp, a saturated vapour pressure (svp) of water
stout pin, against temperature. Pure water is known to
a simple pendulum and a pencil, boil at 100oC and at an atmospheric
describe how you would use thee pressure of 760mmHg. What general
apparatus to determine the centre of conclusion can be drawn form the
gravity of an irregularly shaped piece information given above and from the
of cardboard of a moderate size. graph?

State two precautions necessary to (d) A thread of mercury of length 20cm


obtain accurate results. is used to trap some air in a
capilalary tube with uniform cross-
(b) Using a suitable diagram, explain sectional area and closed at one end.
how the following can be obtained With the tube vertical and the open
form a velocity-time graph: end uppermost, the length of the
(i) acceleration trapped air column is 15cm.
(ii) total distance covered Calculate the length of the air
(c) A body at rest is given an initial column when the tube is held
unform acceleration of 6.0ms-2 for (i) horizontally;
20s after which the acceleration is (ii) vertically with the open end
reduced to 4.0ms-2 for the next 10s. underneath. [Atmospheric
The body maintains the speed pressure = 76 cm Hg].
attained for 30s.
Draw the velocity-time graph of the 13. (a) State two differences between a
motion using the information given sound
above. From the graph, calculate wave and a radio wave.
the: (b) Explain why a vibrating tuning fork
(i) maximum speed attained sounds louder when its stem is
during the motion; pressed against a table top than
(ii) total distance traveled during when held in air.
the first 20s; (c) State two conditions necessary for
(iii) average speed during the the:
same time interval as in (ii) (i) production of stationary wave
above. in a medium;
(ii) formation of interference
12. (a) Explain why is not advisable to wave patterns;
sterilize a (iii) occurrence of total internal
Clinical thermometer in boiling water reflection of wave.
at normal atmosphere pressure. (d) A ray of light is incident on one face
of an equilateral glass prism.
(i) Draw a ray diagram to show
the path of the ray through
the prism.
(ii) Calculate the refractive index
of the glass if the angle of
minimum deviation is 41o.

14. (a) State two essential differences


between a
moving coil galvanometer and a.d.c
generator.
(b) Explain the term eddy currents and
state two devices in which the
current are applied.
(c) State the principle on which the
potentiometer is based when it is
functioning.
(d) A source of e.m.f. 110V and
frequency 60 Hz is connected to a
resistor, an inductor and a capacitor
in series. When the current in the
capacitor 2Λ, the potential
difference across the resistor is 80V
and that across the inductor is 40V.
Draw the vector diagram of the
potential differences across the
inductor, the capacitor and the
resistor. Calculate the:
(i) potential difference across
the capacitor;
(ii) capacitance of the capacitor;
(iii) inductance of the inductor.
[ π = 3.1114]

15. (a) Briefly explain the following terms:


(i) emission line spectra;
(ii) line absorption spectra 2009
(b) Draw a labeled diagram showing the PAPER I PRACTICAL
structure of a simple type of
photocell and explain its mode of 1. (a)
operation.
(c) State tow
(i) reasons to show that x-rays
are waves;
(ii) uses of x-rays other than in
medicine.
(d) An electron jumps from an energy
level of –1.6eV to one of –10,4 eV in
an atom. Calculate the energy and (i) You are provided with two
wavelength of the emitted radiation. metre rules and other
[h = 6.6 x 10-31Js; c = 3.00 x 108ms-1, necessary apparatus.
eV = 1.6 x 10-19J] (ii) Place one of the rules on a
knife edge and determine its
centre of gravity C. Mark this
position with a piece of chalk.
(iii) Read and record the mass MR thermometer and other
of the metre rule written on necessary materials.
the reverse side of it. (ii) Use the measuring cylinder
(iv) Attach the mass M = 100g provided to measure 100cm3
firmly to the rule AB at C of water and point it into the
using sellotape. tin labeled C.
(v) Suspend the metre rule by (iii) Heat the water in the tin
two parallel threads of length almost to boiling (90oC).
h = 40cm each at the 10cm (iv) Remove the tin and place it
and 90cm marks. Ensure that on a cork or wooden stand.
the graduated face of the
metre rule is facing upwards.
(v) Insert the thermometer into
the tin and record the
(vi) Set the rule AB into a small
temperature of water every
angular oscillation about the
minute starting from 85oC
vertical axis through its
until the temperature falls to
centre of gravity.
60oC.
(vii) Determine the time t for 20
complete oscillations.
(vi) Repeat the experiment with
the tin labeled D using
Evaluate the period T and T2.
exactly the same volume of
(viii) Read and record the value of
water and temperature range.
d in metres.
Tabulate your readings.
(ix) Keeping d constant
throughout the experiment,
(vii) On the same graph sheet and
using the same axes and
repeat the procedure for
scales, plot two graphs of
other values of h = 50, 60, 70
temperature on the vertical
and 80cm. In each case
axis and time on the
determine the corresponding
horizontal axis from the
values of t, T and T2. Tabulate
readings obtained using tins
your readings.
C and D.
(x) Plot a graph of T2 on the
vertical axis and h on the
(viii) Label the graph appropriated
as C and D to correspond with
horizontal axis.
the tins used.
(xi) Determine the slope s, of the
graph. (ix) From each graph, read off the
Evaluate k = s/Q; where Q = time taken to cool from 85oC
2/25d2 to 65oC.
(xi) State two precautions taken (x) State two precautions taken
to ensure accurate results. to ensure accurate results.
(b) Define the term couple as it relates
to rotational or oscillatory system. (b) (i) Explain how heat losses by
(ii) Give two practical radiation and convection are
applications of a couple in minimized in a vacuum flask.
everyday life. (ii) State four factors which
affect the rate of evaporation
2. (a) of a liquid in an open
container.

3. (a)

(i) You are provided with a


measuring cylinder, two
different tins labeled C and D,
(i) Connect the circuit as shown
in the diagram above. PQ is a
potentiometer wire 100cm
long and R is a standard
resistor of 5Ω.
(ii) With the jockey J not making
contact with PQ, close the
switch. Read and record the
ammeter reading I. Open the
switch.
(iii) Use the jockey to make
contact with PQ at the 20cm
mark such that PJ=I=20cm.
Close the switch, read and
record the value 11 of the
ammeter. Evaluate 1-1.
(iv) Repeat the procedure for
other values of l=35, 50, 65
and 80cm. In each case
determine the corresponding
values of Ii and l-1. Tabulate
your readings.
(v) Plot a graph of 1-1 on the
vertical axis and Ii on the
horizontal axis starting both
axes from the origin (0,0).
(vi) From your graph deduce the
value Io of I1 when I-1 = 0.z.
(vii) Evaluate 1o/I
(viii) State two precautions taken
to ensure accurate results.
(b) (i) Define the e.m.f. of a battery.
(ii) A cell X of e.m.f. 1.00V is
balance by a length of 40.0cm
on a potentiometer wire.
Another cell Y is balanced by
a length of 60.0cm on the
same wire. Calculate the
e.m.f. of Y.

2010
SSCE PHYSICS
PAPER 2 OBJECTIVE

1. Which of the following properties of a long


steel bar would alter if the bar were melted,
recast into a cube and allowed to cool to its
original temperature?
A. Density
B. Volume
C. Specific heat capacity
D. Electrical resistance
2. Which of the following instruments is the 8. The average speed of an object is
best for measuring the diameter of a thin determined by dividing the sum of its initial
constantan wire? and final speeds by two only when the
A. Callipers acceleration is
B. Metre rule A. changing
C. Micrometer screw gauge B. uniform
D. Venier callipers C. increasing
D. decreasing
3. What is the density of a fuel relative density
0.72? [Density of water = 1000kg m-3] 9. A body is projected horizontally from the top
A. 72kg m-3 of a cliff 45m above the ground. If the body
B. 720kg m-3 lands at a distance 30m from the foot of the
C. 7200kg m-3 cliff, calculate the speed of projection. [g =
D. 72000kg m-3 10ms-2]
A. 10ms-1
4. The types of motion which a cylindrical B. 15ms-1
drum rolling down an inclined plane C. 20ms-1
undergoes are D. 30ms-1
A. circular and translational
B. rotational and translational 10. Which of the following statements best
C. rotational and circular describes a rigid body? A body which
D. rectilinear and circular A. is not easily deformed
B. when deformed remains its shape
5. Which of the following statements about a C. is brittle
moving particle is correct? D. when deformed remains deformed
A. If the resultant force acting on the particle is
zero, then it is dynamic equilibrium 11. A simple harmonic oscillator has a period of
B. If the resultant force acting on the particle is 0.02s and amplitude of 0.25m. Calculate the
greater than zero, then it is moving with a speed in ms-1 at the centre of the oscillation.
uniform velocity A. 1.2 π
C. The rate of change of its momentum with B. 25.0 π
time takes place in a direction opposite to C. 100.0 π
that of its motion D. 400.0 π
D. The particle failing freely covers equal
distances in equal time intervals. 12. A vertical string, suspended from a fixed
point and having a small mass attached to
6. Which of the following statements about the free end is set into oscillations. Which of
static friction is correct? It the following statements about the system
A. is independent of the nature of surfaces in are correct?
contact I. The potential energy of the mass is a
B. depends on the area of surfaces in contact minimum at the middle of the swing
C. depends on the relative motion between the II. Its kinetic energy is a maximum at the
surfaces in contact middle of the swing
D. depends on the weight of the moving body III. The sum of the potential and kinetic
7. Two cars X and Y traveling in opposite energies is constant throughout the swing
directions along the same highway at A. I, II and III
uniform velocities 110kmh-1 and 90kmh-1 B. I and II only
respectively pass each other at a certain C. II and III only
point. The velocity of X relative to Y at the D. I and III only
time they pass each other is
A. 200kmh-1 13. A moving body of mass 25.0kg undergoes a
B. 100kmh-1 uniform retardation of 20.0ms-2. Calculate
C. 40kmh-1 the magnitude of the retaining force.
D. 20kmh-1 A. 1.25 N
B. 8.00N
C. 45.00 N
D. 500.00 N
D. i_ = constant
14. Which of the following sources of energy T PV
is/are exhaustible?
I. Solar 20. Specific heat capacity 26.4J kg-1k-1 of a
II. Fossil fuel body means that when 26.4 J of heat energy
III. Tidal power is supplied to a body of
A. II only A. mass 1000 kg, the temperature rises by 1K
B. III only B. mass 1kg, the temperature rises by 1000 K
C. I and II only C. mass 1kg, the temperature rises by 1K
D. II and III only D. mass 1000kg, the temperature rises by
1000K
15. A machine is said to be a third class lever
when the 21. In electric drill rated 400 W is used to drill a
A. load is between the fulcrum and effort hole in copper of mass 400g in 20s.
B. fulcrum is between the effort and load Calculate the rise in temperature if all the
C. effort is between the fulcrum and load heat produced is absorbed by the copper.
D. fulcrum is directly below the load [Specific heat capacity of copper =400 J kg-
1 -1
k ]
16. A thermometer records 690mmHg at steam A. 100oC
point and 440mm at ice point. The B. 75oC
temperature it records at 380mmHg is C. 50oC
A. –25oC D. 45oC
B. –20oC
C. 20oC 22. A solid of mass 0.5kg initially at 50oC melts
D. 25oC at 60oC. Calculate the quantity of heat
required to melt it completely.
17. The advantages of the thermoelectric [Specific heat capacity of the solid =1.0 kJ
thermometer include the following kg-1k-1]
characteristics except that it [Specific latent heat of fusion of solid =
A. can measure rapidly changing temperatures 2.0kJ kg-1]
B. can measure high and low temperatures A. 1500J
C. can measure temperature almost at a point B. 10000J
D. is fairly sensitive C. 10500J
D. 15000J
18. In a thermos flask, heat loss by radiation is
minimized by the 23. Dew point is not affected by
A. silvered surfaces A. temperature
B. vacuum within the double walls B. wind
C. plastic stopper C. the amount of water vapour in the
D. cork support atmosphere
D. atmospheric pressure
19. The press P, volume V and absolute
temperature T of a given mass of an ideal 24. A certain wave has a speed of 20ms-1. If the
gas, change simultaneously. Which of the frequency of the wave is 0.25Hz, calculate
following equations is correct about the the distance between successive crests of
gas? the wave.
A. 5.0m
A. PV = constant B. 40.0m
T C. 50.0m
D. 80.0m
B. P_ = constant
T 25. The ability of a wave to spread around
corners is called
C. P_ = constant A. polarisation
V T B. dispersion
C. diffraction 30. A lens that is thinner at the middle and
D. reflection thicker at the edges is
A. diverging
26. The following devices use plane mirrors in B. converging
their operations, except C. plano-convex
A. periscope D. converging meniscus
B. sextant
C. kaleidoscope 31. Which of the following electromagnetic
D. binoculars waves has the longest wavelength?
A. Radio wves
27. When an object is placed at the principal B. Gamma rays
focus of a concave mirror, the location of C. Infrared waves
the image formed is D. Ultraviolet rays
A. between principal focus and pole
B. between principal focus and centre of 32. The amplitude of a sound wave determines
curvature its
C. at infinity A. frequency
D. at centre of curvature B. quality
C. loudness
28. D. pitch

33. As the air column of length L in an air pipe


decreases, the frequency of the stationary
wave emitted.
A. decreases
B. increases
C. varies as L2
D. varies as L
From the diagram above, the correct
equation for the refractive index n of the
34. A positively charged glass rod is placed near
medium is
the cap of a positively charged
A. n = tan i
electroscope. The divergence of the leaf is
observed to
B. n = sin i__
A. decrease
Sin 90o
B. increase
C. remain the same
C. n= sin i____
D. increase and collapse immediately
Cos (90o–i)
35. Capacitors are used in the following devices,
D. n = sin 90o
except
A. water pumping machines
29. A ray of light travels from air to water. The
B. ceiling fans
refractive index of water is given by the
C. electric irons
expression.
D. television sets
A. velocity in water
velocity in air

B. sine of angle of refraction


Sine of angle of incidence
C. apparent depth
Uses the diagram below to answer question
real depth
36 and 37.
D. wavelength in air
wavelength in water
C. unlike poles facing each other and the
keepers placed at their ends
D. unlike poles facing each other and the
36. Calculate the current, I. keepers placed between them
A. 0.60A
B. 0.97A 43. The resonant frequency of an a.c. circuit is
C. 1.03A 1000kHz. If each of the capacitance and
D. 5.00A inductance in the circuit is reduced by 50%
and no other changes are made, the
37. Determine the potential difference, V across resonant frequency will become
the parallel resistors A. 250kHz
A. 2.0V B. 750kHz
B. 2.3V C. 1000kHz
C. 3.0V D. 2000kHz
D. 9.7V
44. The energy, E stored in an inductor of
38. A galvanometer of internal resistance 50Ω inductance L when current I passes through
and full scale deflection of 20mA is it is given by the equation
converted into a voltmeter by connecting a A. E = ½ Ll2
resistor of resistance 1950Ω in series to it. B. E = Ll2
Calculate the maximum voltage that can be C. E = 1/2 Li
read by the voltmeter. D. E = ½ L 2l
A. 20V
B. 30V 45. Eddy currents can be reduced by
C. 38V A. using coil of high resistance
D. 40V B. winding the coil on a soft iron core
C. creating holes in the metal plates
39. In the wiring of houses, the fuse is D. using coil of low resistance
connected to the wire coloured
A. blue 46. An electron moves with a speed of 2.0 x
B. brown 106ms-1 in a straight line. Calculate the
C. yellow wavelength of the electron wave.
D. yellow and green [mass of an electron = 9.1 x 10-31 kg]
[Planck’s constant = 6.6 x 10-34 Js]
40. Which of the following factors does not A. 3.63 x 10-10m
affect the electric resistance of a wire? B. 6.51 x 10-9m
A. Length C. 4.24 x 10-8m
B. Mass D. 7.25 x 10-8m
C. Temperature
D. Cross-sectional area 47. Oxidation of the filament in a light bulb is
prevented by the introduction of
41. Which of the following factors will increase A. hydrogen
the sensitivity of a moving coil meter? B. oxygen
A. Strong spring C. argon
B. Low number of turns D. mercury vapour
C. Small area of coil
D. Soft iron core 48. In a cathode ray tube, the function of the X-
plates is to
42. In other to prevent bar magnets from losing A. deflect the electron beam horizontally
their strength, they are stored with keepers B. deflect the electron beam vertically
such that the two bars are placed with C. reflect the electron beam
A. like poles facing each other and the keepers D. generate sinusoidal waves
placed at their ends
B. like poles facing each other and the keepers 49. A nuclide represented as 70X has a neutron
placed between them 32
proton ratio of
A. 0.5
B. 0.8
C. 1.2
D. 1.5

50. Neutrons are used to induce artificial


radioactivity because they
A. are energetic
B. have to charge
C. have no mass
D. are ionizing

2010
PAPER 2 THEORY
SECTION B PART I

1. A stone is projected vertically upward with a


speed of 30ms-1 from the top of a tower of
height 50m. Neglecting air resistance,
determine the maximum height it reached
from the ground. [g = 10ms-2]

2. A force of 40N is applied at the free of a


wire fixed at one end to produce an
extension of 0.24mm. If the original length
and diameter of the wire are 3m and 2.0mm (b) The product of the uncertainties in
respectively, calculate the: Heisenberg Uncertainty Principle is
(a) stress on the wire equal to or greater than a constant.
(b) strain in the wire State the mathematical expression
for this constant.
3. When a lead – acid accumulator is fully
charged, evolution of gases occurs at the PART II
electrodes, Name these gases and the
respective electrodes at which they are 11. (a) Distinguish between perfectly elastic
given off. collision and perfectly inelastic
collision.
4. (a) State two factors which affect the (b) Sketch a distance – time graph for a
mass particle moving in a straight line
of elements deposited during with:
electrolysis. (i) uniform speed
(b) List two non-electrolytes. (ii) variable speed
(c) A body start from rest and travels
5. (a) Differentiate between plane distance of 120, 300 and 180m in
polarization successive equal time intervals of
and interference as applied to 12s. During each interval the body is
waves. uniformly accelerated.
(b) List two uses of polariods. (i) Calculate the velocity of the
body at the of each
6. (a) List two examples each of successive time interval.
substances (ii) Sketch a velocity – time graph
with: for the motion.
(i) low viscosity
(ii) high viscosity 12. (a) Explain the terms:
(b) When is a liquid said to be (i) inertia
viscositatic? (ii) inertial mass
(b) List three factors which affect the
7. State one reach each why cathode rays: rate of evaporation of water in a
(a) are not electromagnetic waves pond
(b) cast sharp shadows of objects in (c) Two ice cubes pressed together for
their some time were found to stick
Path. together when the pressure was
(c) can rotate a light paddle wheel removed. Explain this observation.
inside a discharge tube (d) Two vertical capillary tubes of the
same diameter are lowered into
8. An X-ray tube operates at a potential of beakers situated at the same level,
2500V. If the power of the tube is 750W. containing liquids A and B of
calculate the speed of the electron striking densities 9.2 x 103kgm-3 and 1.30 x
the target. 103kgm-3 respectively. A suction
[e = 1.6 x 10-19C; mass of electron = 9.1x10- pump is used to withdraw air from
31
kg] the top of the liquid columns in the
tubes by means of a T – piece
9. (a) Write down the names of two arrangement until the liquid in A
particles rises to a height of 26.0cm. Calculate
used in explaining the wave nature the height of the liquid in tube B.
of matter.
(b) State the wave characteristics which 13. (a) State two factors which affect the
are exhibited by the particles named angle
in (a) above. of deviation of a ray of light through
10. (a) State the Heisenberg Uncertainty a triangular glass prism.
Principle (b) Seven virtual images of an object are
formed when two plane mirrors are
inclined at an angle θ to each other. n1 -5.7eV

Calculate the value of θ .


(c) By means of a ripple tank, a student n0 12.0eV

was able to generate series of The diagram above illustrates the energy
transverse waves by varying the levels of an electron in an atom. If an
frequency of the dipper and all the excited electron moves from n2 to n0
waves so generated covered a calculate the:
distance of 0.80m in 0.2s. (i) frequency
(i) Determine the speed, v, of (ii) wavelength of the emitted
the waves. Radiation.
[6.6 x 10-34Js; lev = 1.6 x 10-
f/Hz λ /m λ -1/m-1 19
J; C = 3.0 x 108 ms-1]
2.0 (c) The following nuclear equations
4.0 represent two types of radioactivity.
6.0
226
Ra → 222Rn + 4 α (Equation A)
88 86 2
8.0
10.0
14
N+4 α → 17
O+ 1
P (Equation B)
7 2 8 1

(ii) Copy and complete the table Identify each type and explain
given above in your answer briefly the difference between them.
booklet.
(iii) Plot a graph with f on the
vertical axis and λ -1 on the
horizontal axis.
(iv) What does the slope of the
graph represent?

14. (a)

The diagram above illustrates a 9.0V battery


of internal resistance 0.5Ω connected to two
resistors of values 2.0Ω and R. A1, D2 and A3
are ammeters of negligible internal
resistances. If A1 reads 4.0A, calculate the:
(i) equivalent resistance of the
combined resistors 2.0Ω and
RΩ.
(ii) current through A2 and A3
(iii) value of R.

15. (a) State three conclusions that can be


drawn from Rutherford’s experiment
on the scattering of alpha particles
by a thin metal foil in relation to the
structure of the atom.
(b) n3 0.0eV

n2 -2.0eV
(xv) Plot a graph with G on the
vertical axis and h on the
2010 horizontal axis, starting both
PAPER 1 PRACTICAL axes from the origin (0,0).
ALTERNATIVE A (xvi) Determine the intercept I on
the horizontal axis.
1. (a) (xvii) Evaluate A = 1___
19.772
(xviii) State two precautions taken
to obtain accurate results.
(b) (i) Distinguish between the
period
and frequency of oscillation of
a simple pendulum.
(ii) Differentiate between
oscillatory and rotational
(i) fix the drawing paper on the motions.
drawing board and hold the
board with two clamps such 2. (a)
that it is vertical.
(ii) Suspend the pendulum bob
such that it hangs freely in
front of the drawing paper.
(iii) Draw a line RP Representing
the rest position of the
pendulum string and mark
the position P of the
pendulum bob at rest.
(iv) Displace the pendulum bob to (i) Place the glass block on a
one side in a plane parallel to drawing sheet and trace its
the drawing board. outline ABCD as shown in the
(v) Mark the new position P1 of diagram above.
the centre of the bob. (ii) Remove the block, measure
(vi) Measure and record the and record the width W of the
perpendicular distance, d of block.
P1 from the line RP. (iii) Draw a normal ON to DC at a
(vii) Evaluate and record d2. point about one-quarter the
(viii) Measure and record the length of DC.
vertical height h of P1 above (iv) Draw a line making an angle
P. i= 10o with the normal.
(ix) Evaluate G = d2_ (v) Replace the block on its
h outline and mount the plan e
(x) Repeat the procedure for mirror vertically behind the
four other positions of P1. block such that it makes good
(xi) Tabulate your readings. contact with the face AB.
(xii) Remove the drawing board so (vi) Stick two points P1 and P2 on
that the pendulum bob can the line MO.
swing freely. (vii) Looking through the face CD,
(xiii) Set the pendulum bob stick two other pins P3 P4
oscillating through a small such that they appear to be in
amplitude and determine the a straight line with the
time t for 20 oscillations. images of pin P1 and P2 seen
(xiv) Determine and record the through the block.
period, T.
(viii) Join P3 and P4 with a straight (iii) Make contact with the
line and extend it to touch the potentiometer wire AB using
face CD at Ol. the jockey at a point C such
(ix) Draw a perpendicular line that AC = x = 20cm.
from the midpoint of OOl to (iv) Read and record the
meet AB at Q. voltmeter reading, V.
(x) Draw lines OQl, OlQ and (v) Evaluate x-1 and V-1.
normal Ol, Nl produced. (vi) Repeat the procedure for
(xi) Measure and record 0, e and other values of x=30, 40, 50,
d. 60 and 80cm.
(xii) Evaluate m = sin e and n = (vii) Tabulate your readings.
cos ( θ /2). (viii) Plot a graph with V-1 on the
(xiii) Repeat the procedure for i = vertical axis and x-1 on the
20o, 30o, 40o and 50o. horizontal axis, starting both
(xiv) Tabulate your readings. axes from the origin (0,0).
(xv) Plot a graph with m on the (ix) Determine the:
vertical axis and n on the (A) slope, s, of the graph
horizontal axis. (B) intercept, c, on the
(xvi) Determine the slope, s of the vertical axis
graph and evaluate θ = (x) State two precautions taken
2Ws. to ensure accurate results.
(xvii) State two precautions taken (b) (i) State two devices in which
to ensure accurate results. Ohm’s
law does not apply.
(b) (i) Explain the term refractive (ii) A current of 1A is supplied to
index two resistors of resistances
and give a mathematical 2Ω and 3Ω connected in
expression for it in terms of parallel. Calculate the current
wavelength. in each resistor.
(ii) State the conditions
necessary for total internal
reflection to occur for a given
pair of media.

3. (a)

You are provided with a voltmeter V, a


chemical cell/battery E two standard
resistors R1 and R2; a potentiometer AB; a
key K; a jockey and other necessary
materials.
(i) Set up a circuit as shown in
the diagram above.
(ii) Close the key K.

Das könnte Ihnen auch gefallen